GATE Exam 2020 Life Sciences (XL) Question Paper With Answer Key

GATE-2020

XL: Life Sciences

GA-General Aptitude

Q1 – Q5 carry one mark each.

1. Rajiv Gandhi Khel Ratna Award was conferred _____Mary Kom, a six-time world champion in boxing, recently in a ceremony _____ the Rashtrapati Bhawan (the President’s official residence) in New Delhi.

(A)  with, at

(B)  on, in

(C)  on, at

(D)  to, at

Answer: (C)

2. Despite a string of a poor performances, the changes of K. L. Rahul’s selection in the team are ______.

(A)  slim

(B)  bright

(C)  obvious

(D)  uncertain

Answer: (B)

3. Select the word that fits the analogy:

Cover : Uncover :: Associate : _______

(A)  Unassociate

(B)  Inassociate

(C)  Missassociate

(D)  Dissociate

Answer: (D)

4. Hig by floods, he kharif (summer sown) crops in various parts of the county have been affected. Officials believe that the loss in production of the kharif crops can be recovered in the output of the rabi (winter sown) crops so that the country can achieve its food-grain production target of 291 million tons in the crop year 2019-20 (July-June). They are hopeful that good rains in July-August will help the soil retain moisture for a longer period, helping winter sown crops such as wheat and pulses during the November-February period.

Which of the following statements can be inferred from the given passage?

(A)  Officials declared that the food-grain production target will be met due to good rains.

(B)  Officials want the food-grain production target to be met by the November-February period.

(C)  Officials feel that the food-grain production target cannot be met due to floods.

(D)  Officials hope that the food-grain production target will be met due to a good rabi produce.

Answer: (D)

5. The difference between the sum of the first 2n natural numbers and the sum of the first n odd natural numbers is ______.

(A)  n2 – n

(B)  n2 + n

(C)  2n2 – n

(D)  2n2 + n

Answer: (B)

Q6 – Q10 carry two marks each.

6. Repo rate is the at which Reserve Bank of India (RBI) lends commercial banks, and reverse repo rate is the rate at which RBI borrows money from commercial banks.

Which of the following statements can be inferred from the above passage?

(A)  Decrease in repo rate will increase cost of borrowing and decrease lending by commercial banks.

(B)  Increase in repo rate will decrease cost of borrowing and increase lending by commercial banks.

(C)  Increase in repo rate will decrease cost of borrowing and decrease lending by commercial banks.

(D)  Decrease in repo rate will decrease cost of borrowing and increase lending by commercial banks.

Answer: (D)

7. P, Q, R, S, T, U, V, and W are seated around a circular table.

(I) S is seated opposite to W.

(II) U is seated at the second place to the right of R.

(III) T is seated at the third place to the left of R.

(IV) V is a neighbor of S.

Which of the following must be true?

(A)  P is a neighbor of R.

(B)  Q is a neighbor of R.

(C)  P is not seated opposite to Q.

(D)  R is the left neighbor of S.

Answer: (C)

8. The distance between Delhi and Agra is 233 km. A car P started travelling from Delhi to Agra and another car Q started from Agra to Delhi along the same road 1 hour after the car P started. The two cars crossed each other 75 minutes after the car Q started. Both cars were travelling at constant speed. The speed of car P was 10 km/hr more than the speed of car Q. How many kilometers the car Q had travelled when the cars crossed each other?

(A)  66.6

(B)  75.2

(C)  88.2

(D)  116.5

Answer: (B)

9. For a matrix M = [mij], i. j= 1, 2, 3, 4, the diagonal elements are all zero and mij = −mij. The minimum number of elements required to fully specify the matrix is_______.

(A)  0

(B)  6

(C)  12

(D)  16

Answer: (B)

10. The profit shares of two companies P and Q are shown in the figure. If the two companies have invested a fixed and equal amount every year, then the ratio of the total revenue of company P to the total revenue of company Q, during 2013-2018 is ______.

(A)  15 : 17

(B)  16 : 17

(C)  17 : 15

(D)  17 : 16

Answer: (B)

XL: Life Sciences – P : Chemistry (Compulsory)

Q1 – Q5 carry one mark each.

1. An aqueous solution contains a mixture of 108 M NaCl and 108 M HCl.

Choose the correct statement about this solution.

(A)  The solution is a buffer with pH less than 7.00

(B)  The solution is a buffer with pH greater than 7.00

(C)  The solution is not a buffer but has its pH less than 7.00

(D)  The solution is not a buffer but has its pH greater than 7.00

Answer: (C)

2. The coordination complex which has a distorted octahedral structure is

(Given: Atomic numbers of V: 23; Mn: 25; Ni: 28; Cu: 29)

(A)  [Ni(H2O)6]2+

(B)  [Mn(H2O)6]2+

(C)  [V(H2O)6]2+

(D)  [Cu(H2O)6]2+

Answer: (D)

3. In naphthalene, the value of the integer “n” according to Huckel’s rule of aromaticity is ______.

Answer: (2 to 2)

4. The azimuthal quantum number (l) of an electron in the  orbital of a copper atom (atomic number: 29) is ______.

Answer: (2 to 2)

5. The standard enthalpy of reaction (in kJ mol1) for obtaining three moles of H2(g) from atomic hydrogen in gas phase is ______. (Given: Standard enthalpy of formation of atomic hydrogen is gas phase is 218 kJ mol1)

Answer: (-1308 to -1308)

Q6 – Q15 carry two marks each.

6. The correct order of the first ionization energies of He, B, N and O in their corresponding ground state is

(A)  He > N > O > B

(B)  O > N > B > He

(C)  He > B > N > O

(D)  N > O > B > He

Answer: (A)

7. Based on the molecular orbital theory, which one of the following statements with respect to N2, N2+, O2 and O2+ is correct?

(A)  Bond orders of N2 and O2 are higher than their corresponding cations.

(B)  Bond energy of N2+ is higher than that of N2, whereas bond energy of O2+ is lower than that of O2.

(C)  The unpaired electrons in N2+ and O2+ are present in σ and π* orbitals, respectively.

(D)  The bond in N2+ is shorter than that is N2, whereas bond in O2 is shorter than that in O2+.

Answer: (C)

8. Which one of the following statements is incorrect about the diborane molecule?

(A)  B-Ht bond is 2-centre-2-electron bond (Ht : terminal hydrogen).

(B)  BHbB bond is a 3-centre-2-electron bond (Hb: bridged hydrogen).

(C)  The  bond angle HtBHt is 122° (Ht : terminal hydrogen)

(D)  The B-Ht bond distance is longer than B-Hb bond distance (Ht : terminal hydrogen, Hb: bridged hydrogen).

Answer: (D)

9. Given below are Newman projections of ethylene glycol and 1, 2-difluoroethane about their respective C-C bonds. The most stable conformations (lowest energy) of ethylene glycol and 1, 2-difluoroethane are

(A)  I and III respectively.

(B)  I and IV respectively.

(C)  II and III respectively.

(D)  II and IV respectively.

Answer: (A)

10. In the reaction given below, choose the condition that gives an anti-Markovnikov’s product.

(A)  Peroxide/HCl

(B)  Aqueous mercuric acetate treatment

(C)  Diborane addition

(D)  Sulfuric acid addition

Answer: (C)

11. Which one of the following hexoses will give an osazone that has a different melting point from that of the osazone obtained from D (+) glucose?

Answer: (C)

12. A molecule in solution crystallizes into two different crystal forms with rate constants of 0.02 s1 and 0.13 s1. If the crystallization is assumed to be under kinetic control, then the half-life (in seconds, rounded off to one decimal place) of the molecular is ________.

Answer: (4.5 to 4.7)

13. The standard potential (Eθcell) for a cell reaction given below is +0.7 V. The standard reactions free energy (∆rGθ) for this cell is _____ kJ mol1 (correct up to two decimal places). (Given: Faraday constant, F = 96500 C mol1)

Au3+ (aq) + 3 Ag(s) → Au(s) + 3Ag+ (aq)

Answer: (-202.66 to -202.64)

14. The activation energy (Ea) estimated for a reaction from the Arrhenius equation is 21 kJ mol1. If the frequency factor is assumed to be independent of temperature, then the ratio of the rate constants determined at 298 K and 260 K is ______ (rounded off to two decimal places). (Given: Gas constant, R = 8.315 J K−1 mol−1)

Answer: (3.44 to 3.46)

15. At a given pressure, a substance is heated from 2000 K to 2600 K. If the entropy of the substance is 60 J K1 mol1, and is assumed to be constant over the given temperature range, then the change in the chemical potential (in kJ mol1) of the substance is _________.

Answer: (-36 to -36)

XL: Life Sciences – Q : Biochemistry

Q1 – Q10 carry one mark each.

1. Which one of the following hormones initiates a signaling cascade by directly binding to an intra-cellular receptor?

(A)  Insulin

(B)  Gonadotropin

(C)  Progesterone

(D)  Epinephrine

Answer: (C)

2. Which one of the following bonds is NOT present in ATP ?

(A)  Phosphoester

(B)  Phosphoanhydride

(C)  N-Glycosidic

(D)  α-Glycosidic

Answer: (D)

3. The reaction involved in the direct conversion of L-phenylalanine of L-tyrosine is

(A)  Hydroxylation

(B)  Decarboxylation

(C)  Transamination

(D)  Reduction

Answer: (A)

4. The humn major histocompatibility complex (MHC) is

(A)  Polygenic and monomorphic

(B)  Polygenic and polymorphic

(C)  Monogenic and polymorphic

(D)  Monogenic and monomorphic

Answer: (B)

5. Har Gobind Khorana and Marshall Nirenberg elucidated the genetic code by using a cell-free protein synthesizing system. It was found that poly(U) and poly(C) result in the synthesis of poly(L-Phe) and poly(L-pro), respectively. Based on these observations, which one of the following conclusions is correct?

(A)  Codon GGG specifies L-Phe and codon AAA specifies L-Pro

(B)  Codon CCC specifies L-Phe and codon UUU specifies L-pro

(C)  Codon AAA specifies L-Phe and codon GGG specifies L-Pro

(D)  Codon UUU specifies L-Phe and codon CCC specifies L-Pro

Answer: (D)

6. Binding of an antibody to its cognate antigen does NOT involve

(A)  Covalent bonds

(B)  Electrostatic foces

(C)  Van der Waals forces

(D)  Hydrogen bonds

Answer: (A)

7. A globular protein of molecular weight 50 kDa exists as a mixture of monomers and dimmers in solution. The most appropriate technique for the separation of these two forms of the protein is

(A)  Thin layer chromatography

(B)  Ion exchange chromatography

(C)  Gel filtration chromatography

(D)  Paper chromatography

Answer: (C)

8. Choose the correct order of molecules according to their ability to diffuse across as lipid bilayer.

(A)  CO2 > H2O > Glucose > RNA

(B)  CO2 > Glucose > H2O > RNA

(C)  RNA > Glucose > CO2 > H2O

(D)  H2O > CO2 > RNA > Glucose

Answer: (A)

9. When one glucose unit from glycogen gets converted to lactate in the muscle, the ne number of ATP molecules produced is _______.

Answer: (3 to 3)

10. Considering that the three pKas of histidine are pK1 = 1.8, pK2 = 9.2 and pKR = 6.0, its isoelectric point will be ______ (rounded off to one decimal place).

Answer: (7.5 to 7.7)

Q11-  Q20 carry two marks each.

11. One mole of a native protein upon N-terminal analysis yielded one mole each of Asp and Val. Therefore, the protein in its native state exists as a

(A)  Monomer

(B)  Homo-dimer

(C)  Hetero-dimer

(D)  Tetramer

Answer: (C)

12. The prosthetic groups/cofactors involved in both 1e and 2e transfer in the mitochondrial electron transport chain are

(A)  NAD and NADP

(B)  NAD and FAD

(C)  Heme and FMN

(D)  Coenzyme Q and FMN

Answer: (D)

13. Match the items in Group I with the most appropriate items in Group II and choose the correct option.

(A)  P-2, Q-1, R-3, S-4

(B)  P-4, Q-1, R-3, S-2

(C)  P-1, Q-2, R-3, S-4

(D)  P-4, Q-1, R-2, S-3

Answer: (B)

14. The correct combination of glycosidic linkages present in glycogen is

(A)  α 1 → 4 and α 1 → 6

(B)  α 1 → 4 and β 1 → 6

(C)  α 1 → 6 and β 1 → 4

(D)  α 1 → 6 and β 1 → 6

Answer: (A)

15. Polypeptides are either biosynthesized on the ribosomes using an mRNA template or chemically synthesized by the Merrifield’s solid phase method. The correct directions of peptide synthesis are

(A)  C→ N direction on the ribosomes and N → C direction on the solid phase

(B)  N → C direction on the ribosomes and C → N direction on the solid phase

(C)  N → C direction in both cases

(D)  C → N direction in both cases

Answer: (B)

16. A solution absorbs 20% of the incident light in a cuvette of path length 1.0 cm. The amount of light transmitted by the same solution in a curvette of 3.0 cm path length is _______% (rounded off to one decimal place).

Answer: (51.1 to 51.3)

17. The second pKa of phosphoric acid is 6.8. The ratio of Na2HPO4 to NaH2PO4 required to obtain a buffer of pH 7.0 is _______ (rounded off to two decimal places).

Answer: (1.58 to 1.60)

18. A PCR in a 100 μL reaction volume, containing two primers at a concentration of 0.2 μM each, is set up to amplify a 250 base pair DNA fragment. Consider the average molecular weight of one base pair as 660 Da. If the primers are fully consumed by the end of the reaction, the amount of the final PCR product formed is ________μg (rounded off to one decimal place).

Answer: (3.2 to 3.4)

19. An enzyme obeying Michaelis-Menten kinetics shows as reaction velocity (v) of 10 μmol/min when the substrate concentration [S] equals its KM. The maximal velocity Vmax for this enzyme is _______ μmol/min (correct to integer number). (KM is Michaelis-Menten constant)

Answer: (20 to 20)

20. The enzyme glucose isomerase catalyzes the inter-conversion of glucose and fructose as shown.

The ∆G’0 for this reaction is zero kcal/mol. After adding glucose isomerase  to a 0.12 M glucose solution and allowing the reaction to attain equilibrium, the final concentration of fructose in the reaction mixture will be _____ mM.

Answer: (60 to 60)

XL: Life Sciences – R: Botany

Q1 – Q10 carry one mark each.

1. Indefinite stamen is a characteristic feature of which of the following plant families?

(A)  Malvaceae

(B)  Apocynaceae

(C)  Poaceae

(D)  Brassicaceae

Answer: (A)

2. In natural condition, which of the following plants DOES NOT exhibit anomalous secondary growth?

(A)  Rice

(B)  Aloe

(C)  Yucca

(D)  Dracaena

Answer: (A)

3. In a typical angiosperm under natural condition, primary meristems are usually established during

(A)  Gametogenesis

(B)  Embryogenesis

(C)  Vegetative phase development

(D)  Secondary growth

Answer: (B)

4. 2-Methoxy-3, 6-dichlorobenzoic acid belongs to which class of plant growth regulators?

(A)  Synthetic auxin

(B)  Synthetic cytokinin

(C)  Strigolactone

(D)  Brassinosteroid

Answer: (A)

5. In a typical green plant, the first product of Calvin cycle is

(A)  Oxaloacetic acid

(B)  Succinic acid

(C)  Maleic acid

(D)  3-Phosphoglyceric acid

Answer: (D)

6. Among the following, which best describes an organism that lives at the expense of other organisms, harmful but usually not killing?

(A)  Predator

(B)  Symbiotic

(C)  Prey

(D)  Parasite

Answer: (D)

7. The oleo-gum resin asafoetida (hing) is obtained from the cut surface of

(A)  Stem

(B)  Root

(C)  Leaf

(D)  Fruit

Answer: (B)

8. ‘Bakanae’ disease or ‘foolish seedling’ disease is caused by

(A)  Fungus

(B)  Bacterium

(C)  Virus

(D)  Mycoplasma

Answer: (A)

9. Which of the following chemicals is used for doubling of chromosome numbers during production of ‘doubled haploids’ in crop plants?

(A)  Hygromycin

(B)  Kanamycin

(C)  Colchicine

(D)  Glufosinate

Answer: (C)

10. An mRNA of a nuclear encoded plant gene, DSH20 has an ORF of 1353 nucleotides. Provided that average molecular weight of amino acid is 110 Dalton (Da) calculated molecular weight of DSH20 protein in kDa (round off to 1 decimal place) is ______

Answer: (49.5 to 49.5)

Q11 – Q20 carry two marks each.

11. Group I, Group II and Group III represent enzyme, product of the enzymatic reaction, and metabolic process, respectively.

The CORRECT combination fro Group I, Group II and Group III is

(A)  P-ii-1, Q-iv-3, R-v-2, S-iii-4

(B)  P-ii-1, Q-i-3, R-v-4, S-iii-2

(C)  P-ii-2, Q-v-3, R-i-4, S-iii-1

(D)  P-iii-1, Q-i-3, R-iv-4, S-ii-2

Answer: (B)

12. Match the following in CORRECT combination between Group I and Group II with reference to the agents that interfere with oxidative phosphorylation

(A)  P-iv, Q-i, R-ii, S-iii

(B)  P-v, Q-i, R-iii, S-iv

(C)  P-iv, Q-iii, R-ii, S-v

(D)  P-v, Q-ii, R-iii, S-iv

Answer: (A)

13. In relation to Agrobacterium mediated genetic engineering in plants, match the following in CORRECT combination

(A)  P-iv, Q-iii, R-ii, S-v

(B)  P-ii, Q-i, R-iii, S-v

(C)  P-i, Q-ii, R-iii, S-iv

(D)  P-iii, Q-i, R-ii, S-iv

Answer: (C)

14. Match the plant part (Group I) with the product obtained (Group II) and the representative plant species (Group III) in CORRECT combination

(A)  P-ii-1, Q-i-3, R-iv-2, S-iii-4

(B)  P-ii-1, Q-i-2, R-iv-4, S-iii-3

(C)  P-ii-2, Q-iv-3, R-i-4, S-iii-1

(D)  P-iv-3, Q-i-2, R-ii-4, S-iii-1

Answer: (D)

15. Select the CORRECT combination by matching the disease, causal organism and the affected

(A)  P-iv-1, Q-iii-3, R-i-2, S-ii-4

(B)  P-ii-1, Q-i-6, R-iv-4, S-iii-2

(C)  P-iii-1, Q-iv-3, R-i-5, S-ii-4

(D)  P-iv-1, Q-ii-3, R-iii-2, S-i-5

Answer: (A)

16. Match the following alkaloids with their uses and source plants in CORRECT combination

(A)  P-ii-3, Q-iv-1, R-iii-4, S-i-2

(B)  P-ii-1, Q-i-3, R-iv-4, S-iii-2

(C)  P-ii-2, Q-iv-1, R-i-4, S-iii-3

(D)  P-iii-4, Q-ii-1, R-iv-3, S-i-2

Answer: (A)

17. Match the following ecological terms with their appropriate definitions

(A)  P-i, Q-ii, R-iv, S-iii

(B)  P-ii, Q-iv, R-i, S-iii

(C)  P-iv, Q-iii, R-i, S-ii

(D)  P-iii, Q-i, R-ii, S-iv

Answer: (B)

18. Arrange the following ‘water reservoirs of earth’ in decreasing order of water volume

P-Streams

Q-Groundwater

R-Glaciers

S-Lakes and inland seas

(A)  R-Q-S-P

(B)  P-Q-R-S

(C)  S-P-R-Q

(D)  R-P-Q-S

Answer: (A)

19. Selection markers and the corresponding genes used in plant genetic engineering are given below

Choose the CORRECT combination

(A)  P-ii, Q-i, R-iv, S-iii

(B)  P-iv, Q-ii, R-i, S-iii

(C)  P-iv, Q-i, R-ii, S-iii

(D)  P-iii, Q-iv, R-ii, S-i

Answer: (C)

20. A double homozygous mutant develops green and wrinkled seeds. When it was crossed with true-breeding plant having yellow and round seeds, all the F1 plants developed yellow and round seeds. After self-fertilization of F1, the calculated percentage probability of plants with green and wrinkled seeds in the F2 population (round off to 2 decimal places) is ________

Answer: (6.25 to 6.25)

XL: Life Sciences – S: Microbiology

Q1 – Q10 carry one mark each.

1. The technique of microbial “pure culture” was pioneered by

(A)  Edward Jenner

(B)  Louis Pasteur

(C)  Robert Hooke

(D)  Robert Koch

Answer: (D)

2. The antibacterial trimethoprim is an inhibitor of

(A)  dihydrofolate reductase

(B)  dihydropteroate synthetase

(C)  N5, N10-methenyl tetrahydrofolate synthetase

(D)  serine hydroxymethyl transferase

Answer: (A)

3. Choose the correct taxonomical hierarchy among the following:

(A)  Species, Genus, Family, Order, Class, Phylum, Domain

(B)  Species, Genus, Order, Class, Family, Phylum, Domain

(C)  Species, Genus, Order, Family, Class, Phylum, Domain

(D)  Species, Genus, Family, Class, Order, Phylum, Domain

Answer: (A)

4. Shifting a Sacharomyces cerevisiae culture from fermentative to aerobic respiratory mode will

(A)  decrease carbon dioxide production

(B)  increase alcohol production

(C)  increase glucose consumption

(D)  decrease ATP generation per mole of glucose

Answer: (A)

5. Which one of the following diseases is treated by a neuraminidase inhibitor?

(A)  Chickenpox

(B)  Polio

(C)  Influenza

(D)  Japanese encephalitis

Answer: (C)

6. Which one of the following does NOT provide three-dimensional images?

(A)  Atomic force microscopy

(B)  Confocal scanning laser microscopy

(C)  Differential interference contrast microscopy

(D)  Phase-contrast microscopy

Answer: (D)

7. Which one of the following will increase the resolution of a light microscope ?

(A)  Decreasing the numerical aperture of the objective lens

(B)  Using an objective lens with a longer working distance

(C)  Using a medium of higher refractive index

(D)  Increasing the wavelength of light

Answer: (C)

8. Which one of the following conditions favors maximum expression of lac operon genes in coli?

(A)  Glucose-low, lactose-law, cAMP-high

(B)  Glucose-high, lactose-low, cAMP-high

(C)  Glucose-low, lactose-high, cAMP-high

(D)  Glucose-high, lactose-high, cAMP-low

Answer: (C)

9. Match the cellular organelle in Group I with its function in Group II

(A)  P-3, Q-2, R-1, S-4

(B)  P-3, Q-4, R-1, S-2

(C)  P-1, Q-2, R-4, S-3

(D)  P-3, Q-1, R-4, S-2

Answer: (B)

10. A 250 μl of bacteriophage stock containing 8 × 108 phages/ml is added to 500 μl of coli culture containing 4 × 108 cells/ml. The multiplicity of infection is _______.

Answer: (1 to 1)

Q11 – Q20 carry two marks each.

11. Digestion of an immunoglobulin G (IgG) molecule with pepsin will NOT

(A)  generate a bivalent antigen binding fragment

(B)  generate monovalent antigen binding fragments

(C)  destroy the complement binding site

(D)  cleave the heavy chain of IgG molecule

Answer: (B)

12. Match the process involved in nitrogen or sulfur cycle in Group I with the corresponding microbe in Group II.

(A)  P-2, Q-3, R-4, S-1

(B)  P-2, Q-1, R-3, S-4

(C)  P-3, Q-4, R-1, S-2

(D)  P-3, Q-1, R-2, S-4

Answer: (D)

13. Determine the correctness or otherwise of the following Assertion [a] and the Reason [r].

Assertion [a]: Diphtheria exotoxin is an example of A-B type toxin.

Reason [r]: The A component of the toxin is released from the host cell while the B component inhibits protein synthesis and kills the host cell.

(A)  Both [a] and [r] are true and [r] is the correct reason for [a]

(B)  Both [a] and [r] are true but [r] is not the correct reason for [a]

(C)  Both [a] and [r] are false

(D)  [a] is true but [r] is false

Answer: (D)

14. Which one of the following statements about control of microbial growth is NOT correct?

(A)  Nonionizing radiation leads to thymine dimmers formation in DNA

(B)  Spirochetes and mycoplasma can pass through membrane filters (0.22-0.45 μm)

(C)  Use of high concentration of salts and sugars to preserve food is a chemical method of microbial control

(D)  Thermoduric bacteria can survive pasteurization

Answer: (C)

15. An example of a differential and selective medium in which colonies of Gram-negative bacteria produce large amounts of acidic products and appear green with a metallic sheen is

(A)  Blood agar

(B)  EMB agar

(C)  MacConkey agar

(D)  Mannitol salt agar

Answer: (B)

16. Which one of the following is an example of substrate level phosphorylation?

(A)  Glucose to Glucose 6-phosphate

(B)  Fructose 6-phosphate to Fructose 1, 6-biphosphate

(C)  1, 3-bisphosphoglycerate to 3-phosphoglycerate

(D)  2-phosphoglycerate to Phosphoenolpyruvate

Answer: (C)

17. A bacterial culture containing 3 × 105 live cells was exposed to a newly developed sterilizing agent. After 30 minutes of exposure, 3 live cells remained in culture. The decimal reduction time (in minutes) for the new agent is ______.

Answer: (6 to 6)

18. A bacterial culture has a generation time of 34 minutes. The time taken (in minutes, rounded off to two decimal places) for the OD550 of this exponentially growing culture to increase from 0.25 to 0.85 is ______.

Assume that OD550 has a linear relationship with the cell density.

Answer: (58.50 to 61.50)

19. A 100 μl aliquot (104 dilution) of the bacterial culture plated on the nutrient agar gave 4 colonies. The bacterial stock concentration (in million cells/ml, rounded off to one decimal place) is _____.

Answer: (0.4 to 0.4)

20. A continuous bacterial culture carried out in a chemostat is set to a flow rate of 40 ml/hr. The culture volume is equivalent to that of a cubical container having 10 cm sides. The dilution rate (in hr1, rounded off to two decimal places) of this system is ________.

Answer: (0.04 to 0.04)

XL: Life Sciences – T: Zoology

Q1 – Q10 carry one mark each.

1. Which ONE of following leucocytes is phagocytic and has clear cytoplasm?

(A)  Eosinophil

(B)  Monocyte

(C)  TH-lymphocyte

(D)  Basophil

Answer: (B)

2. Which ONE of the following techniques can be used for detecting the subcellular localization of serotonin receptor in intact cells?

(A)  Immunoelectron microscopy

(B)  SDS-PAGE

(C)  Fluorescence in-situ hybridization

(D)  Differential centrifugation

Answer: (A)

3. Which ONE of the following is NOT a site for in situ conservation?

(A)  Biosphere reserve

(B)  Wild life sanctuary

(C)  Zoological garden

(D)  Biodiversity hotspot

Answer: (C)

4. Which ONE of the following is the precursor molecule for corticosteroids?

(A)  Androgen

(B)  Estrogen

(C)  Pregnenolone

(D)  Mineralocorticoids

Answer: (C)

5. Transitional epithelia is found in which ONE of the following organs?

(A)  Liver

(B)  Lung

(C)  Brain

(D)  Urinary bladder

Answer: (D)

6. Visual signal transduction cascade is activated by rhodopsin and involves degradation rather than synthesis of which ONE of the following second messenger molecules?

(A)  cAMP

(B)  IP3

(C)  cGMP

(D)  DAG

Answer: (C)

7. The genomes of both human and Drosophila code for an amylase that acts on the same substrate. However, the sequence of nucleotides in the genes encoding the two is dissimilar. This is an example of which ONE of the following types of evolution?

(A)  Neutral

(B)  Directional

(C)  Convergent

(D)  Divergent

Answer: (C)

8. “Round dance” is performed by forager bees to indicate the distance between a food source and their colony. Which ONE of the following best represents this distance?

(A)  45 meters

(B)  450 meters

(C)  1000 meters

(D)  More than 2000 meters

Answer: (A)

9. Which ONE of the following phyla have choanocytes?

(A)  Ctenophora

(B)  Nematoda

(C)  Cnidaria

(D)  Porifera

Answer: (D)

10. Which ONE of the following glial cells is NOT derived from the ectoderm?

(A)  Astrocytes

(B)  Microglial cells

(C)  Oligodendrocytes

(D)  Ependyma

Answer: (B)

Q11 – Q20 carry two marks each.

11. Tarantulas and mosquitoes both belong to the phylum Arthropoda. Which ONE of the following represents the correct number of legs in them respectively?

(A)  6 and 6

(B)  6 and 8

(C)  8 and 8

(D)  8 and 6

Answer: (D)

12. Match the following subcellular organelles in Column I with associated functions in Column II

(A)  P-(iii), Q-(ii), R-(i), S-(iv)

(B)  P-(i), Q-(ii), R-(iii), S-(iv)

(C)  P-(iv), Q-(ii), R-(i), S-(iii)

(D)  P-(ii), Q-(iii), R-(i), S-(iv)

Answer: (C)

13. Match the following genetic disorders in Column I with associated typical chromosomal changes mentioned in Column II

(A)  P-(iv), Q-(iii), R-(ii), S-(i)

(B)  P-(iv), Q-(ii), R-(i), S-(iii)

(C)  P-(iii), Q-(iv), R-(ii), S-(i)

(D)  P-(iii), Q-(iv), R-(i), S-(ii)

Answer: (D)

14. Match the following components listed in Column I with their respective organs in Column II

(A)  P-(ii), Q-(iv), R-(i), S-(iii)

(B)  P-(ii), Q-(i), R-(iv), S-(iii)

(C)  P-(iii), Q-(iv), R-(i), S-(ii)

(D)  P-(iii), Q-(iv), R-(ii), S-(i)

Answer: (A)

15. Match the following digestive enzymes in Column I with their respective functions in Column II.

(A)  P-(iv), Q-(iii), R-(ii), S-(i)

(B)  P-(iv), Q-(iii), R-(i), S-(ii)

(C)  P-(iii), Q-(iv), R-(i), S-(ii)

(D)  P-(iii), Q-(iv), R-(ii), S-(i)

Answer: (B)

16. Which ONE of the following graphs represents the relationship between ventricular end-diastolic volume and cardiac output in a healthy adult individual at rest (solid line) and upon exercise (dotted line)?

Answer: (A)

17. Match the household insect vectors in Column I with their associated diseases in Column II.

(A)  P-(iv), Q-(iii), R-(ii), S-(i)

(B)  P-(iii), Q-(ii), R-(i), S-(iv)

(C)  P-(i), Q-(iv), R-(iii), S-(ii)

(D)  P-(iii), Q-(iv), R-(ii), S-(i)

Answer: (D)

18. Match the proteins in Column I with the organs in which they are maximally expressed in Column II.

(A)  P-(iv), Q-(i), R-(iii), S-(ii)

(B)  P-(iii), Q-(iv), R-(ii), S-(i)

(C)  P-(iv), Q-(iii), R-(ii), S-(i)

(D)  P-(i), Q-(ii), R-(iii), S-(iv)

Answer: (C)

19. The graph below shows the activity of enzyme pepsin in the presence of inhibitors aliphatic alcohols (P) or N-acetyl-1-phenylalanine (Q). Which ONE of the following represents the nature of inhibition by P and Q, respectively?

(A)  Non-competitive and competitive

(B)  Competitive and non-competitive

(C)  Non-competitive and uncompetitive

(D)  Competitive and uncompetitive

Answer: (B)

20. In Drosophila, the red eye phenotype (W) is dominant over the recessive white eye mutant (w). In a mixed population of red and white eye flies of 10,000 individuals, 3,600 flies were white eyed. The percentage of the heterozygous red eye flies in this population is ________.

Answer: (48 to 48)

XL: Life Sciences – U : Food Technology

Q1 – Q10 carry one mark each.

1. The enzyme majorly involved in postmortem degradation of muscle proteins is

(A)  Trypsin

(B)  Calpin

(C)  Transglutaminase

(D)  Pepsin

Answer: (B)

2. Which of the following is the correct pair of essential fatty acid?

(A)  Oleic acid and Lenoleic acid

(B)  Lenoleic acid and Linolenic acid

(C)  Linolenic acid and Lauric acid

(D)  Linolenic acid and Oleic acid

Answer: (B)

3. Nisin A is produced by

(A)  Aspergillus niger

(B)  Acttobacter aceti

(C)  Lactobacillus lactis

(D)  Clostridium perfringens

Answer: (C)

4. Which of the following bacteria will stain purple color after Gram staining?

(A)  Bacillus subtilis

(B)  Escherichia coli

(C)  Pseudomonas aeruginaosa

(D)  Yersinia pestis

Answer: (A)

5. The enzyme system used for removal of glucose from egg white prior to its drying consists of

(A)  Glucose oxidase and Catalase

(B)  Glucosidase and Glucoisomerase

(C)  Glucoisomerase and Catalase

(D)  Gluccoamylase and Glucose oxidase

Answer: (A)

6. The INCORRECT pair of food borne illness and its causative microorganism is

(A)  Brucellosis-Brucella Sp.

(B)  Peptic ulcers-Bacillus subtilis

(C)  Bubonic plague-Yersinia pestis

(D)  Q fever-Coxiella burnatii

Answer: (B)

7. Which of the following is commonly used as preservative in the tomato sauce?

(A)  Sodium sulphite

(B)  Potassium sorbate

(C)  Potassium sulphite

(D)  Sodium benzoate

Answer: (D)

8. A fluid with flow behvaviour index less than one (n < 1) is

(A)  Dilatant

(B)  Pseudoplastic

(C)  Bingham plastic

(D)  Newtonian

Answer: (B)

9. The velocity of 2.2 μm diameter fat particles inside a cetribfuge, running at 6000 rpm and 20°C, is 0.25 mm s1. The velocity of 1.5 μm diameter fat particles inside the same centrifuge running at 7500 rpm and same temperature (round off to 2 decimal places) will be ____ mm s1.

Answer: (0.15 to 0.21)

10. The initial population of a bacterial strain increases from 1 × 104 cells per mL to 1 × 106 cells per mL in 120 minutes. The generation time for this strain (round off to 2 decimal places) is ______ minutes.

Answer: (17.00 to 19.00)

Q11 – Q20 carry two marks each.

11. Match the protein in Column I with its food source in Column II.

(A)  P-4, Q-1, R-2, S-3

(B)  P-4, Q-3, R-1, S-2

(C)  P-2, Q-3, R-1, S-4

(D)  P-2, Q-4, R-1, S-3

Answer: (D)

12. Match the carbohydrate in Column I with corresponding enzyme used for its hydrolysis in Column II.

(A)  P-3, Q-2, R-1, S-4

(B)  P-2, Q-4, R-1, S-3

(C)  P-1, Q-2, R-3, S-4

(D)  P-4, Q-3, R-1, S-2

Answer: (A)

13. Match the edible oil refining stage in Column I with its purpose in Column II.

(A)  P-3, Q-1, R-2, S-4

(B)  P-1, Q-4, R-2, S-3

(C)  P-4, Q-3, R-1, S-2

(D)  P-3, Q-4, R-2, S-1

Answer: (D)

14. Match the food material in Column I with its related term in Column II.

(A)  P-4, Q-2, R-1, S-3

(B)  P-3, Q-4, R-1, S-2

(C)  P-3, Q-4, R-2, S-1

(D)  P-1, Q-3, R-4, S-2

Answer: (B)

15. Match the components/system in Column I with the peeling method for fruits and vegetables in Column II.

(A)  P-4, Q-3, R-2, S-1

(B)  P-3, Q-4, R-1, S-2

(C)  P-4, Q-3, R-1, S-2

(D)  P-3, Q-4, R-2, S-1

Answer: (C)

16. Which among the given options correctly explains the nature of the microbial culture represented by lines 1, 2 and 3 in the following figure?

Answer: (B)

17. Match the equation/law in Column I with its application in Column II.

(A)  P-1, Q-3, R-4, S-2

(B)  P-2, Q-3, R-1, S-4

(C)  P-2, Q-3, R-4, S-1

(D)  P-4, Q-3, R-1, S-2

Answer: (C)

18. Match the absorber used in modified atmosphere packaging and storage in Column I with the scavenger in Column II.

(A)  P-2, Q-2, R-4, S-1

(B)  P-1, Q-2, R-4, S-3

(C)  P-2, Q-3, R-4, S-1

(D)  P-3, Q-2, R-1, S-4

Answer: (A)

19. During extrusion cooking, food materials are generally subjected to a combination of

(A)  high shear and low pressure

(B)  high temperature and high shear

(C)  low shear and high temperature

(D)  low shear and low pressure

Answer: (B)

20. An orange juice flowing at 0.80 kg s1 enters a counter current double pipe heat exchanger at 20°C and leaves at 72° Inlet and outlet temperatures of the hot water used as heating medium in the exchanger are 81°C and 74°C, respectively. The specific heat of the orange juice is 3.74 kJ kg1 K1 and overall heat transfer coefficient is 492 W m2 K1. The heat transfer surface area (round off to 2 decimal places) will be ______ m2.

Answer: (11.00 to 14.00)

GATE Exam 2020 Engineering Sciences (XE) Question Paper With Answer Key

GATE-2020

XE: Engineering Sciences

GA-General Aptitude

Q1 – Q5 carry one mark each.

1. Rajiv Gandhi Khel Ratna Award was conferred _____Mary Kom, a six-time world champion in boxing, recently in a ceremony _____ the Rashtrapati Bhawan (the President’s official residence) in New Delhi.

(A)  with, at

(B)  on, in

(C)  on, at

(D)  to, at

Answer: (C)

2. Despite a string of a poor performances, the changes of K. L. Rahul’s selection in the team are ______.

(A)  slim

(B)  bright

(C)  obvious

(D)  uncertain

Answer: (B)

3. Select the word that fits the analogy:

Cover : Uncover :: Associate : _______

(A)  Unassociate

(B)  Inassociate

(C)  Missassociate

(D)  Dissociate

Answer: (D)

4. Hig by floods, he kharif (summer sown) crops in various parts of the county have been affected. Officials believe that the loss in production of the kharif crops can be recovered in the output of the rabi (winter sown) crops so that the country can achieve its food-grain production target of 291 million tons in the crop year 2019-20 (July-June). They are hopeful that good rains in July-August will help the soil retain moisture for a longer period, helping winter sown crops such as wheat and pulses during the November-February period.

Which of the following statements can be inferred from the given passage?

(A)  Officials declared that the food-grain production target will be met due to good rains.

(B)  Officials want the food-grain production target to be met by the November-February period.

(C)  Officials feel that the food-grain production target cannot be met due to floods.

(D)  Officials hope that the food-grain production target will be met due to a good rabi produce.

Answer: (D)

5. The difference between the sum of the first 2n natural numbers and the sum of the first n odd natural numbers is ______.

(A)  n2 – n

(B)  n2 + n

(C)  2n2 – n

(D)  2n2 + n

Answer: (B)

Q6 – Q10 carry two marks each.

6. Repo rate is the at which Reserve Bank of India (RBI) lends commercial banks, and reverse repo rate is the rate at which RBI borrows money from commercial banks.

Which of the following statements can be inferred from the above passage?

(A)  Decrease in repo rate will increase cost of borrowing and decrease lending by commercial banks.

(B)  Increase in repo rate will decrease cost of borrowing and increase lending by commercial banks.

(C)  Increase in repo rate will decrease cost of borrowing and decrease lending by commercial banks.

(D)  Decrease in repo rate will decrease cost of borrowing and increase lending by commercial banks.

Answer: (D)

7. P, Q, R, S, T, U, V, and W are seated around a circular table.

(I) S is seated opposite to W.

(II) U is seated at the second place to the right of R.

(III) T is seated at the third place to the left of R.

(IV) V is a neighbor of S.

Which of the following must be true?

(A)  P is a neighbor of R.

(B)  Q is a neighbor of R.

(C)  P is not seated opposite to Q.

(D)  R is the left neighbor of S.

Answer: (C)

8. The distance between Delhi and Agra is 233 km. A car P started travelling from Delhi to Agra and another car Q started from Agra to Delhi along the same road 1 hour after the car P started. The two cars crossed each other 75 minutes after the car Q started. Both cars were travelling at constant speed. The speed of car P was 10 km/hr more than the speed of car Q. How many kilometers the car Q had travelled when the cars crossed each other?

(A)  66.6

(B)  75.2

(C)  88.2

(D)  116.5

Answer: (B)

9. For a matrix M = [mij], i. j= 1, 2, 3, 4, the diagonal elements are all zero and mij = −mij. The minimum number of elements required to fully specify the matrix is_______.

(A)  0

(B)  6

(C)  12

(D)  16

Answer: (B)

10. The profit shares of two companies P and Q are shown in the figure. If the two companies have invested a fixed and equal amount every year, then the ratio of the total revenue of company P to the total revenue of company Q, during 2013-2018 is ______.

(A)  15 : 17

(B)  16 : 17

(C)  17 : 15

(D)  17 : 16

Answer: (B)

XE: Engineering Sciences – A: Engineering Mathematics (compulsory)

Q1 – Q7 carry one mark.

1. Let A be a 4 × 3 non-zero matrix and let b be a 4 × 1 column vector. Then Ax = b has

(A)  a solution for every b.

(B)  no solution for some b.

(C)  a solution only when b = 0.

(D)  a solution if b and the columns of A form a linearly independent set.

Answer: (B)

2. Let x0, x1, x2, … be the sequence generated by the Newton-Raphson method applied to the function f(x) = x3 – 2x + 2 with x0 = 1 . Then the sequence

(A)  converges to 0.

(B)  becomes unbounded.

(C)  converges to a root of f(x).

(D)  does not converge.

Answer: (D)

3. Let z(t) be the solution of the initial value problem

If the planar curve parameterized by t having x-coordinate z(t) and y-coordinate  is closed, then necessarily

(A)  b > 0.

(B)  b < 0.

(C)  b = 0.

(D)  b is a non-zero rational number.

Answer: (B)

4. Let z be a complex number. Then the series 

(A)  converges for all z.

(B)  converges for |z| ≤ 1 and diverges for |z| > 1.

(C)  converges for z = 0 and diverges for any z ≠ 0.

(D)  converges for |z| < 1 diverges for |z| ≥ 1.

Answer: (A)

5. Let  be a vector field whose curl is zero. Then necessarily

(A)  a = b  c.

(B)  a = −b = c.

(C)  b = c.

(D)  b = −c.

Answer: (D)

6. Let f(x) be a continuous function on the real line such that for any x,  Then f(2) is ______.

Answer: (5 to 5)

7. The number of points at which the function  has local minima is _____.

Answer: (2 to 2)

8. Let f(t) be a real-valued differentiable function on (−1, 1) such that f(0) = 0 and 

Then the series   

(A)  converges but not absolutely.

(B)  is unbounded.

(C)  converges absolutely.

(D)  is bounded but does not converge.

Answer: (C)

9. Let X be a random variable with probability density function 

Let 0 < a < b. Then the probability P(X ≤ b \ X ≥ a) depends only on

(A)  b – a.

(B)  b

(C)  a

(D)  a + b.

Answer: (A)

10. Let A be a 3 × 3 matrix such that A2 = A. Then it is necessary that

(A)  A is the identity matrix or the zero matrix.

(B)  the determinant of A4 is either 0 or 1.

(C)  the rank of A is 3.

(D)  A has one imaginary eigenvalue.

Answer: (B)

11. Players A and B take turns to throw a fair dice with six faces. If A is the first player to throw, then the probability of B being the first one to get a six is _____ (round off to two decimal places).

Answer: (0.43 to 0.47)

XE: Engineering Sciences – B: Fluid Mechanics

Q1 – Q9 carry one mark each.

1. Figures given below show the velocity and shear stress profiles for the flow in a duct. In each option, ‘1’ represents velocity profile and ‘2’ represents shear stress profile.

Choose the correct option that closely represents the turbulent flow condition.

Answer: (A)

2. The variation of shear stress (r) against strain rate (du/dy) is given in the Figure. identify the line/curve among P, Q, R and S, that represents an ideal fluid.

(A)  S

(B)  P

(C)  Q

(D)  R

Answer: (B)

3. A body is under stable equilibrium in a homogeneous fluid, where CG and CB are center of gravity and center of buoyancy, respectively.

Two statements ‘P’ and ‘Q’ are given below:

P: For a fully submerged condition, CG should always be below CB

Q: For a floating body, CG need not be below CB

Choose the option that is valid for the present situation.

(A)  P is false; Q is True when metacentre is below CG

(B)  P is false; Q is True when metacentre is above CG

(C)  P is true; Q is True when metacentre is below CG

(D)  P is True; Q is true when metacentre is above CG

Answer: (D)

4. A laminar hydrodynamics boundary layer over a smooth flat plate is shown in the Figure. The shear stress at the wall is denoted by τw. Which one of the following conditions is correct.

(A)  Pressure is varying along ‘x’ and (τw)x1 > (τw)x2

(B)  Pressure is constant along ‘x’ and (τw)x2 > (τw)x1

(C)  Pressure is constant along ‘x’ and (τw)x1 > ((τw)x2

(D)  Pressure is varying along ‘x’ and (τw)x2 > (τw)x1

Answer: (C)

5. A non-dimensional number known as Weber number is used to characterize which one of the following flows.

(A)  Motion of fluid in open channel

(B)  Motion of fluid droplets

(C)  Motion of fluid at high velocity

(D)  Motion of fluid through a pipe

Answer: (B)

6. A uniform approach flow is subjected to an unsteady and periodic flapping plate as shown in the Figure. Tracer is released to obtain flow visualization lines, which are marked as ‘P’, ‘Q’ and ‘R’.

Choose the correct option that the line ‘R’ represents

(A)  Streakline

(B)  Streamline

(C)  Pathline

(D)  Timeline

Answer: (A)

7. The volume flow between any two points not lying on the same streamline in a flow field is equal to

(A)  Change in strain rate between the points

(B)  Change in vorticity between the points

(C)  Change in potential function between the points

(D)  Change in stream function between the points

Answer: (D)

8. A liquid flow through a horizontal smooth pipe the diameter 5 cm and discharges into a collection tank of dimension 50 cm × 50 cm × 50 cm. Time taken for a 10 cm rise of liquid level in the collection tank is 40 s.

The flow velocity in the pipe is ——– m/s (rounded off to two decimal places).

Answer: (0.30 to 0.33)

9. The potential function for a two dimensional incompressible flow filed is given as:

Magnitude of the velocity vector at point (2, 1) is ______m/s

Answer: (5 to 5)

Q10 – Q22 carry two marks each.

10. Column I represents a list of elementary plane flows and Column II represents flow past geometry obtained by superposition of these elementary plane flows.

The correct match between Columns I and II is,

(A)  P-3; Q-2; R-1; S-4

(B)  P-1; Q-2; R-3; S-4

(C)  P-3; Q-4; R-1; S-2

(D)  P-1; Q-4; R-3; S-2

Answer: (C)

11. The velocity field for a flow is  where t is time. Choose the correct option representing the total acceleration at (x, y, z, t)

Answer: (B)

12. An incompressible viscous fluid is placed between two infinite horizontal parallel plates as shown in the Figure. The plates move in opposite direction with constant velocities U1 and U2. The pressure gradient in the x-direction is zero and the only body force is due to the fluid weight. The flow is steady, laminar and two-dimensional. Assume velocity component in ‘y’ direction to be zero.

The correct expression for the velocity distribution between the plates is:

Answer: (A)

13. The stream function of a flow field is Ψ = k(x2 – y2x) where k is a constant. Which one of the following represents the voriticity?

(A)  -2k

(B)  2k(x + 1)

(C)  2k(x – 1)

(D)  -2k(x + 1)

Answer: (C)

14. Consider a two dimensional, incompressible steady flow of a Newtonian fluid in which the velocity field is u = −2xy, v = y2 – x2. Pressure gradients in the x-and y-directions are

Answer: (A)

15. A hydroelectric power plant takes in 30 m3/s of water through its turbine and discharges it to the atmosphere with V = 2 m/s. The total head loss in the turbine and penstock system is 20 m. (Assume turbulent flow with kinetic energy correction factor as 1.1. Density of water is 1000 kg/m3 and acceleration due to gravity, g is 10 m/s2).

The net  head available to the turbine for power generation is —————- m.

(rounded off to one decimal place).

Answer: (79.60 to 79.90)

16. Water flows at an average velocity, v of 10 m/s through a horizontal smooth tube of diameter, d 5 cm. The friction actor, f is 0.02. Head loss is obtained using Darcy-Weisbach relation The fluid pressure, p measured at various stations are reported in the table below. The length of the pipe, l between station 0 and station 6 is 6 m.

If acceleration due to gravity, g = 10 m/s2 and density of water = 1000 kg/m3, then the fluid pressure at station 6 is ________ kPa (rounded off to one decimal place)

Answer: (183.5 to 184.5)

17. A sphere model of 10 cm diameter is tested in water flowing at 2 m/s. The drag force is measured as 5 N. Prototype of 1.5 m in diameter is tested in air with dynamic similarity conditions. (Density of water is 1000 kg/m3, density of air is 1.2 kg/m3, viscosity of water is 0.001 Ns/m2 and viscosity of air is 1.78 × 105 Ns/m2).

Drag force experienced by the prototype is ______ N (rounded off to two decimal places).

Answer: (1.21 to 1.35)

18. A liquid of viscosity 1.74 × 103 Ns/m2 is flowing through a horizontal capillary tube of diameter 0.5 mm. The flow in the tube is steady, incompressible, and fully developed laminar flow. The pressure drop across two locations spaced 1 m apart in the tube is 1.0 MPa.

The flow rate in the tube _______ mm3/s.

Answer: (857 to 924)

19. A venturimeter with 75 mm diameter throat is placed in a 150 mm diameter pipeline carrying water at 25° The pressure drop between the upstream tap and the venture throat is 40 kPa. (Density of water = 1000 kg/m3).

The flow rate is ______ m3/s (rounded off to three decimal places).

Answer: (0.035 to 0.045)

20. A water jet with velocity  impinges normal to a moving flat plat with velocity  such that the jet splits equally into two halves as shown in Figure. The jet cross-sectional area is 2 cm2,  is 20 m/s and  is 10 m/s and density of water is 1000 kg/m3. Consider steady flow and neglect weight of the jet, weight of the plate and frictional losses.

The absolute value of the force required to keep the plate moving at constant velocity  is ____ N.

Answer: (20 to 20)

21. In an inverted manometer (as shown in the Figure), the pressure difference, pB – pA is 100 kPa.

Use specific gravity of oil as 08, density of water as 1000 kg/m3, density of mercury as 13600 kg/m3 and acceleration due to gravity as 10 m/s2.

The height of the water column, H is _____ cm. (rounded off to one decimal place).

Answer: (30.5 to 34.5)

22. An incompressible, steady flow with uniform velocity condition at the inlet between parallel plates is shown in Figure. The flow develops into a parabolic laminar profile with u = ay(y0 – y) at the downstream end, where ‘a’ is a constant. Assume unit depth of the plate. For U0 = 7.5 cm/s, y0 = 3 cm and the fluid with density, ρ = 800 kg/m3

The value of ‘a’ is _____.

Answer: (5 to 5 OR 500 to 500)

XE: Engineering Sciences – C: Materials Science

Q1 – Q9 carry one mark each.

1. A Pb-Sn sample of eutectic composition, containing α-and β-phases, is examined in a scanning electron microscope. The α-phase contains ~97% Pb (atomic number 82) while β-phase contains ~99 wt% Sn (atomic number 50). The ratio of number of backscattered electrons from α-phase to that from β-phase would be:

(A)  Less than 1

(B)  Equal to 1

(C)  Greater than 1

(D)  Equal to 0

Answer: (C)

2. Smallest or minimum feature size that can be theoretically resolved in an optical microscope does NOT depend on:

(A)  Refractive index of the medium between the lens and the focal point

(B)  Intensity of radiation

(C)  Wavelength of radiation

(D)  Numerical aperture of the objective lens

Answer: (B)

3. Following diagram shows a square 2-D lattice with a hexagonal motif(dark colored). The rotational symmetry element that must be present in the system is:

(A)  Six-fold rotation

(B)  Two-fold rotation

(C)  Three-fold rotation

(D)  Four-fold rotation

Answer: (B)

4. Density of states, D(E), in a three dimenstional solid varies with energy (E) as

(A)  E1/2

(B)  E0

(C)  E1/2

(D)  E3/2

Answer: (A)

5. The variation of molar volume (Vm) of a liquid showing glass transition temperature (Tg) while cooling from its melting temperature ™ is depicted by:

(A)  I

(B)  II

(C)  III

(D)  IV

Answer: (A)

6. Find the correct match between polymer name in Column I and the monomer type in Column II.

(A)  I-P, II-S, III-R, IV-Q

(B)  I-R, II-Q, III-S, IV-P

(C)  I-S, II-P, III-Q, IV-R

(D)  I-S, II-R, III-Q, IV-P

Answer: (C)

7. A ceramic has a fracture toughness (KIc) of 1 MPa.m1/2. If this ceramic is to be exposed to a maximum stress (σ) of 200 MPa, the maximum value of half crack length ‘a’ (in micrometer, μm), below which the material does not fail, is _____ μm (round off to one decimal place). Loading condition for the sample is shown in the schematic. Assume geometrical factor f = 1.2.

Answer: (5.3 to 5.7)

8. A ceramic material is periodically heated and cooled between 25°C and a higher temperature, Tf. During thermal cycling, the material remains dimensionally constrained. The material can withstand a maximum compressive stress of 200 MPa without failure. Material’s coefficient of thermal expansion is 7.5 × 106 °C1 and modulus of elasticity (E) is 200 GPa. The lowest value of T­f(in °C) at which material will fails is _______ °C (round-off to the nearest integer). Assume that there is no plastic deformation during thermal cycling.

Answer: (157 to 159)

9. During homogeneous solidification of a liquid metal, the radius of critical nucleus (in nanometer, nm) at a temperature TS which is below the melting point (Tm), is ______ nm (round-off to one decimal place). Given that γsl (solid liquid interfacial energy) is 0.18 J.m2 and ∆Gv (change in volume free energy upon transformation from liquid to solid) at TS is 0.18 × 109m3.

Answer: (1.29 to 2.1)

Q10 – Q22 carry two marks each.

10. Read the two statements related to sintering and select the correct option.

Statement-1: Sintering in vacuum leads to improved densification as compared to sintering under ambient (at atmospheric pressure) condition.

Statement-2: Closed pores formed during sintering inhibit full densification.

(A)  Both Statement-1 and Statement-2 are FALSE

(B)  Both Statement-1 and Statement-2 are TRUE

(C)  Statement-1 is TRUE but Statement-2 is FALSE

(D)  Statement-1 is FALSE but Statement-2 is TRUE

Answer: (B)

11. Select the correct option that appropriately matches the process to the material/product that can be fabricated using them.

(A)  I-S, II-P, III-R, IV-Q

(B)  I-S, II-R, III-Q, IV-P

(C)  I-S, II-P, III-Q, IV-R

(D)  I-P, II-R, III-Q, IV-S

Answer: (C)

12. Consider a FCC structured metal with lattice parameter a = 3.5 Å. If the material is irradiated using X-rays of wavelength λ = 1.54056 Å, the Bragg angle (2θ) corresponding to the fourth reflection will be:

(A)  88.21°

(B)  76.99°

(C)  99.35°

(D)  93.80°

Answer: (D)

13. The number of Schottky defects per mole of KCl at 300°C under equilibrium condition will be:

Given:

Activation energy for the formation of Schottky defect = 250 kJ.mol1

Avogadro number = 6.023 × 1023 mol1

Universal Gas Constant = 8.314 J.K1.mol1

(A)  1.21 × 1018

(B)  1.52 × 1016

(C)  9.75

(D)  2.42 × 1012

Answer: (D)

14. In an industry, the probability of an accident occurring in a given month is 1/100. Let P(n) denote the probability that there will be no accident over a period of ‘n’ months. Assume that the events of individual months are independent of each other. The smallest integer value of ‘n’ such that P(n) ≤ 1/2 is _____ (round off to the nearest integer).

Answer: (69 to 69)

15. For a FCC metal, the ratio of surface energy of {111} surface to {100} surface is _____ (round-off to two decimal places). Assume that only the nearest neighbor broken bonds contribute to the surface energy.

Answer: (0.84 to 0.90)

16. Pure silicon (Si) has a band gap (Eg) of 1.1 eV. This Si is doped with 1 ppm (part per million) of phosphorus atoms. Si contains 5 × 1028 atoms per m3 in pure form. At temperature T = 300 K, the shift in Fermi energy upon doping with respect to intrinsic Fermi level of pure Si will be ______ eV (with appropriate sign and round-off to two decimal places).

Intrinsic carrier concentration of Si, ni, is given as:

Given:

Mass of an electron, m = 9.1 × 1031 kg

Charge of an electron, e = 1.6 × 1019 C

Boltzmann constant, kB = 1.38 × 1023 J.K1

Planck’s constant, h = 6.6 × 1034 J.s1

Answer: (0.33 to 0.45)

17. The schematic diagram shows the light of intensity I0 incident on a material (shaded grey) of thickness, x, which has an absorption coefficient, α and reflectance, R. The intensity of transmitted light is I. The reflection of light (of a particular wavelength) occurs at both the surfaces (surfaces indicated in the diagram). The transmittance is estimated to be _______ (round-off to three decimal places).

Given that for the wavelength used, α = 103 m1 and R = 0.05.

Answer: (0.330 to 0.334)

18. Fe3O4 (also represented as FeO.Fe2O3) is a FCC structured inverse spinel (AB2O4) material where 1/8 of tetrahedral sites are occupied by half of B cations and 1/2 of the octahedral sites are occupied by remaining B and A cations. The magnetic moments of cations on octahedral sites are antiparalel with respect to those on tetrahedral sites. Atomic number of Fe is 26 and that of O is 8. The saturation magnetic moment of Fe3O4 per formula unit in terms of Bohr magnetons (μB) will be _____ μB. Ignore contribution from orbital magnetic moments.

Answer: (4 to 4)

19. A piezoelectric ceramic with piezoelectric coefficient (d­zz) value of 100 × 1012N1 is subjected to force Fz, of 10 N, applied normal to its x-y face, as shown in the figure. If relative dielectric constant (εr) of the material is 1100, the voltage developed along the z-direction of the sample will be _______ Volts (round-off to two decimal places). Ignore any nonlinear effects.

Given: Permittivity of free space (ε0) is 8.85 × 1012 F.m1.

Answer: (0.97 to 1.09)

20. Silicon carbide (SiC) particles are added to Aluminum (Al) matrix to fabricate particle reinforced Al-SiC composite. The resulting composite is required to possess specific modulus (E/ρ; E: elastic modulus, ρ: density) three times that of pure Al. Assuming iso-strain condition, the volume fraction of SiC particles in the composite will be _______ (round-off to two decimal places).

Answer: (0.36 to 0.45)

21. Isothermal weight gain per unit area (∆W/A, where ∆W is the weight gain (in mg) and A is the area (in cm2)) during oxidation of a metal at 600°C follows parabolic rate law, where, ∆W/A = 1.0 mg.cm2 after 100 min of oxidation. The ∆W/A after 500 min at 600°C will be _______ mg.cm2 (round-off to two decimal places).

Answer: (2.20 to 2.28)

22. A plain carbon steel sample containing 0.1 wt% carbon is undergoing carburization at 1100°C in a carbon rich surroundings with fixed carbon content of 1.0 wt% all the time. The carburization time necessary to achieve a carbon concentration of 0.46 wt% at a depth of 5 mm at 1100°C is ______ hour (round off to the nearest integer).

Given: Diffusivity of carbon in iron at 1100°C is 6.0 × 1011 m2.s1 and

Answer: (77 to 83)

XE: Engineering Sciences – D: Solid Mechanics

Q1 – Q9 carry one mark each.

1. Which among the following statements is true for a body moving on a dry surface under the action of applied forces?

(A)  Kinetic-friction force is zero.

(B)  Kinetic-friction force is equal to the static-friction force.

(C)  Kinetic-friction force is greater than the static-friction force.

(D)  Kinetic-friction force is lower than the static-friction force.

Answer: (D)

2. Consider an isotropic material with Young’s modulus E and Poisson’s ratio v. The bulk modulus of this material is given by_____.

Answer: (C)

3. A body subjected to ______ does not undergo change in volume.

(A)  uniform tension

(B)  pure shear

(C)  pure bending

(D)  hydrostatic pressure

Answer: (B)

4. The angular momentum of a particle moving under a central force is

(A)  zero

(B)  constant in both magnitude and direction.

(C)  constant in magnitude but not direction.

(D)  constant in direction but not magnitude.

Answer: (B)

5. According to Euler-Bernoulli beam theory, which one of the following statements best describes the state of a beam subjected to pure bending?

(A)  Transverse shear stress and transverse shear strain are zero.

(B)  Transverse shear stress is not zero but transverse shear strain is zero.

(C)  Transverse shear stress is zero but transverse shear strain is not zero.

(D)  Transverse shear stress and transverse shear strain are not zero.

Answer: (A)

6. A rigid square ABCD is subjected to planar forces at the corners as shown.

For this planar force system, the equivalent force couple system at corner A can be represented as

(A)  System I

(B)  System II

(C)  System III

(D)  System IV

Answer: (B)

7. A particle of mass 0.1 kg, which is released from rest, falls vertically downward under gravity in a fluid. The fluid offers a resistive force, which is linearly proportional to the particle velocity with 0.1 N.s/m as the constant of proportionality. The uniform gravitational acceleration is 10 m/s2 throughout the trajectory of the particle. The magnitude of the particle velocity (in m/s) at time 1 s after release (rounded off to two decimal places) is _______.

Answer: (6.25 to 6.40)

8. The state of two-dimensional plane stress at a point in a body is shown on the triangular element ABC, where cos θ = 3/5 and sin θ = 4/5. The normal stress (in MPa) on the plane AC is ________.

Answer: (90 to 90)

9. Consider two point masses m = 10 kg and M = 30 kg connected by a massless inextensible string passing over a massless and frictionless pulley with radius a = 100 mm as shown. The masses are released from rest and move vertically under the action of gravity. Let acceleration due to gravity, g = 10 m/s2. The tension (in N) in the string is_______.

Answer: (150 to 150)

Q10 – Q22 carry two marks each.

10. The cantilever beam AC is composed of two segments AB and BC that are rigidly connected at B. The flexural rigidity of the segment AB is EI, whereas, the flexural rigidity of the segment BC is assumed to be infinite. Determine the magnitude of slope at B due to a force P applied at C.

Answer: (D)

11. Determine the correctness or otherwise of the following Assertion [a] and Reason [r].

Assertion [a]: Efficient columns are designed so that most of the column’s cross-sectional area is located as far away as possible from the principal centroidal axes of the section.

Reason [r]: Load carrying capacity of columns will increase as the moment of inertia of the cross-section increases.

(A)  Both [a] and [r] are true and [r] is the correct reason for [a].

(B)  Both [a] and [r] are but [r] is not the correct reason for [a].

(C)  Both [a] and [r] are false.

(D)  [a] is true but [r] is false.

Answer: (A)

12. Consider the structure consisting of two massless elastic bars AB and BC, each of length L, cross-sectional area A, and Young’s modulus E. Connections at A, B, C are all pinned. A horizontal force P acts on the joint B as shown. Calculate the horizontal deflection of the joint B.

Answer: (B)

13. A rigid bar ABC of mass m and length L is hinged at A and has a point mass M attached at C. An elastic spring with linear stiffness k is attached at B as shown. Ignore the effect of gravity and damping. The natural frequency of small oscillations of this system is _______.

Answer: (D)

14. A beam of flexural rigidity EI is fixed at A and supported by a linear spring of stiffness k = EI/L3 at B. Determine the compressive force developed in the spring, when the beam is subjected to a uniformly distributed load of w per unit length.

Answer: (A)

15. The bar AB is fixed at A and is separated by a gap of 0.005 mm from wall at C as shown. The temperature of the bar is increased by 10° If the Young’s modulus of the bar is E = 200 GPa and the coefficient of thermal expansion is α = 10 × 106/°C, then the magnitude of the compressive stress (in MPa) developed in the bar is _________.

Answer: (9.99 to 10.00)

16. A thin walled spherical pressure vessel has mean radius 1000 mm and wall thickness 10 mm. The material has Young’s modulus 200 GPa and Poisson’s ratio 0.25. If the internal pressure is 100 MPa, the radial displacement (in mm) of the spherical pressure vessel ( rounded off to two decimal places) is _____.

Answer: (18.60 to 18.90)

17. A particles of mass m = 100 kg is released from rest and falls under gravity through a height of H = 1 mm directly onto an upright massless elastic bar of length L = 200 mm, Young’s modulus 200 GPa, and cross-sectional area 100 mm2. Assume the following during impact : (a) particle mass sticks to the bar, (b) the bar does not buckle, and (c) no energy is lost. Use gravitational acceleration g = 10 m/s2. The maximum axial compression (in mm) of the bar due to the impact (rounded off to three decimal places) is _______.

Answer: (4.477 to 4.487)

18. A pin-jointed truss has a pin support at A and a roller support at C. All the members are made of same material and have the same cross-section. Neglect the self-weight of the members. Due to the applied loading shown, the total number of zero force members is _______.

Answer: (7 to 7)

19. Two beams AB and BC having diameter of 100 mm are connected by an internal hinge at B. The structure is fixed at A and roller supported at C. Load of P = 1 kN is applied at B. Ignoring the effect of any transverse shear stress, the tensile stress (in MPa) developed at A due to bending (rounded off to three decimal places) is _______.

Answer: (10.160 to 10.340)

20. The shear force diagram for a beam AD, which is simply supported at A and D, is shown. The magnitude of the maximum bending moment (in kN.m) is ______ (rounded off to three decimal places).

Answer: (5.600 to 5.650)

21. A rectangular thin plate with Young’s modulus 200 GPa and Poisson’s ratio 0.30 is subjected to uniform stress distribution at its edges as shown. However, it is stated the dimension b of the plate does not change under the action of the stress components σxx and σyy. Considering micro-strains (in 106), the change in the length of dimension a (in mm) is ______ (rounded off to three decimal places).

Answer: (0.175 to 0.190)

22. A solid transmission shaft has length 10 m and diameter 100 mm. The shaft is supported by frictionless bearings at ends that act as simple supports. In addition to its self-weight acting as a uniformly distributed load per unit length, an operational torque of 5πm is applied. The density and yield strength of the material are 8000 kg/m3 and  350 MPa, respectively. Use gravitational acceleration as 10 m/s2 and ignore the effect of transverse shear stress. The factor of safety of the shaft as per maximum shear stress failure theory (Tresca criterion) is _________(rounded off to two decimal places).

Answer: (1.90 to 2.05)

XE: Engineering Sciences – E: Thermodynamics

Q1 – Q9 carry one mark each.

1. If x and y are two independent intensive properties of a thermodynamic system, then which relation among the followings fails to identify z as another thermodynamics property?

(A)  dz = x dy + y dx

(B)  dz = x dy – y dx

(C)  dz = 2 dy + dx

(D)   

Answer: (B)

2. Internal energy of a thermodynamic system is defined by the

(A)  zeroth law of thermodynamics

(B)  first law of thermodynamics

(C)  second law of thermodynamics

(D)  third law of thermodynamics

Answer: (B)

3. In a polytropic process described by PVn = constant, if n = 0, the process is called as

(A)  isobaric

(B)  isochoric

(C)  isothermal

(D)  isentropic

Answer: (A)

4. The relation between the coefficient of performance of a refrigerator (COP)R and the coefficient of performance of a heat pump (COP)HP is

(A)  (COP)HP = (COP)R + 1

(B)  (COP)HP = (COP)R − 1

(C)  (COP)HP = 1 − (COP)R

(D)  (COP)HP × (COP)R = 1

Answer: (A)

5. If L1, L2 and L3 are the latent heats of vaporization at the critical temperature of nitrogen, water and ammonia, respectively, then which one of the following is true?

(A)  L1 > L2 > L3

(B)  L1 > L2 and L­2 = L3

(C)  L1 < L2 < L3

(D)  L1 = L2 = L3

Answer: (D)

6. A new temperature scale (°N) has been proposed where the normal freezing and normal boiling points of water are marked as 500 °N and 100 °N, respectively. If the temperature of a system is measured to be 0 °N, its temperature according to the Celsius scale (in °C) is ________.

Answer: (125 to 125)

7. Let Z1 represents the compressibility factor of air at 2 bar and 600 K, and Z2 represents the compressibility factor of air at 1 bar and 300 K. If air is assumed to be an ideal gas having gas constant of 0.287 kJ/kg.K, then Z1/Z2 is _______.

Answer: (1 to 1)

8. The rate of heat received by a heat engine from a source at 900 K is600 kJ/s. The engine rejects heat to the sink of 300 K. The heat engine produces a power of 200 kW. The irreversibility rate (in kW) of the process is _______.

Answer: (198 to 202)

9. A engine working on the air standard Diesel cycle has compression ratio of 18. The cycle has a cut-off of 1.7. If the ratio of specific heats of air is 1.4, then the thermal efficiency (in %) of the cycle (rounded off to 1 decimal place) is ________.

Answer: (63.5 to 65.5)

Q10 – Q22 carry two marks each.

10. A system with rigid wall is initially at a temperature of T1. It is used as the heat source for a heat engine, which rejects heat to a reservoir maintained at T0 (T0 < T1). The specific heats of the system are constant. If the temperature of the system finally reduces to T0, then the maximum work recoverable from the heat engine per unit mass of the system is

Answer: (A)

11. A reversible heat engine is operating between two reservoirs maintained at T1 and T2, where T1 > T2. Which one of the following is the most effective option for increasing its thermal efficiency ?

(A)  increasing T1, while keeping T2 constant

(B)  decreasing T1, while keeping T­2 constant

(C)  increasing T2, while keeping T1 constant

(D)  decreasing T2, while keeping T1 constant

Answer: (D)

12. A 4-m3 reservoir contains 10 kg of a real gas at 200 K. If this gas follows the van der Waal’s equation of state with a = 0.0687 m6.kPa/kg2, b = 0.00657 m3/kg and R = 0.187 kJ/kg.K, then the reservoir pressure (in kPa) is

(A)  93.5

(B)  94.6

(C)  95.7

(D)  101.3

Answer: (B)

13. Air at a pressure of 86 kPa and specific volume of 1 m3/kg is heated at constant pressure till it reaches 627 ° Air is assumed to be an ideal gas with constant specific heats. It has the gas constant of 0.287 kJ/kg.K and ratio of specific heats of 1.4. The change in specific entropy of air (in kJ/kg.K) during this process will be

(A)  1.104

(B)  0.740

(C)  0.788

(D)  0.529

Answer: (A)

14. An air standard Otto cycle has compression ratio of 4. The compression ratio of this cycle is changed to 6. If the ratio of specific heats is 1.4, the percentage increase in its thermal efficiency will be

(A)  20.2

(B)  27.2

(C)  42.6

(D)  51.2

Answer: (A)

15. In a mixture of gas there are 0.1 kmol of oxygen (O2), 0.1 kmol of nitrogen (N­2) and 0.8 kmol of methane (CH4). If the molar mass of O2, N2 and CH4 are 32 kg/mol, 28 kg/mol and 16 kg/mol, respectively, then the mass fraction of N2 in the gas mixture is

(A)  0.100

(B)  0.170

(C)  0.148

(D)  0.680

Answer: (C)

16. A particular gas sample is initially maintained at 6000 cm3 and 100 kPa. It is compressed during a quasistatic process following the relation PV2 = constant. The compression continues till the volume becomes 2000 cm3. The magnitude of the corresponding work transfer (in kJ) (rounded off to 20 decimal places) is

Answer: (1.15 to 1.25)

17. Carbon dioxide (CO2) enters an adiabatic rigid nozzle steadily at 1 MPa and 500 °C with a mass flow rate of 1.5 kg/s. The inlet area of the nozzle is 40 cm2 and the exit velocity is 10 times of that at the inlet. If CO2 can be considered as an ideal gas with gas constant of 0.19 kJ/kg.K and the ratio of specific heats of 1.29, the exit temperature (in K) (rounded off to 1 decimal place) is ________.

Answer: (590 to 600)

18. A closed system containing 8 kg of gas undergoes an expansion process following the relation PV2 = constant. The initial and final pressures are 1 MPa and 5 kPa, respectively, while the initial volume is 1 m3. If the specific internal energy of the gas decreases by 40 kJ/kg during the process, the heat transfer (in kJ) associated with the process (rounded off to 1 decimal place) is _____.

Answer: (2610 to 2614)

19. Saturation pressure of water at 5 °C is 0.8725 kPa. If the latent heat of vaporization is 2489.1 kJ/kg and gas constant is 0.4615 kJ/kg.K, then the saturation pressure at 10 °C (in kPa) (rounded off to 2 decimal places) is ________.

Answer: (1.20 to 1.26)

20. The turbine inlet conditions of a Rankine cycle are 10 MPa and 500 °C, while the condenser pressure is 10 kPa. The enthalpy and entropy of saturated liquid at 10 kPa are 191.8 kJ/kg and 0.6492 kJ/kg.K, respectively, while the enthalpy and entropy of vapourization at 10 kPa are 2392.1 kJ/kg and 7.4996 kJ/kg.K, respectively. The enthalpy and entropy at the inlet to the turbine are 3375.1 kJ/kg and 6.5995 kJ/kg.K, respectively. The condenser outlet has saturated liquid. Neglecting the pump work, the thermal efficiency (in %) of the cycle (rounded off to 1 decimal place) is _______.

Answer: (39 to 42)

21. The minimum and maximum temperatures of an air standard Brayton cycle are 300 K and 1100 K, respectively. The pressure ratio of this cycle is 6. The ratio of specific heats is 1.4 and the specific heats are constant. For this cycle, the ratio of network output to the turbine work (rounded off to 2 decimal places) is _______.

Answer: (0.52 to 0.57)

22. The specific humidity of air at 100 kPa is 0.015 kg of vapour per kg of dry air. The partial pressure of vapour (in kPa) in the existing state (rounded off to 2 decimal places) is ________.

Answer: (2.30 to 2.40)

XE: Engineering Sciences – F: Polymer Science and Engineering

Q1 – Q9 carry one mark each.

1. The solvent in which chain transfer is maximum in a radical polymerization is

(A)  Benzene

(B)  Chloroform

(C)  Carbon tetrachloride

(D)  Toluene

Answer: (C)

2. The monomer that can NOT be polymerized by anionic polymerization is

(A)  Styrene

(B)  Ethyl vinyl ether

(C)  Butadiene

(D)  Methyl methacrylate

Answer: (B)

3. The elastomer retaining flexibility at the lowest temperature is

(A)  Styrene butadiene rubber

(B)  Nitrile rubber

(C)  Silicone rubber

(D)  Butyl rubber

Answer: (C)

4. The polymer with minimum number of branches is

(A)  LDPE

(B)  LLDPE

(C)  HDPE

(D)  VLDPE

Answer: (C)

5. The nearest value of conductivity of Nylon 6 is

(A)  106 S/m

(B)  100 S/m

(C)  1013 S/m

(D)  1021 S/m

Answer: (C)

6. Aramid is a

(A)  Polyamide

(B)  Polyether

(C)  Polyester

(D)  Polyimide

Answer: (A)

7. A miscible blend in 1 : 1 (by weight) composition is formed with

(A)  Polystyrene and polybutadiene

(B)  Polystyrene and poly(phenylene oxide)

(C)  Polystyrene and poly(methyl methacrylate)

(D)  Polystyrene and poly(dimethyl siloxane)

Answer: (B)

8. Dicumyl peroxide is

(A)  Plasticizer

(B)  Cross-linking agent

(C)  Mold release agent

(D)  Peptizer

Answer: (B)

9. The change in stress of a polymer as a function of time at a fixed strain is known as

(A)  Fatigue

(B)  Creep

(C)  Stress relaxation

(D)  Fracture toughness

Answer: (C)

Q10 – Q22 carry two marks each.

10. Match the polymers in Column A with their corresponding polymerization methods of Column B

(A)  P-3, Q-2, R-4, S-1

(B)  P-4, Q-3, R-2, S-1

(C)  P-2, Q-3, R-4, S-1

(D)  P-1, Q-3, R-4, S-2

Answer: (C)

11. Match the appropriate processing technique in Column A to fabricate the product in Column B

(A)  P-2, Q-3, R-4, S-1

(B)  P-3, Q-4, R-2, S-1

(C)  P-4, Q-2, R-1, S-3

(D)  P-4, Q-3, R-1, S-2

Answer: (D)

12. Match the appropriate characterization technique in Column A used to determine the polymer attributes in Column B

(A)  P-2, Q-3, R-4, S-1

(B)  P-3, Q-4, R-1, S-2

(C)  P-2, Q-4, R-1, S-3

(D)  P-4, Q-1, R-3, S-2

Answer: (D)

13. Match each additive in Column A with its function given in Column B

(A)  P-2, Q-3, R-1, S-2

(B)  P-3, Q-4, R-1, S-2

(C)  P-4, Q-3, R-2, S-1

(D)  P-1, Q-3, R-4, S-2

Answer: (B)

14. Plot of shear stress against rate for various types of fluids is given below. The appropriate assignment for P, Q, R and S is

(A)  P-Dilatant, Q-Bingham plastic, R-Pseudoplastic, S-Newtonian

(B)  P-Bingham plastic, Q-Pseudoplastic, R-Dilatant, S-Newtonian

(C)  P-Pseudoplastic, Q-Bingham  plastic, R-Newtonian, S-Dilatant

(D)  P-Newtonian, Q-Diltant, R-Pseudoplastic, S-Bingham plastic

Answer: (B)

15. The number average molecular weight of a polyester formed from equimolar mixture of adipic acid and ethylene glycol at a conversion of 99.5% will be _____ (round off to nearest integer).

Answer: (17150 to 17250)

16. For a freely jointed linear polyethylene chain with molar mass of 1.4 × 105 g mol1, the value of root mean square end-to-end distance in nanometer is _________(round off to 1 decimal place). [Given: C-C bond length = 0.154 nanometer]

Answer: (15.3 to 15.5)

17. Viscosity measurements were performed for a set of PMMA solutions of different concentrations in toluene at 25° The plot of reduced viscosity against concentration (c) of the PMMA solutions produced an intercept of 21.0 cm3 g1 on the ordinate at c = 0. The value of viscosity average molecular weight of PMMA in toluene at 25 °C is ________ (round off to nearest integer). [Given: Mark-Houwink constants K = 7.5 × 103 cm3 g1 and a = 0.72 for PMMA is toluene at 25°C]

Answer: (60000 to 62000)

18. Glass fibre reinforced PP composite is to be prepared with 20 volume % of glass fibre. The densities of glass fibre and PP are 2540 kg m3 and 900 kg m3, respectively. The mass of glass fibre required to produce 1 kg of the composite in kg is ______ (round off to 2 decimal places).

Answer: (0.40 to 0.42)

19. A polymer solution flows through a cylindrical tube with a diameter of 4 mm at a volumetric flow rate of 109 m3 s1. Under laminar flow condition and assuming the polymer solution to be a Newtonian fluid with viscosity 102 N s m2, the value of pressure drop per unit length of the tube in N m3 is _______ (round off the nearest integer). [Consider the value of π as 3.14]

Answer: (15800 to 16000)

20. A molten polymer with a bulk modulus of 1 GPa is pressurized to 200 MPa during injection molding.. The fractional decrease in volume of the molten polymer at this pressurized condition is ________ (round off to 1 decimal place).

Answer: (0.2 to 0.2)

21. Assume that each cross-link produced by vulcanization of polyisoprene contains an average of two sulphur atoms and that the sulphur is present only in the cross-links. If 40% of the isoprene units are cross-linked, the sulphur content in weight percentage is _______ (round off to 2 decimal places)/.

Answer: (15.75 to 15.95)

22. A tensile force of 160 N is applied to a piece of vulcanized rubber of dimension 30 mm × 4 mm × 4 mm. Assuming the vulcanized rubber to be incompressible, if the sample is elongated to 150% of its original length under the same applied force, the true stress in N mm−2 will be ________ (round off to 1 decimal place).

Answer: (14.9 to 15.1)

XE: Engineering Sciences – G: Food Technology

Q1 – Q9 carry one mark each.

1. The enzyme majorly involved in postmortem degradation of muscle proteins is

(A)  Trypsin

(B)  Calpin

(C)  Transglutaminase

(D)  Pepsin

Answer: (B)

2. Which of the following is the correct pair of essential fatty acids?

(A)  Oleic acid and Lenoleic acid

(B)  Lenoleic acid and Linolenic acid

(C)  Linolenic acid and Lauric acid

(D)  Linolenic aci and Oleic acid

Answer: (B)

3. Nisin A is produced by

(A)  Aspergillus niger

(B)  Acteobacter acetis

(C)  Lactobacillus lactis

(D)  Clostridium perfringers

Answer: (C)

4. Which of the following bacteria will stain purple color after Gram staining?

(A)  Bacillus subtilis

(B)  Escherichia coli

(C)  Pseudomonas aeruginaosa

(D)  Yersinia pestis

Answer: (A)

5. The enzyme system used for removal of glucose form egg white prior to its drying consists of

(A)  Glucose oxidase and Catalase

(B)  Glucosidase and Glucoisomerase

(C)  Glucoisomerase and Catalase

(D)  Glucoamylase and Glucose oxidase

Answer: (A)

6. The INCORRECT pair of food borne illness and its causative microorganism is

(A)  Brucellosis-Brucella Sp.

(B)  Peptic ulcers-Bacillus subtilis

(C)  Bubonic plague-Yersinia pestis

(D)  Q fever-Coxiella burnatii

Answer: (B)

7. Which of the following is commonly used as a preservative in the tomato sauce?

(A)  Sodium sulphite

(B)  Potassium sorbate

(C)  Potassium sulphite

(D)  Sodium benzoate

Answer: (D)

8. The velocity of 2.2 μm diameter fat particles inside a centrifuge, running at 6000 rpm and 20 °C, is 0.25 mm s1. The velocity of 1.5 μm diameter fat particles inside the same centrifuge running at 7500 rpm and same temperature (round of to 2 decimal places) will be ______ mm s1.

Answer: (0.15 to 0.21)

9. The initial population of a bacterial strain increases from 1 × 104 cells per mL to 1 × 106 cells per mL in 120 minutes. The generation time for this strain (round off to 2 decimal places) is _______ minutes.

Answer: (17.00 to 19.00)

Q10 – Q22 carry two marks each.

10. Match the protein in Column I with its food source in Column II.

(A)  P-4, Q-1, R-2, S-3

(B)  P-4, Q-3, R-1, S-2

(C)  P-2, Q-3, R-1, S-4

(D)  P-2, Q-4, R-1, S-3

Answer: (D)

11. Match the carbohydrate in Column I with corresponding enzyme used for its hydrolysis in Column II.

(A)  P-3, Q-2, R-1, S-4

(B)  P-2, Q-4, R-1, S-3

(C)  P-1, Q-2, R-3, S-4

(D)  P-4, Q-3, R-1, S-2

Answer: (A)

12. Match the edible oil refining stage in Column I with its purpose in Column II.

(A)  P-3, Q-1, R-2, S-4

(B)  P-1, Q-4, R-2, S-3

(C)  P-4, Q-3, R-1, S-2

(D)  P-3, Q-4, R-2, S-1

Answer: (D)

13. Match the food material in Column I with its related term in Column II.

(A)  P-4, Q-2, R-1, S-3

(B)  P-3, Q-4, R-1, S-2

(C)  P-3, Q-4, R-2, S-1

(D)  P-1, Q-3, R-4, S-2

Answer: (B)

14. Match the component/system in Column I with the peeling method for fruits and vegetables in Column II.

(A)  P-4, Q-3, R-2, S-1

(B)  P-3, Q-4, R-1, S-2

(C)  P-4, Q-3, R-1, S-2

(D)  P-3, Q-4, R-2, S-1

Answer: (C)

15. Which among the given options correctly explains the nature of the microbial culture represented by curves 1, 2 and 3 in the following figure?

Answer: (B)

16. Match the equation/law in Column I with its application in Column II.

(A)  P-1, Q-3, R-4, S-2

(B)  P-2, Q-3, R-1, S-4

(C)  P-2, Q-3, R-4, S-1

(D)  P-4, Q-3, R-1, S-2

Answer: (C)

17. Match the absorber used in modified atmosphere packaging and storage in Column I with the scavenger in Column II.

(A)  P-3, Q-2, R-4, S-1

(B)  P-1, Q-2, R-4, S-3

(C)  P-2, Q-3, R-4, S-1

(D)  P-3, Q-2, R-1, S-4

Answer: (A)

18. During extrusion cooking, food materials are generally subjected to a combination of

(A)  high shear and low pressure

(B)  high temperature and high shear

(C)  low shear and high temperature

(D)  low shear and low pressure

Answer: (B)

19. The whole milk at 22°C is pumped through a stainless steel pipe at a flow rate of 3 L s1. The length and inner diameter of the pipe are 40 m and 4 cm, respectively. If viscosity and density of the milk at the pumping temperature of 0.2 Pa s and 1032 kg m3, respectively, the Revnolds number (rounded off to nearest integer) will be ______.

Answer: (491.00 to 495.00)

20. A hammer mill, operating at a feed rate of 108 ton h1, consumes 10 kW power for reducing size of wheat gain from 3.92 mm to 1.25 mm. If Bond’s law holds good, the feed rate (round off to 2 decimal places) for reducing the size of the wheat grain to 0.75 mm at the same power consumption level is _______ ton h1.

Answer: (63.00 to 67.00)

21. During spray drying of a milk sample, inlet and outlet temperatures are maintained at 132°C and 80°C, respectively. If the ambient temperature is 29°C, the thermal efficiency (round of to 2 decimal places) of the dryer will be _______ %.

Answer: (49.00 to 52.00)

22. An orange juice flowing at 0.80 kg s1 enters a counter current double pipe heat exchanger at 20°C and leaves at 72° Inlet and outlet temperatures of the hot water used as heating medium in the exchanger are 81°C and 74°C, respectively. The specific heat of the orange juice is 3.74 kJ kg1 K1 and overall heat transfer coefficient is 492 W m2 K1. The heat transfer surface are (round off to 2 decimal places) will be ______m3.

Answer: (11.00 to 14.00)

XE: Engineering Sciences – H: Atmospheric and Oceanic Sciences

Q1 – Q9 carry one mark each.

1. In the northern hemisphere, the flow in the middle depths of the ocean is geostrophic. As we go down from that level and start approaching the bottom of the ocean, the flow deflects to the left of the geostrophic current because

(A)  friction decreases and Coriolis force increases

(B)  friction decreases and Coriolis force decreases

(C)  friction increases and Coriolis force increases

(D)  friction increases and Coriolis force decreases

Answer: (D)

2. Which one of the following is the definition of a monsoon?

(A)  Seasonal reversal of wind direction

(B)  High rainfall

(C)  Occurs in the summer

(D)  Occurs in the tropics

Answer: (A)

3. Anthropogenic emission of ______ is the main contributor to the ongoing ocean acidification.

(A)  Methane

(B)  Carbon dioxide

(C)  Nitrous oxide

(D)  Sulpuric acid

Answer: (B)

4. What are phytoplankton?

(A)  Microscopic animal life floating on surface of water bodies

(B)  Pollen floating freely on surface of water bodies

(C)  Microscopic plant life floating on surface of water bodies

(D)  Microscopic plant life living on the floor of water bodies

Answer: (C)

5. Consider the two atmospheric virtual temperature profiles observed in Delhi given in Figures (i) and (ii) blow.

At what times of the day are you most likely to see such profiles?

(A)  (i) Midnight and (ii) noon

(B)  (i) 3 pm and (ii) 3 am

(C)  (i) Sunrise and (ii) sunset

(D)  (i) 3 am and (ii) 3 pm

Answer: (B)

6. A south-easterly wind is blowing towards which direction?

(A)  135°

(B)  157.5°

(C)  315°

(D)  225°

Answer: (C)

7. Consider a dry parcel at 30°C in an isothermal environment at 25° The parcel rises adiabatically by 1 km. Assuming g = 10 ms2 and air density = 1 kgm3, the buoyancy force at the new location (rounded off to 2 decimal places) is _______ ms2.

Answer: (-0.17 to -0.16)

8. The emissivity of polluted air that reflects and transmits 20% and 60% of the incoming solar radiation, respectively, at a given wavelength (correct up to 1 decimal place) is ______.

Answer: (0.2  to 0.2)

9. Given that the angular velocity of rotation of the Earth = 7.3 × 105 s1, the period of inertial oscillations generated in the oceans by surface winds at 30° N latitude (rounded off to the nearest integer) is ______ hours.

Answer: (24 to 24)

Q10 – Q22 carry two marks each.

10. Consider a high pressure centre in the northern hemisphere with tangential winds of 10 ms1 at a distance of 500 km from the centre. Assuming solid body rotation principles, what is the relative vorticity of the flow?

(A)  2 × 105 s1

(B)  −2 × 105 s1

(C)  4 × 105 s1

(D)  −4 × 105 s1

Answer: (D)

11. Which one of the following statements is true for atmospheric and oceanic general circulation models?

(A)  Vertical velocity is ignored in oceanic models but not in atmospheric models.

(B)  Boussinesq approximation is adequate in oceanic models but not in atmospheric models.

(C)  Atmospheric models need a longer spin-up and integration time than oceanic models.

(D)  Atmospheric models need parameterizations for subgrid scale processes but oceanic models do not.

Answer: (B)

12. Consider a scenario where air temperature increases by 2 ° We know that saturation vapour pressure for water is increases with temperature. As a result of this effect, the water vapour content of the atmosphere will _____ and the net warming will be ______ than 2°C. The correct pair of words to fill in the blanks (in the right order) is

(A)  increase, more

(B)  increase, less

(C)  decrease, more

(D)  decrease, less

Answer: (A)

13. The prevailing Trade winds over the Equator in the Pacific Ocean result in piling up of water in the ______ part of the ocean. As a result, the gradients of the thermocline and the ocean surface have ______ signs. The correct pair of words to fill in the blanks (in the right order) is

(A)  western, opposite

(B)  western, same

(C)  eastern, opposite

(D)  eastern, same

Answer: (A)

14. Consider two different cases, shown in the Figures (i) and (ii) below, with two layers of water of same density on top of each other.

Which one of the following statements is true about convective plumes across the interface of the two layers?

(A)  Upward convective plumes in (i) and downward convective plumes in (ii)

(B)  Downward convective plumes in (i) and upward convective plumes in (ii)

(C)  No convective plume in (i) and (ii)

(D)  No convective plume in (i) but upward convective plumes in (ii)

Answer: (B)

15. During the Indian summer monsoon, surface outgoing longwave radiation (OLR) over the Arabian Sea is often observed to be low because

(A)  of enhanced convection

(B)  monsoon winds advect the OLR away into the Indian subcontinent

(C)  monsoon clouds limit incoming solar radiation

(D)  surface Bowen ratio is low

Answer: (C)

16. A tsunami wave in the ocean is approaching the coast. Assuming g = 10 ms2, the correct group speed of the wave at a depth of 1 km is

(A)  1 ms1

(B)  10 ms1

(C)  100 ms1

(D)  1000 ms1

Answer: (C)

17. A tornado is in cyclostrophic balance where the horizontal pressure gradient and centrifugal forces balance each other. Consider a tornado with 100 m radius and a tangential velocity of 100 ms1 at the edge. Assuming air density = 1 kgm3, the magnitude of the pressure-drop between the centre and the edge of the tornado is ______ kgm1 s2.

Answer: (10000 to 10000)

18. While driving south a distance of 1000 km, the temperature outside your car increases from 10 °C to 20 ° Assuming the air is completely dry, g = 10 ms2 and Coriolis parameter = 104 s1, the vertical gradient of the geostrophic wind (rounded off to 2 decimal places) is _______ ms1km1.

Answer: (3.40 to 3.50)

19. A westerly wind of 10 ms1 is blowing at a location in the Pacific Ocean in the northern hemisphere. Assuming density of sea water = 1000 kgm3, Coriolis parameter = 104 s1 and drag coefficient for sea water = 106, the Ekman transport due to the wind at that location is ______ kgm1 s1.

Answer: (-1000 to -1000)

20. Mx and My represent the ocean mass transport in the x and y directions, respectively. Lx and Ly are the corresponding east-west and north-south length scales. For a typical equatorial ocean gyre, if the ratio of zonal to meridional mass transport ≈ 10, then Lx ≈ _______ Ly.

Answer: (10 to 10)

21. Assume the pressure varies exponentially with height: p(z) = p0ez/H, where p(z) is the pressure at a height z above the surface, p0 is the surface pressure, and the scale height H= 7.5 km. Under these conditions, one-fourth of the total mass of the atmosphere lies above a height (rounded off to 1 decimal place) of km above the surface.

Answer: (10.3 to 10.5)

22. Consider an atmospheric column of depth 300 m at the Earth’s surface with an average temperature of 300 K. If the temperature of the layer rises by ∆T = 10 °C, the layer depth h will increase by ∆ Assuming ∆T/T ≈ ∆h/h, air density remains unchanged at 1 kgm3 and g = 10 ms2, the change in surface pressure is _______ kgm1s2.

Answer: (-100 to -100 or 100 to 100)

GATE Exam 2020 Textile Engineering and Fibre Science (TF) Question Paper With Answer Key

GATE-2020

TF: Textile Engineering and Fibre Science

GA- General Aptitude

Q1 – Q5 carry one mark each.

1. This book, including all its chapters, ______ interesting. The students as well as the instructor ______ in agreement about it.

(A)  is, was

(B)  are, are

(C)  is, are

(D)  were, was

Answer: (C)

2. People were prohibited ______ their vehicles near the entrance of the main administrative building.

(A)  to park

(B)  from parking

(C)  parking

(D)  to have parked

Answer: (B)

3. Select the word that fits the analogy:

Do : Undo :: Trust : ________

(A)  Entrust

(B)  Intrust

(C)  Distrust

(D)  Untrust

Answer: (C)

4. Stock markets ________ at the news of the coup.

(A)  poised

(B)  plunged

(C)  plugged

(D)  probed

Answer: (B)

5. If P, Q, R, S are four individuals, how many teams of size exceeding one can be formed, with Q as a member ?

(A)  5

(B)  6

(C)  7

(D)  8

Answer: (C)

Q6 – Q10 carry two marks each.

6. Non-performing Assets (NPAs) of a bank in India is defined as an asset, which remains unpaid by a borrower for a certain period of time in terms of interest, principal, or both. Reserve Bank of India (RBI) has changed the definition of NPA thrice during 1993-2004, in terms of the holding period of loans. The holding period was reduced by one quarter each time. In 1993, the holding period was four quarters (360 days).

Based on the above paragraph, the holding period of loans in 2004 after the third revision was ______ days.

(A)  45

(B)  90

(C)  135

(D)  180

Answer: (B)

7. Select the next element of the series: Z, WV, RQP, ______

(A)  LKJI

(B)  JIHG

(C)  KJIH

(D)  NMLK

Answer: (C)

8. In four-digit integer numbers from 1001 to 9999, the digit group “37” (in the same sequence) appears ______ times.

(A)  270

(B)  279

(C)  280

(D)  299

Answer: (C)

9. Given a semicircle with O as the centre, as shown in the figure, the ratio  is ______. where  are chords.

(A)  √2

(B)  √3

(C)  2

(D)  3

Answer: (A)

10. The revenue and expenditure of four different companies P, Q, R and S in 2015 are shown in the figure. If the revenue of company Q in 2015 was 20% more than that in 2014, and company Q had earned a profit of 10% on expenditure in 2014, then its expenditure (in million rupees) in 2014 was _______.

(A)  32.7

(B)  33.7

(C)  34.1

(D)  35.1

Answer: (C)

TF: Textile Engineering and Fibre Science

Q1 – Q25 carry one mark each.

1. For the matrix  the eigenvalues of matrix A2 are

(A)  1, 0, 1

(B)  1, 0, 0

(C)  1, 1, 0

(D)  1, 1, 1

Answer: (D)

2. The integrating factor of the differential equation  is

(A)  ex

(B)  ex

(C)  xex

(D)  xex

Answer: (A)

3. Laplace transform of cosh(t) is

Answer: (C)

4. In wool, the sulfur containing amino acid is

(A)  Alanine

(B)  Cystine

(C)  Glycine

(D)  Serine

Answer: (B)

5. Viscose rayon is soluble in

(A)  Acetone

(B)  Chloroform

(C)  Formic acid 85% (v/v)

(D)  Sulfuric acid 59% (w/w)

Answer: (D)

6. In carding, the highest draft is kept between

(A)  Lap roller and feed roller

(B)  Feed roller and licker-in

(C)  Licker-in and cylinder

(D)  Cylinder and doffer

Answer: (B)

7. The spinning system in which one revolution of twisting element imparts several turns to the fibre strand is

(A)  Ring

(B)  Rotor

(C)  Friction

(D)  Wrap

Answer: (C)

8. The technology that does NOT produce a nonwoven fabric is

(A)  Spunbonding

(B)  Hydroentangling

(C)  Meltblowing

(D)  Braiding

Answer: (D)

9. For the same, yarn and fabric sett, the weave that gives the maximum tearing strength is

(A)  Plain

(B)  2 × 2 matt

(C)  5-end satin

(D)  2/1 twill

Answer: (C)

10. Two yarns have variance of strength as V1 and V2. If V1 < V2, the variance ratio ‘F’ would be

(A)  V2/V1

(B)  V1/V2

(C)  V12/V22

(D)  V22/V12

Answer: (A)

11. Cotton fibre length parameter that CANNOT be obtained from Baer Sorter diagram is

(A)  Mean length

(B)  Dispersion

(C)  Uniformity ratio

(D)  Modal length

Answer: (C)

12. The purpose of cabonization of wool fibres is to remove

(A)  Waxy matter

(B)  Surface scales

(C)  Vegetable matter

(D)  Ortho-cortex

Answer: (C)

13. Bio-polishing of cotton fabrics is done using

(A)  Cellulase

(B)  Amylase

(C)  Proteinase

(D)  Esterase

Answer: (A)

14. For the given system of linear equations, 2x – z = 1; 5x + y = 7; y + 3z = 5, the sum of x, y and z is_______.

Answer: (4 to 4)

15. If F = xi + yj + zk, then the magnitude of ∇ × F is ______.

Answer: (0 to 0)

16. A polypropylene filament is drawn in two stages with draw ratios of 1.5 and 2 respectively. The overall draw ratio is _______.

Answer: (3 to 3)

17. The refractive indices of a filament in axial and radial directions are 1.58 and 1.52 respectively. The birefringence of the filament (correct up to 2 decimal places) is ______.

Answer: (0.06 to 0.06)

18. A twin-delivery drawframe, running at a delivery speed of 800 m/min with an efficiency of 95%, is producing 5.39 ktex sliver. The rate of production of the drawframe in kg/h (rounded off to 2 decimal places) is _____.

Answer: (537.00 to 539.00)

19. The diameter (mm) of a yarn having twist of 700 turns per meter and surface-twist angle of 20° (rounded off to 2 decimal places) is ______.

Answer: (0.16 to 0.18)

20. A magazine creel has 800 package holders. The effective creel capacity (number) is ______.

Answer: (400 to 400)

21. A shuttle loom is running at 180 picks per minute. The angular velocity of crank shaft (degree/second) is ______.

Answer: (1080 to 1080)

22. The length (km) of 5 kg of 30 Ne yarn (rounded off to the nearest integer) is _____.

Answer: (253 to 255)

23. The limit irregularity and measured irregularity of a yarn are 8.4% and 9.6%, respectively. The index of Irregularity (rounded off to 2 decimal places) is _____.

Answer: (1.10 to 1.15)

24. A padding mangle is processing a fabric at 1320 m/h. The bottom bowl of the mangle is rotating at 25 rpm. Assuming zero slippage at the nip, the diameter (cm) of this bowl is ______.

Answer: (28.00 to 29.00)

25. A Procion H (monochlorotriazine based) reactive dye used for printing of cotton has a molecular weight of 471. Taking the atomic weight of H = 1, C = 12, N = 14, O = 16, Cl = 35.5, the molecular weight of the fully hydrolyzed dye (correct up to 1 decimal place) would be _______.

Answer: (452.5 to 452.5)

Q26 – Q55 carry two marks each.

26. Let  The value o L is

(A)  0

(B)  1

(C)  2

(D)  ∞

Answer: (B)

27. The solution of the differential equation  which satisfies the conditions, y(0) = 0, y’(0) = 3 is

(A)  ex

(B)  ex

(C)  ex + e2x

(D)  ex – e2x

Answer: (D)

28. In melt spinning of poly(ethylene terephthalate), pre-drying of polymer chips is essential to avoid

(A)  Hydrolytic degradation

(B)  Oxidative degradation

(C)  Microbial degradation

(D)  Photo-induced degradation

Answer: (A)

29. Determine the correctness or otherwise of the following Assertion [a] and Reason [r]

[a]: Caprolactam is polymerized in the presence of small amount of water of produce fibre grade nylon 6.

[r]: Water reacts as a catalyst and converts caprolactam to aminocaproic acid.

(A)  Both [a] and [r] and true and [r] is the correct reason for [a]

(B)  Both [a] and [r] are true but [r] is not the correct reason for [a]

(C)  Both [a] and [r] are false

(D)  [a] is true but [r] is false

Answer: (A)

30. Determine the correctness or otherwise of the following Assertion [a] and Reason [r]

[a]: Melting point of nylon 66 fibre is much higher that of polyethylene fibre.

[r]: The molecular weight of nylon 66 fibre is significantly higher than that of polyethylene fibre.

(A)  Both [a] and [r] and true and [r] is the correct reason for [a]

(B)  Both [a] and [r] are true but [r] is not the correct reason for [a]

(C)  Both [a] and [r] are false

(D)  [a] is true but [r] is false

Answer: (D)

31. Carding of polyester fibres requires that the values of wire-point density (points/inch2) of

(P) Licker-in       (Q) Cylinder    (R) Flat

follow the order

(A)  P < Q < R

(B)  P < R < Q

(C)  Q < R < P

(D)  Q < P < R

Answer: (B)

32. For combing with forward feed, the given parameters are:

Detachment setting = 15 mm, length of feed per combing cycle = 6 mm, Longest fiber length = 30 mm. According to Gegauff’s theory, the noil (%) would be

(A)  9

(B)  16

(C)  30

(D)  49

Answer: (B)

33. Determine the correctness or otherwise of the following Assertion [a] and Reason [r]

[a]: Open-loop autolevelling system needs a signal storage device with time delay function.

[r]: The signal must be stored until the material reaches the adjusting point.

(A)  Both [a] and [r] and true and [r] is the correct reason for [a]

(B)  Both [a] and [r] are true but [r] is not the correct reason for [a]

(C)  Both [a] and [r] are false

(D)  [a] is true but [r] is false

Answer: (A)

34. If both the concentration (%, w/w) of size paste and target add-on are 12%, the total wet pick-up (kg) by 12 kg bone-dry warp sheet is

(A)  6

(B)  12

(C)  18

(D)  24

Answer: (B)

35. At front centre (0°) and a back centre (180°) of a shuttle loom.

(A)  The sley velocities are the same but accelerations are different

(B)  The sley velocities are different but accelerations are the same

(C)  The sley velocities are the same and also accelerations are the same

(D)  The sley velocities are different and also accelerations are different

Answer: (A)

36. Match the looms listed in Group with the corresponding components given in Group II. The correct option is

(A)  P-1, Q-4, R-2, S-3

(B)  P-1, Q-2, R-4, S-3

(C)  P-4, Q-1, R-2, S-3

(D)  P-3, Q-1, R-2, S-4

Answer: (C)

37. Consider two yarns, one 100% wool and the other 100% cotton, each containing 100 fibres in the yarn cross-section. The respective limit irregularities (%) of wool and cotton yarns will approximately be

(A)  11.2 and 10.6

(B)  10.6 and 11.2

(C)  11.8 and 11.2

(D)  11.8 and 10.6

Answer: (A)

38. If the numerical value X of yarn linear density, expressed in denier is the same as that expressed in English (Ne) system, then X approximately is

(A)  24.3

(B)  48.6

(C)  72.9

(D)  97.2

Answer: (C)

39. Determine the correctness or otherwise of the following Assertion [a] and Reason [r]

[a]: Sodium chlorite is a bleaching agent for cotton.

[r]: Sodium chlorite is an effective reducing agent.

(A)  Both [a] and [r] and true and [r] is the correct reason for [a]

(B)  Both [a] and [r] are true but [r] is not the correct reason for [a]

(C)  Both [a] and [r] are false

(D)  [a] is true but [r] is false

Answer: (D)

40. Determine the correctness or otherwise of the following Assertion [a] and Reason [r]

[a]: Acrylic fibres are dyed with basic dyes in acidic medium.

[r]: In acidic medium the acrylic fibre acquires positive charge.

(A)  Both [a] and [r] and true and [r] is the correct reason for [a]

(B)  Both [a] and [r] are true but [r] is not the correct reason for [a]

(C)  Both [a] and [r] are false

(D)  [a] is true but [r] is false

Answer: (D)

41. Determine the correctness or otherwise of the following Assertion [a] and Reason [r]

[a]: Foam finishing significantly reduces the energy consumed in drying.

[r]: The specific heat of air is significantly lower than that of water.

(A)  Both [a] and [r] and true and [r] is the correct reason for [a]

(B)  Both [a] and [r] are true but [r] is not the correct reason for [a]

(C)  Both [a] and [r] are false

(D)  [a] is true but [r] is false

Answer: (B)

42. If the probability density function of a continuous random variable X is given by f(x) = ex, 0 ≤ x < ∞, the mean of random variable X is _______.

Answer: (1 to 1)

43. Assuming the step size h = 1, the numerical value of the definite integral  obtained using Trapezoidal rule (rounded off to 2 decimal places) is ______.

Answer: (0.70 to 0.75)

44. In the production of PET, diglycol terephthalate (DGT) is an intermediate. Taking the atomic weights of H = 1, C = 12, O = 16, the molecular weight of DGT is______.

Answer: (254 to 254)

45. In wet spinning of acrylic filament yarn, the volumetric flow rate of the spinning dope per spinneret hole is 0.1 cm3/min. If the surface speed at the first take up roller is 1.5 m/min and the diameter of spinneret hole is 0.02 cm, then the jet stretch (rounded off to 2 decimal places) is _______.

Answer: (0.46 to 0.48)

46. Two types of polyester staple fibers of fineness 3 and 6 denier and having the same length are mixed in a ratio of 2 : 3 by weight. The mean fibre fineness (denier) of the mix (rounded off to 2 decimal places) is _____.

Answer: (4.25 to 4.35)

47. Two rovings, each with mass CV of 10%, are fed to a ring spinning machine that adds a mass CV of 20%. The mass CV (%) of the yearn (rounded off to 2 decimal places) is _____.

Answer: (21.00 to 22.00)

48. In a drum-driven winder, the grooved drum having a width of 20 cm is rotating at 1000 rpm. If the drum makes 5 revolutions per double traverse, the traverse speed (m/min) is ______.

Answer: (80 to 80)

49. The wale constant and course constant are 4.2 and 5.04 respectively. If the loop length is 4.2 mm, then stitch density (number/cm2) is _______.

Answer: (120 to 120)

50. A cotton fibre has degree of cell wall thickening (θ) of 0.9 and perimeter of 40 μ The actual cross-sectional area of the wall (μm2) of the fibre (rounded off two 1 decimal place) is _______.

Answer: (112.0 to 117.0)

51. A fabric with mass per unit area of 250 g/m2 has flexural rigidity of 275 μN-m. The bending length (mm) of the fabric (rounded off to 2 decimal places) is______.

Answer: (46.00 to 50.00)

52. The ‘standard machine rate of loading’ of a tensile tester, working on pendulum lever principle, is 440 N/cm. As the pendulum lever swings from 30° to 45°, the ‘machine rate of loading’ (N/cm), reduces by (rounded off to 2 decimal places) ________.

Answer: (69.00 to 71.00)

53. Under a load of 500 cN, the extension of a yarn of 300 mm length is 10%. If the elastic recovery is 90%, then the length (mm) of the yarn after removal of load is _______.

Answer: (303 to 303)

54. Given that one gram mole of a gas occupies 22.4 L of volume at STP, the atomic weights of H = 1, and that of O = 16, the concentration (g/L) of hydrogen peroxide solution of 25 volume strength (rounded off to 2 decimal places) is _______.

Answer: (75.00 to 77.00)

55. The work of adhesion (WSL) depends on the surface tension (γLV) of the liquid and the contact angle (θ) formed on a surface and is expressed as γLV(1 + cos θ). The WSL for a given fabric and a liquid is reduced to 1/3rd of the original value after oil repellent treatment. If the measured contact angle of untreated fabric is 60°, the percent change in the contact angle after the treatment is ______.

Answer: (100 to 100)

GATE Exam 2020 Stastics (ST) Question Paper With Answer Key

GATE-2020

ST: Statistics

GA-General Aptitude

Q1 – Q5 carry one mark each.

1. Rajiv Gandhi Khel Ratna Award was conferred _____Mary Kom, a six-time world champion in boxing, recently in a ceremony _____ the Rashtrapati Bhawan (the President’s official residence) in New Delhi.

(A)  with, at

(B)  on, in

(C)  on, at

(D)  to, at

Answer: (C)

2. Despite a string of a poor performances, the changes of K. L. Rahul’s selection in the team are ______.

(A)  slim

(B)  bright

(C)  obvious

(D)  uncertain

Answer: (B)

3. Select the word that fits the analogy:

Cover : Uncover :: Associate : _______

(A)  Unassociate

(B)  Inassociate

(C)  Missassociate

(D)  Dissociate

Answer: (D)

4. Hig by floods, he kharif (summer sown) crops in various parts of the county have been affected. Officials believe that the loss in production of the kharif crops can be recovered in the output of the rabi (winter sown) crops so that the country can achieve its food-grain production target of 291 million tons in the crop year 2019-20 (July-June). They are hopeful that good rains in July-August will help the soil retain moisture for a longer period, helping winter sown crops such as wheat and pulses during the November-February period.

Which of the following statements can be inferred from the given passage?

(A)  Officials declared that the food-grain production target will be met due to good rains.

(B)  Officials want the food-grain production target to be met by the November-February period.

(C)  Officials feel that the food-grain production target cannot be met due to floods.

(D)  Officials hope that the food-grain production target will be met due to a good rabi produce.

Answer: (D)

5. The difference between the sum of the first 2n natural numbers and the sum of the first n odd natural numbers is ______.

(A)  n2 – n

(B)  n2 + n

(C)  2n2 – n

(D)  2n2 + n

Answer: (B)

Q6 – Q10 carry two marks each.

6. Repo rate is the at which Reserve Bank of India (RBI) lends commercial banks, and reverse repo rate is the rate at which RBI borrows money from commercial banks.

Which of the following statements can be inferred from the above passage?

(A)  Decrease in repo rate will increase cost of borrowing and decrease lending by commercial banks.

(B)  Increase in repo rate will decrease cost of borrowing and increase lending by commercial banks.

(C)  Increase in repo rate will decrease cost of borrowing and decrease lending by commercial banks.

(D)  Decrease in repo rate will decrease cost of borrowing and increase lending by commercial banks.

Answer: (D)

7. P, Q, R, S, T, U, V, and W are seated around a circular table.

(I) S is seated opposite to W.

(II) U is seated at the second place to the right of R.

(III) T is seated at the third place to the left of R.

(IV) V is a neighbor of S.

Which of the following must be true?

(A)  P is a neighbor of R.

(B)  Q is a neighbor of R.

(C)  P is not seated opposite to Q.

(D)  R is the left neighbor of S.

Answer: (C)

8. The distance between Delhi and Agra is 233 km. A car P started travelling from Delhi to Agra and another car Q started from Agra to Delhi along the same road 1 hour after the car P started. The two cars crossed each other 75 minutes after the car Q started. Both cars were travelling at constant speed. The speed of car P was 10 km/hr more than the speed of car Q. How many kilometers the car Q had travelled when the cars crossed each other?

(A)  66.6

(B)  75.2

(C)  88.2

(D)  116.5

Answer: (B)

9. For a matrix M = [mij], i. j= 1, 2, 3, 4, the diagonal elements are all zero and mij = −mij. The minimum number of elements required to fully specify the matrix is_______.

(A)  0

(B)  6

(C)  12

(D)  16

Answer: (B)

10. The profit shares of two companies P and Q are shown in the figure. If the two companies have invested a fixed and equal amount every year, then the ratio of the total revenue of company P to the total revenue of company Q, during 2013-2018 is ______.

(A)  15 : 17

(B)  16 : 17

(C)  17 : 15

(D)  17 : 16

Answer: (B)

ST: Statistics

Q1 – Q25 carry one mark each.

1. Let M be a 3 × 3 non-zero idempotent matrix and let I3 denote the 3 × 3 identity matrix. Then which of the following statements is FLASE?

(A)  The eigenvalues of M are 0 and 1

(B)  Rank(M) = Trace(M)

(C)  I3 – M is idempotent

(D)  (I3 + M)1 = I3 – 2M

Answer: (D)

2. Let ℂ denote the set of all complex numbers. Consider the vector space

over the field of real numbers, where for any complex number z,  denotes its complex conjugate. If i = √−1, then a basis of V is

(A)  {(1, −1, 1), (i, i, i)}

(B)  {(1, −1, 1), (i, −i, i)}

(C)  {(1, −i, 1), (i, 1, i)}

(D)  {(1, −i, 1), (i, 1, −i)

Answer: (A)

3. Let S = {(x, y) ∈ ℝ × ℝ: x2 – y2 = 4} and f : S → ℝ be defined by f(x, y) = 6x + y2, where ℝ denotes the set of all real numbers. Then

(A)  f is bounded of S

(B)  the maximum value of f on S is 13

(C)  the minimum value of f on S is −14

(D)  the minimum value of f on S is −13

Answer: (D)

4. Let f : ℝ × ℝ → ℝ be defined by

        

where ℝ denotes the set of all real numbers and c ∈ ℝ is fixed constant. Then, which of the following statements is TRUE?

(A)  There does NOT exists a value of c for which f is continuous at (0, 0)

(B)  f is continuous at (0, 0) if c = 0

(C)  f is continuous at (0, 0) if c = 10

(D)  f is continuous at (0 0) if c = 16

Answer: (D)

5. The moment generating function of a random variable X is given by

Then P(X ≤ 2) equals

(A)  1/3

(B)  1/6

(C)  1/2

(D)  5/6

Answer: (D)

6. Consider the following two-way fixed effects analysis of variance model

Yijk = μ + αi + βj + ϵijk, i = 1, 2; j = 1, 2, 3; k = 1, 2, 3;

where ϵijk’s are independently and identically distributed N(0, σ2) random variables, σ ∈ (0, ∞), α1 + α2 = 0 and β1  + β2 + β3 = 0. Let SSE denote the sum of squares due to error. For any positive integer v and any α ∈ (0, 1), let χ2v,α denote the (1 – α)-th quantile of the central chi-square distribution with v degrees of freedom. Then a 95% confidence interval for σ2 is given by

Answer: (D)

7. Let X1, …, X20 be independent and identically distributed random variables with the common probability density function 

Then the distribution of the random variableis

(A)  central chi-square with 10 degrees of freedom

(B)  central chi-square with 20 degrees of freedom

(C)  central chi-square with 30 degrees of freedom

(D)  central chi-square with 40 degrees of freedom

Answer: (D)

8. Let X1, …, X10 be a random sample from a Weibull distribution with the probability density function 

where θ ∈ (0, ∞). For any positive integer v and any α ∈ (0, 1), let χ2v,α denote the (1 – α)-th quantile of the central chi-square distribution with v degrees of freedom. Then, a 90% confidence interval for θ is

Answer: (A)

9. Let X1, …, Xn be a random sample of size n (≥2) from a uniform distribution on the interval [−θ, θ], where θ ∈ (0, ∞). A minimal sufficient for θ is

Answer: (C)

10. Let X1 …, Xn be a random sample of size n(≥2) from N(θ, 2θ2) distribution, where θ ∈ (0, ∞). Which of the following statements is TRUE?

(A)  is the unique unbiased estimator of θ2 that is a function of minimal sufficient statistic

(B)  is an unbiased estimator of θ2

(C)  There exist infinite number of unbiased estimators of θ2 which are functions of minimal sufficient statistic

(D)  There does NOT exist any unbiased estimator of θ(θ + 1) that is a function of minimal sufficient statistic

Answer: (C)

11. Let {N(t), t ≥ 0} be a Poisson process with rate λ = 2. Given that N(3) = 1, the expected arrival time of the first event of the process is

(A)  1

(B)  3/2

(C)  2/3

(D)  3

Answer: (B)

12. Consider the regression model

Yi = β0 + β1xi2 + ϵi, i = 1, 2, …, n(n ≥ 2);

where β0 and β1 are unknown parameters and ϵis are random errors. Let y­i be the observed value of Yi, i = 1, …, n. Using the method of ordinary least squares, the estimate of β1 is

Answer: (B)

13. Let  be a random sample of size n (≥2) from  distribution, where 1 ≤ p ≤ n – 1 and ∑ is a positive definite matrix. Define

where for any column vector  denotes its transpose. Then the distribution of the statistic  is

(A)  χ2p, the central chi-square distribution with p degrees of freedom

(B)  Fp,np, the central F distribution with p and n – p degrees of freedom

(C)   where Fp,np, is the central F distribution with p and n – p degrees of freedom

(D)   where Fp,np, is the central F distribution with n – p and p degrees of freedom

Answer: (C)

14. Consider a two-way fixed effects analysis of variance model without interaction effect and one observation per cell. If there are 5 factors and 4 columns, then the degrees of freedom for the error sum of squares is

(A)  20

(B)  19

(C)  12

(D)  11

Answer: (C)

15. Let X1, …, Xn be a random sample of size n (≥ 2) from an exponential distribution with the probability density function

where θ ∈ {1, 2}. Consider the problem of testing H0­ : θ = 1 against H1 : θ = 2, based on X1, …, Xn. Which of the following statements is TRUE?

(A)  Likelihood ratio test at level α (0 < α < 1) leads to the same critical region as the corresponding most powerful test at the same level.

(B)  Critical region of level α (0 < α < 1) likelihood ratio test is  is the α-th quantile of the central chi-square distribution with 2n degrees of freedom

(C)  Likelihood ratio test for testing H0 against H1 does not exist

(D)  At any fixed level α (0 < α < 1), the power of the likelihood ratio test is lower than that of the most powerful test

Answer: (A)

16. Te characteristic function of a random variable X is given by

Then P(|X| ≤ 3/2) = _____ (correct up to two decimal places).

Answer: (0.75 to 0.75)

17. Let the random  follow  distribution, where

Then P(X1 + X2 + X3 + X4 > 0) = _______ (correct up to one decimal  place).

Answer: (0.5 to 0.5)

18. Let {Xn}n0 be a homogeneous Markov chain with state space {0, 1} and one-step transition probability matrix   If P(X0 = 0) = 1/3, then 27 × E(X2) = ______ (correct  up to two decimal places).

Answer: (16.25 to 16.25)

19. Let E, F and G be mutually independent events with P(E) = 1/2, P(F) = 1/3 and P(G) = 1/4. Let p be the probability that at least two of the events among E, F and G occur. Then 12 × p = _______ (correct up to one decimal place).

Answer: (3.5 to 3.5)

20. Let the joint probability mass function of (X, Y, Z) be

where k = 10 – x – y – z; x, y, z = 0, 1, … , 10; x + y + z ≤ 10. Then the variance of the random Y + Z equals ______ (correct up to one decimal place).

Answer: (2.1 to 2.1)

21. The total number of standard 4 × 4 Latin squares is _______

Answer: (4 to 4)

22. Let  be a 4 × 1 random vector with  and variance-covariance matrix

Let  be the 4 × 1 random vector of principal components derived from ∑. The proportion of total variation explained by the first two principal components equals ______ (correct up to two decimal places).

Answer: (0.75 to 0.75 OR 75 to 75)

23. Let X1, …, Xn be a random sample of size n (≥ 2) from an exponential distribution with the probability density function

where θ ∈ (0, ∞). If X(1) = min{X1, …, Xn} then the conditional expectation

Answer: (2 to 2)

24. Let Yi = α + βxi + ϵi, i = 1, 2, …, 7, where xi’s are fixed covariates and ϵi’s are independent and identically distributed random variables with mean zero and finite variance. Suppose that  are the least squares estimators of α and β, respectively. Given the following data:

where yi is the observed value of Yi, i = 1, …, 7. Then the correlation coefficient between equals_____

Answer: (0 to 0)

25. Let {0, 1, 2, 3} be an observed sample of size from N(θ, 5) distribution, where θ ∈ [2, ∞). Then the maximum likelihood estimate of θ based on the observed sample is ________.

Answer: (2 to 2)

Q26 – Q55 carry two marks each.

26. Let f : ℝ × ℝ → ℝ be defined by

f(x, y) = x4 – 2x3y + 16y + 17,

where ℝ denotes the set of all real numbers. Then

(A)  f has a local minimum at (2, 4/3)

(B)  f has a local maximum at (2, 4/3)

(C)  f has a saddle point at (2, 4/3)

(D)  f is bounded

Answer: (C)

27. Consider the linear transformation T : ℂ3 = ℂ × ℂ × ℂ. Which of the following statements is TRUE?

(A)  There exists a non-zero vector X such that T(X) = −X

(B)  There exist a non-zero vector Y and a real number λ ≠ 1 such that T(Y) = λY

(C)  T is diagonalizable

(D)  T2 = I3, where I3 is the 3 × 3 identity matrix

Answer: (C)

28. For real numbers a, b and c, let 

Then, which of the following statements is TRUE?

(A)  Rank (M) = 3 for every, a, b, c ∈ ℝ

(B)  If a + c = 0 then M is diagonalizable for every b ∈ ℝ

(C)  M has a pair of orthogonal eigenvectors for every a, b, c ∈ ℝ

(D)  If b= 0 and a + c = 1 then M is NOT idempotent

Answer: (C)

29. Let M be a 4 × 4 matrix with (x – 1)2 (x – 3)2 as its minimal polynomial. Then, which of the following statements is FALSE?

(A)  The eigenvalues of M are 1 and 3

(B)  The algebraic multiplicity of the eigenvalue 1 is 3

(C)  M is NOT diagonalizable

(D)  Trace(M) = 8

Answer: (B)

30. Let f : ℝ × ℝ → ℝ be defined by 

where ℝ denotes the set of all real numbers. Then which of the following statements is TRUE?

(A)  f is differentiable at (1, 2)

(B)  f is continuous at (1, 2) but NOT differentiable at (1, 2)

(C)  The partial derivative of f, with respect to x, at (1, 2) does NOT exist

(D)  The directional derivative of f at (1, 2) along  equals 1

Answer: (A)

31. Which of the following functions is uniformly continuous on the specified domain?

Answer: (C)

32. Let the random vector  have the joint probability density function

Which of the following statements is TRUE?

(A)  X1, X­2 and X3 are mutually independent

(B)  X1, X2 and X3 are pairwise independent

(C)  (X1, X2) and X3 are independently distributed

(D)  Variance of X1 + X2 is π2

Answer: (B)

33. Suppose that P1 and P2 are two populations having bivariate normal distributions with mean vectors  respectively, and the same variance-covariance matrix  two new observations. If the prior probabilities for P1 and P2 are assumed to be equal and the misclassification costs are also assumed to be equal then, according to linear discriminant rule,

(A)  Z1 is assigned to P1 and Z2 is assigned to P2

(B)  Z1 is assigned to P2 and Z2 is assigned to P1

(C)  both Z1 and Z2 are assigned to P1

(D)  both Z1 and Z2 are assigned to P2

Answer: (B)

34. Let X1, …, Xn be a random sample of size n (≥2) from an exponential distribution with the probability density function

where θ ∈ (0, ∞). Which of the following statements is TRUE?

Answer: (D)

35. Let the joint distribution of (X, Y) be bivariate normal with mean vector  and variance-covariance matrix   where −1 < ρ < 1. Then E[max(X, Y)] equals

(A)   

(B)   

(C)  0

(D)  1/2

Answer: (B)

36. Let  be independent and identically distributed  random vectors, where I3 is the 3 × 3 identity matrix. Let

where J3 is the 3 × 3 matrix with each entry 1 and for any column vector  denotes its transpose. Then the distribution of T is

(A)  central chi-square with 5 degrees of freedom

(B)  central chi-square with 10 degrees of freedom

(C)  central chi-square with 20 degrees of freedom

(D)  central chi-square with 30 degrees of freedom

Answer: (C)

37. Let  be independent and identically distributed  random vectors, where ∑ is a positive definite matrix. Further, let  be a 3 × 4 matrix, where for any matrix M, Mt denotes its transpose. If Wm (n, ∑) denotes a Wishart distribution of order m with n degrees of freedom and variance-covariance matrix ∑, then which of the following statements is TRUE?

(A)  ∑1/2 XtX∑1/2 follows W4(3, I4) distribution

(B)  ∑1/2XtX∑1/2 follows W3(4, I3) distribution

(C)  Trace(X∑1Xt) follows χ24 distribution

(D)  XtX follows W3(4, ∑) distribution

Answer: (A)

38. Let the joint distribution of the random variables X1, X2 and X3 be  where

Then which of the following statements is TRUE?

(A)  X1 – X2 + X3 and X1 are independent

(B)  X1 + X2 and X3 – X1 are independent

(C)  X1 – X2 + X3 and X1 + X2 are independent

(D)  X1 – 2X2 and 2X1 + X2 are independent

Answer: (C)

39. Consider the following one-way fixed effects analysis of variance model

Yij = μ + τi + ϵij, i = 1, 2, 3; j = 1, 2, 3, 4;

where ϵij’s are independent and identically distributed N(0, σ2) random variables, σ ∈ (0, ∞) and τ1 + τ2 + τ3 = 0. Let MST and MSE denote the mean sum of squares due to treatment and the mean sum of squares due to error, respectively. For testing H0 : τ1 = τ2 =  τ3 = 0 against H1 : τi ≠ 0, for some i = 1, 2, 3, consider the test based on the statistic  For positive integers v1 and v2, let Fv1,v2 be a random variable having the central F-distribution with v1 and v2 degrees of freedom. If the observed value of  is given to be 104.45, then the p-value of this test equals

(A)  P(F2, 9 > 104.45)

(B)  P(F9, 2 < 104.45)

(C)  P(F3, 11 < 104.45)

(D)  P(F2, 6 > 104.45)

Answer: (A)

40. Let X1, …, Xn be a random sample of size n (≥ 2) from N(θ, 1) distribution, where θ ∈ (−∞, ∞). Consider the problem of testing H0 : θ ∈ [1, 2] against H1 : θ < 1 or θ > 2, based on X1, …, Xn. Which of the following statements is TRUE?

(A)  Critical region, of level α (0 < α < 1) of uniformly most powerful test for H0 against H1 is of the form  where c1 and c2 are such that test is of level α

(B)  Critical region, of level α(0 < α < 1) of uniformly most powerful test for H0 against H1 is of the form {(x1, …, xn) :   where c and d are such that the test is level α

(C)  At any level α ∈ (0, 1), uniformly most powerful test for H0 against H1 does NOT exist

(D)  At any level α ∈ (0, 1), the power of uniformly most powerful test for H0 against H1 is less than α

Answer: (C)

41. In a pure birth process with birth rates λn = 2n, n ≥ 0, let the random variable T denote the time taken for the population size to grow from 0 to 5. If Var(T) denotes the variance of the random variable T, then 256 × Var(T) = _______

Answer: (341 to 341)

42. Let {Xn}n0 be a homogenous Markov chain whose state space is {0, 1, 2} and whose one-step transition probability matrix is  Then   _____ (correct up to one decimal place).

Answer: (0.7 to 0.7)

43. Let (X, Y) be a random vector such that, for any y > 0, t he conditional probability density function of X given by Y = y is fx|Y=y (x) = yeyx, x > 0.

If the marginal  probability density function of Y is g(y) = yey, y > 0 then E(Y|X = 1) = ______ (correct up to one decimal place).

Answer: (1.5 to 1.5)

44. Let (X, Y) be a random vector with the joint moment generating function

Let Φ(∙) denote the distribution function of the standard normal distribution and p = P(X + 2Y < 1). If Φ(0) = 0.5, Φ(0.5) = 0.6915, Φ(1) = 0.8413 and Φ(1.5) = 0.9332 then the value of 2p + 1 (round off to two decimal places) equals______

Answer: (1.61 to 1.63)

45. Consider a homogeneous Markov chain {Xn}n0 with state space {0, 1, 2, 3} and one-step transition probability matrix

Assume that P(X0 = 1) = 1. Let p be the probability that state 0 will be visited before state 3. Then 6 × p = ______

Answer: (4 to 4)

46. Let (X, Y) be a random vector with joint probability mass function

where  Then the variance of Y equals ______

Answer: (1 to 1)

47. Let X be a discrete random variable with probability mass function f ∈ {f0, f1}, where

The power of the most powerful level α = 0.1 test for testing H0 : X ~ f0 against H1:X ~ f1, based on X, equals _______ (correct up to two decimal places).

Answer: (0.12 to 0.12)

48. Let   be a random vector following  distribution, where  Then the partial correlation coefficient between X2 and X3, with fixed X1, equals _______ (correct up to two decimal places).

Answer: (0.25 to 0.25)

49. Let X1, X2, X3 and X4 be a random sample from a population having probability density function fθ(x) = f(x – θ), −∞ < x < ∞, where θ ∈ (−∞, ∞) and f(−x) = f(x), for all x ∈ (−∞, ∞). For testing H0: θ = 0 against H1: θ < 0, let T+ denote the Wilcoxon Signed-rank statistic. Then under H0, 32 × P(T+ ≤ 5) ______

Answer: (18 to 18)

50. A simple linear regression model with unknown intercept and unknown slope is fitted to the following data

using the method of ordinary least squares. Then the predicted value of y corresponding to x = 5 is _______

Answer: (16 to 16)

51. Let D = {(x, y, z) ∈ ℝ × ℝ × ℝ : 0 ≤ x, y, z ≤ 1, x + y + z ≤ 12}, where ℝ denotes the set of all real numbers. If  then 84 × I = _____

Answer: (7 to 7)

52. Let the random vector (X, Y) have the joint distribution function 

Let Var(X) and Var(Y) denote the variances of random variables X and Y, respectively. Then 16 Var(X) + 32 Var(Y) = _______

Answer: (22 to 22)

53. Let {Xn}n1 be a sequence of independent and identically distributed random variables with E(X1) = 0, E(X12) = 1 and E(X14) = 3. Further, let 

If 

where Φ(∙) denotes the cumulative distribution function of the standard normal distribution, then c2 = ______(correct up to one decimal place.

Answer: (1.5 to 1.5)

54. Let the random vector  have the joint probability density function

Then the variance of the random variable X1 +  X2 + X3 equals ______ (correct up to one decimal place).

Answer: (1.8 to 1.8)

55. Let X1, …, X5 be a random sample from a distribution with the probability density function

where θ (−∞, ∞). For testing H0 : θ = 0 against H1 : θ, let  be the sign test statistic, where

Then the size of the test, which rejects H0 if and only if  equals ______ (correct up to one decimal place).

Answer: (0.5 to 0.5)

GATE Exam 2020 Production and Industrial Engineering (PI) Question Paper With Answer Key

GATE-2020

PI: Production and Industrial Engineering

GA-General Aptitude

Q1 – Q5 carry one mark each.

1. While I agree _____ his proposal this time, I do not often agree _____ him.

(A)  to, with

(B)  with, to

(C)  with, with

(D)  to, to

Answer: (A)

2. The recent measures to improve the output would _______ the level of production to our satisfaction.

(A)  increase

(B)  decrease

(C)  speed

(D)  qualize

Answer: (A)

3. Select the word that fits the analogy:

White : Whitening : Light: ______

(A)  Lightning

(B)  Lightening

(C)  Lighting

(D)  Enlightening

Answer: (B)

4. In one of the greatest innings ever seen in 142 years of Test history, Ben Stokes upped the tempo in a five-and-a-half hour long stay of 219 balls including 11 fours and 8 sixes that saw him finish on a 135 not out as England squared the five-match series.

Based on their connotations in the given passage, which one of the following meanings DOES NOT match?

(A)  upped = increased

(B)  squared = lost

(C)  tempo = enthusiasm

(D)  saw = resulted in

Answer: (B)

5. There are five levels {P, Q, R, S, T} in a linear supply chain before a product reaches customers, as shown in the figure.

At each of the five levels, the price of the product is increased by 25%. If the product is produced at level P at the cost of Rs. 120 per unit, what is the price paid (in rupees) by the customers?

(A)  187.50

(B)  234.38

(C)  292.26

(D)  366.21

Answer: (D)

Q6 – Q10 carry two marks each.

6. Climate change and resilience deal with two aspects – reduction of sources of non-renewable energy resources and reducing vulnerability of climate change aspects. The terms ‘mitigation’ and ‘adaptation’ are used to refer to these aspects, respectively.

Which of the following assertions is best supported by the above in information?

(A)  Mitigation deals with consequences of climate change.

(B)  Adaptation deals with causes of climate change.

(C)  Mitigation deals with actions taken to reduce the use of fossil fuels.

(D)  Adaptation deals with actions taken to combat gree-house gas emissions.

Answer: (C)

7. Find the missing element in the following figure.

(A)  d

(B)  e

(C)  w

(D)  y

Answer: (A)

8. It was estimated that 52 men can complete a strip in a newly constructed highway connecting cities P and Q in 10 days. Due to an emergency, 12 men were sent to another project. How many number of days, more than the original estimate, will be required to complete the strip?

(A)  3 days

(B)  5 days

(C)  10 days

(D)  13 days

Answer: (A)

9. An engineer measures THREE quantities X, Y and Z in an experiment. She finds that they follow a relationship that is represented in the figure below : (the product of X and Y linearly varies with Z)

Then, which of the following statements is FALSE?

(A)  For fixed Z; X is proportional to Y

(B)  For fixed Y; X is proportional to Z

(C)  For fixed X; Z is proportional to Y

(D)  XY/Z is constant

Answer: (A)

10. The two pie-charts given below show the data of total students and only girls registered in different streams in a university. If the total number of students registered in the university is 5000, and the total number of the registered girls is 1500; then, the ratio of boys enrolled in Arts to the girls enrolled in Management is ______.

(A)  2:1

(B)  9:22

(C)  11:9

(D)  22:9

Answer: (D)

PI: Production and Industrial Engineering

Q1 – Q25 carry one mark each.

1. The divergence of the vector  is

(A)  2x

(B)  2y

(C)  2z

(D)  0

Answer: (D)

2. An integrating factor for the differential equation  is

(A)  em

(B)  em

(C)  emx

(D)  emx

Answer: (D)

3. For the complex numbers z1 = 2 + 3i and z2 = 4 – 5i, the value of (z1 + z2)2 is

(A)  32 – 24i

(B)  −32 – 24i

(C)  32 + 24i

(D)  −32 + 24i

Answer: (A)

4. To solve x2 – 2 = 0, the Newton-Raphson method has been employed. If the initial guess x0 = 1.0, the next estimate of the root, x1, will be

(A)  0.5

(B)  1.0

(C)  1.5

(D)  2.0

Answer: (C)

5. If x is a random variable with the expected value of 5 and the variance of 1, then the expected value of x2 is

(A)  24

(B)  25

(C)  26

(D)  36

Answer: (C)

6. Group I lists phases of steel and Group II lists crystal structures in the table below.

Match the phase with the corresponding crystal structure.

(A)  P-2, Q-4, R-3

(B)  P-4, Q-2, R-3

(C)  P-2, Q-4, R-1

(D)  P-4, Q-2, R-1

Answer: (A)

7. The figure shows two bodies P and Q. The body Q is placed on the ground and the body P is placed on top of it. The weights of P and Q are WP and WQ, respectively. The bodies are at rest and all the surfaces are assumed to be frictionless. R represents reaction force, if any, between the bodies.

The correct free body diagram of the body P is

Answer: (B)

8. The figure shows as mechanism with 3 revolute pairs (between the links 1 and 2, 2 and 3, and 3 and 4) and a prismatic pair (between the links 1 and 4). Which one of the four links should be fixed to obtain the mechanism that forms the basis of the quick-return mechanism widely used in a shaper?

(A)  Link 1

(B)  Link 2

(C)  Link 3

(D)  Link 4

Answer: (B)

9. The state of stress at a point in the body under plane stress condition is shown in the figure. The positive directions of x and y axes are also shown. The material of the body is homogeneous and isotropic, with modulus of elasticity E and Poisson’s ratio v. The longitudinal strain in the x-direction is

Answer: (C)

10. The figure shows two bodies connected through a riveted joint with one rivet. The diameter of the rivet is d (in m). The joint transmits a load of F(in N) whose line of action is perpendicular to and intersects the vertical axis of the rivet. Neglect any effect of bending of the rivet. If the allowable shear stress for the material of the rivet is τ N/m2, the diameter of the rivet required to prevent failure in shear is

Answer: (B)

11. Consider flow of an oil with Reynolds number 1500 in a pipe of diameter 5 cm. The kinematic viscosity of the oil, v = 0.75 cm2/s. The value of average velocity in m/s is

(A)  0.75

(B)  1.50

(C)  2.25

(D)  4.50

Answer: (C)

12. A Carnot heat engine receives 600 kJ of heat per cycle from a source at 627°C and rejects heat to a sink at 27° The amount of heat rejected to the sink per cycle (rounded off to the nearest integer) in kJ is

(A)  26

(B)  200

(C)  400

(D)  574

Answer: (B)

13. The process used for producing long bars of fiber reinforced plastics (FRP) with uniform cross-section is

(A)  Extrusion

(B)  Pultrusion

(C)  Injection Molding

(D)  Thermoforming

Answer: (B)

14. The purpose of the ratchet in a micrometer is to

(A)  impart smooth movement to the spindle

(B)  compensate for the wear of the screw thread

(C)  prevent rotation of the spindle while reading the scale

(D)  maintain sufficient and uniform measuring pressure

Answer: (D)

15. End mill cutters are mounted on the spindle of a vertical milling machine using

(A)  vice

(B)  collet

(C)  fact plate

(D)  driver plate

Answer: (B)

16. Self-sharpening tendency of a conventional grinding wheel depends upon

(A)  wheel structure

(B)  wheel grade

(C)  grit hardness

(D)  grit size

Answer: (B)

17. A non-traditional machining process which utilizes mechanical energy as the principal energy source for removing the material is

(A)  Electric discharge machining

(B)  Laser beam machining

(C)  Ultrasonic machining

(D)  Plasma arc machining

Answer: (C)

18. In a manufacturing of self-lubricating bearings by powder metallurgy, an important secondary operation that is carried out after sintering is

(A)  Cold isostatic pressing

(B)  Hot isostatic pressing

(C)  Impregnation

(D)  Infiltration

Answer: (C)

19. Which of the following is a causal forecasting method?

(A)  Naïve approach

(B)  Moving average

(C)  Exponential smoothing

(D)  Linear regression

Answer: (D)

20. An approach used in the product development which combines the efforts of design, manufacturing, and other functions to reduce the total time in introducing a new product in the market is

(A)  Concurrent engineering

(B)  Lean manufacturing

(C)  Value engineering

(D)  Break-even anlaysis

Answer: (A)

21. The Bellman’s principle of optimality is related to

(A)  Linear programming problem

(B)  Transportation problem

(C)  Dynamic programming problem

(D)  Assignment problem

Answer: (C)

22. The process capability ratio Cp is given by

Answer: (A)

23. In a uniaxial tensile test on a specimen of a ductile material, the ultimate tensile strength is found to be 400 MPa and the elongation up to the maximum load is 25%. The true stress at the maximum load in MPa is ______.

Answer: (500 to 500)

24. Suppose the control system of a fighter jet consists of three unrelated components in series, and it is desired to have 98% reliability of the system. If the reliability level of all the components is the same, then the reliability of each component (rounded off to three decimal places) is ________.

Answer: (0.993 to 0.994 OR 99.300 to 99.400)

25. The product structure tree in the figure below shows the components needed to assemble one unit of product P.

The number of units of component D needed to assemble 10 units of product P is ___________.

Answer: (220 to 220)

Q26 – Q55 carry two marks each.

26. General solution of  is

Answer: (C)

27. For the matrix  the eigenvectors are

Answer: (B)

28. A truss with two bars PR and QR, making angles α and β, respectively, with the vertical, is shown in the figure below. The connections at P, Q and R are hinged connections. The truss supports body of weight W (in N) at R as shown. The tension in the bar QR(in N) is

Answer: (D)

29. The figure shows a beam of length L (in m) with a uniformly distributed transverse load of W (in N/m) acting over it. The width and depth of the beam cross section are b (in m) and t (in m), respectively. The magnitude of the maximum bending stress in the beam in N/m2 is

Answer: (A)

30. The vertices of rectangle PQRS are as follows in a 2-D CAD system.

P(-4, -2); Q(-2, -3); R(-3, -5); S(-5, -4)

The coordinates of the corresponding new vertices, P’, Q’, R’, S’ after translation of the rectangle along x-axis in the positive direction by 6 units and along y-axis in the positive direction by 3 units are

(A)  P’(-10, -5); Q’(-8, -6); R’(-9, -8); S’(-11, -7)

(B)  P’(2, 1); Q’(4, 0); R’(3, -2); S’ (1, -1)

(C)  P’(2, -5); Q’(4, -6); R’(3, -8); S’(1, -7)

(D)  P’(-10, 1); Q’(-8, 0); R’(-9, -2); S’(-11, -1)

Answer: (B)

31. The statement that best describes the function of a GO gauge in the context of Taylor’s principle of gauging is

(A)  GO gauge checks the Maximum Material Condition and is designed to check as many dimensions as possible

(B)  GO gauge checks the Least Material Condition and is designed to check as many dimensions as possible

(C)  Go gauge checks the maximum Material Condition and is designed to check only one dimension

(D)  GO gauge checks the Least Material Condition and is designed to check only one dimension

Answer: (A)

32. The figures shows revenue generated over different product life cycle stages marked as P, Q, R and S. Group I lists these product life cycle stages. Group II lists typical efforts leading to revenue maximizing during a stage.

Match the stage with the efforts.

(A)  P-3; Q-4; R-2; S-1

(B)  P-1; Q-4; R-2; S-3

(C)  P-1; Q-3; R-4; S-2

(D)  P-3; Q-1; R-2; S-4

Answer: (D)

33. A company manufactures products P and Q in quantities x1 and x2, respectively, using two resources. The following Linear Programming Problem (LPP) is Maximize Z = 3x1 + 2x2

subject to x1 + 2x2 ≤ 2 (for Resource 1)

               2x1 + x2 ≤ 2 (for Resource 2)

   and x1, x2 ≥ 0.

The shadow price for Resource 2 is

(A)  0

(B)  2/3

(C)  1/

(D)  4/3

Answer: (D)

34. A rectifying inspection is performed on a lot of size W = 1000 using a Single-Sampling Plant with the sample size n = 60 and the acceptance number c = 1. If t he Acceptable Quality Level is 1.0%, the producer’s risk associated with the sampling plan (rounded off to the nearest integer) in % is

(A)  12

(B)  33

(C)  67

(D)  88

Answer: (A)

35. For y = −x2 + 9x – 2, the value of  using Simpson’s 1/3 rule with two intervals (rounded off to two decimal places) is ______.

Answer: (58.00 to 59.00)

36. If the probability density function of a random variable x is given by

                                         

the value of k is ______.

Answer: (3 to 3)

37. A solid shaft has to transmit 50 kW of power at a speed of 1910 RPM. Ignore any possible bending of the shaft. The maximum allowable shear stress for the material of the shaft is 80 MPa. The minimum diameter of the shaft required to prevent failure due to shear (rounded off to one decimal place) in cm is _______.

Answer: (2.0 to 3.0)

38. A flywheel is to be used in an IC engine to limit fluctuation of angular speed. The average of the maximum and the minimum angular speed is 500 RPM, and the maximum fluctuation of energy is 10,000 N-m. Neglecting rotary inertia of any other components, the moment of inertia of the flywheel about its axis of rotation required to limit the maximum fluctuation of speed of 30 RPM (rounded off to one decimal place) in kg-m2 is ________.

Answer: (59.0 to 62.0)

39. A tank of large cross-sectional area contains water up to a height of 5 m as shown in the figure. The top water surface is under a pressure of p1 = 0.2 MPa. A small, smooth and round tap at the bottom of the tank is opened to the atmosphere (p2 = 0.1 MPa).

Use the acceleration due to gravity, g = 9.81 m/s2 and the density of water, ρ = 1000 kg/m3. The velocity with which the water will exit from the tap under the conditions shown in the figure (rounded off to one decimal place) is m/s is _______.

Answer: (17.2 to 17.4)

40. A steel ball of 12 mm diameter is heated to 1225 K. It is then slowly cooled in air to a temperature of 475 K. During the cooling process, the ambient temperature is 325 K and the heat transfer coefficient is 30 W/m2-k. Assume the density of steel is 7800 kg/m3 and the specific heat is 600 J/kg-K. Using the lumped capacitance method of analysis, the calculated time for the required cooling (rounded off to one decimal place) in second is _______.

Answer: (557.0 to 561.0)

41. A mass of 3 kg of Argon gas at 3 bar, 27°C is contained in a rigid, insulated vessel. Paddle wheel work is done on the gas for 30 minutes at the rate of 0.015 kW. Specific heat at constant volume, Cv, for Argon is 0.3122 kJ/kg-K. The final temperature of the gas (rounded off to one decimal place) in Kelvin is ________.

Answer: (327.0 to 331.0)

42. The figure shows drawing of a part with dimensions and tolerances, both in mm. The permissible tolerance for slot A (rounded off to one decimal place) in mm is ±_______.

Answer: (0.6 to 0.6)

43. To manufacture a product by casting, molten metal is poured in a cavity of rectangular cross section in a sand mold with a side blind riser as shown in the figure. The dimensions of the mold cavity are 60 cm × 40 cm × 20 cm.

The riser is cylindrical in shape with diameter equal to height. It is required that the solidification time of the riser should be 25% greater than that of the mold. Using Chvorinov’s rule, the diameter of the riser (rounded off to one decimal place) in cm should be ______.

Answer: (36.0 to 37.0)

44. A cylindrical billet of 90 mm diameter is extruded to produce an I-section as shown in the figure (all dimensions in mm).

The total extrusion pressure (p­e) in MPa required for the above process is given by

                                           

where, σm is the mean flow stress of the material, and A0 and Af are the initial and the final cross-sectional areas, respectively. If the mean flow stress of the extruded material is 80 MPa, the force required for the above extrusion (rounded off to one decimal place) in kN is _________.

Answer: (1672.0 to 1675.0)

45. The heat generated in resistance spot welding operation for joining two metal sheets with a certain set of process parameters is 2000 J. For a second spot welding operation on the same sheets without any change in the overall resistance of the system, the current is increased by 25% and the time for which the current is applied is reduced to half. The heat generated in the second operation (rounded off to one decimal place) in J is _______.

Answer: (1560.0 to 1563.0)

46. A vertical boring operation is performed in a cast iron plate to enlarge a blind hole to a diameter of 25 mm up to a depth of 100 mm in a single pass. The cutting speed and the feed used in the process are 100 m/min and 0.1 mm/rev, respectively. Considering the allowance for tool approach as 2 mm, the actual machining time (rounded off to two decimal places) in minutes is ________.

Answer: (0.78 to 0.82)

47. For a particular tool-workpiece combination, the value of exponent n in Taylor’s tool life equation is 0.5. If the cutting speed is reduced by 50% keeping all the other machining conditions same, the increase in tool life in % is _______.

Answer: (300 to 300)

48. In a waterjet machining process, the water pressure used is 500 MPa. The diameter of orifice of the nozzle through which the waterjet comes out is 0.25 mm. Neglecting frictional and other losses, and using the density of water as 1000 kg/m3, the mass flow rate of the waterjet (rounded off to two decimal places) in kg/min is _________.

Answer: (2.85 to 3.00)

49. The movement along the z-axis of a CNC drilling machine is controlled by using a servo motor, a lead screw and an incremental encoder. The lead screw has 2 threads/cm and it is directly coupled to the servo motor. The incremental encoder attached to the lead screw emits 100 pulses/revolution. The control resolution in microns is _______.

Answer: (50 to 50)

50. A project consists of seven activities as listed in the table. The time required for each activity and its immediate predecessor(s) are also given.

The project completion time using Critical Path Method (CPM) in weeks is  _______.

Answer: (20 to 20)

51. A company is planning to procure a machine to produce a component. There are two alternatives available – machine A and machine B. The cost of producing x units of the component (in Rs.) using machine A is given as C(x) = 10000 + 170x + x2. The cost of producing x units of the component (in Rs.) using machine B is given as CB(x) = 15000 + 400x. If machine B is to be preferred, then the minimum number of units to be produced should be ________.

Answer: (250 to 250)

52. The availability of an old photocopier was 90% and the Mean Time between Failure (MTBF) was 200 days. It has been replaced with a new photocopier having an availability of 95%. Now, the Mean Time to Repair (the time during which the photocopier is unavailable for service) has increased by 5 days. The MTBF of the new photocopier (rounded off to the nearest integer) in days is _______.

Answer: (500 to 523)

53. A car company manufactures 200 units of a component per day. The component is installed in different car models at a rate of 15000 units per year. The company operates its production facility 300 days per year to manufacture the component. The setup cost for each production run is Rs. 200 and the inventory holding cost per year is Rs. 2 per The Economic Production Quantity (EPQ) is ________.

Answer: (2000 to 2000)

54. A company has to perform five tasks (P, Q, R, S and T) to make an assembly. Task times and immediate predecessors of the tasks are listed below. If an assembly line is designed to obtain the maximum output rate, the efficiency of the line in % is ______.

Answer: (95 to 95)

55. In a work study experiment, it is observed that a worker completes a job in an average time of 15 minutes with a performance rating of 120%. The time required for another worker having a performance rating of 80% to complete the same job (rounded off to one decimal place) in minutes is ______.

Answer: (22.5 to 22.5)

GATE Exam 2020 Physics (PH) Question Paper With Answer Key

GATE-2020

PH: Physics

GA – General Aptitude

Q1 – Q5 carry one mark each.

1. He is known for his unscrupulous ways. He always sheds______ tears to deceive people.

(A)  fox’s

(B)  crocodile’s

(C)  crocodile

(D)  fox

Answer: (C)

2. Jofra Archer, the England fast bowler, is ____ than accurate.

(A)  more fast

(B)  faster

(C)  less fast

(D)  more faster

Answer: (A)

3. Select the word that fits the analogy:

Build : Building :: Grow : ______

(A)  Grown

(B)  Grew

(C)  Growth

(D)  Growed

Answer: (C)

4. I do not think you know the case well enough to have opinions. Having said that, I agree with your other point.

What does the phrase “having said that” mean in the given text?

(A)  as opposed to what I have said

(B)  despite what I have said

(C)  in addition to what I have said

(D)  contrary to what I have said

Answer: (B)

5. Define [x] as the greatest integer less than or equal to x, for each x ∈ (−∞, ∞). If y = [x], then area under y for x ∈ [1, 4] is _______.

(A)  1

(B)  3

(C)  4

(D)  6

Answer: (D)

Q6 – Q10 carry two marks each.

6. Crowd funding deals with mobilization of funds for a project from a large number of people, who would be willing to invest smaller amounts through web-based platforms in the project.

Based on the above paragraph, which of the following is correct about crowd funding?

(A)  Funds raised through unwilling contributions on web-based platforms.

(B)  Funds raised through large contributions on web-based platforms.

(C)  Funds raised through coerced contributions on web-based platforms.

(D)  Funds raised through voluntary contributions on web-based platforms.

Answer: (D)

7. P, Q, R and S are to be uniquely coded using α and β. If P is coded as αα and Q as αβ, then R and S, respectively, can be coded as ______.

(A)  βα and αβ

(B)  ββ and αα

(C)  αβ and ββ

(D)  βα and ββ

Answer: (D)

8. The sum of the first n terms in the sequence 8, 88, 888, 8888, … is ______.

(A)   

(B)   

(C)   

(D)   

Answer: (D)

9. Select the graph that schematically represents BOTH y = xm and y = x1/m properly in the interval 0 ≤ x ≤ 1, for integer values of m, where m > 1.

Answer: (A)

10. The bar graph shows the data of the students who appeared and passed in an examination for four schools P, Q, R and S. The average of success rates (in percentage) of these four schools is ________.

(A)  58.5%

(B)  58.8%

(C)  59.0%

(D)  59.3%

Answer: (C)

PH: Physics

Q1 – Q25 carry one mark each.

1. Which one of the following is a solution of  for k real?

(A)  ekx

(B)  sin kx

(C)  cos kx

(D)  sin hx

Answer: (A)

2. A real, invertible 3 × 3 matrix M has eigenvalues λi, (i = 1, 2, 3) and the corresponding eigenvectors are  Which one of the following is correct?

Answer: (B)

3. A quantum particle is subjected to the potential

The ground state wave function of the particle is proportional to

Answer: (D)

4. Let  respectively denote the lowering and raising operators of a one-dimensional simple harmonic oscillator. Let  be the energy eigenstate of the simple harmonic oscillator. Given that is also an eigenstate of   the corresponding eigenvalue is

(A)  n(n – 1)

(B)  n(n + 1)

(C)  (n + 1)2

(D)  n2

Answer: (A)

5. Which one of the following is a universal logic gate?

(A)  AND

(B)  NOT

(C)  OR

(D)  NAND

Answer: (D)

6. Which one of the following is the correct binary equivalent of the hexadecimal F6C?

(A)  0110 1111 1100

(B)  1111 0110 1100

(C)  1100 0110 1111

(D)  0110 1100 0111

Answer: (B)

7. The total angular momentum j of the ground state of the  nucleus is

(A)  1/2

(B)  1

(C)  3/2

(D)  5/2

Answer: (D)

8. A particle X is produced in the process π+ + p → K+ + X via the strong interaction. If the quark content of the K+ is , the quark content of X is

(A)   

(B)  u u d

(C)  u u s

(D)   

Answer: (C)

9. A medium (εr > 1, μr = 1, σ > 0) is semi-transparent to an electromagnetic wave when

(A)  Conduction current >> Displacement current

(B)  Conduction current << Displacement current

(C)  Conduction current = Displacement current

(D)  Both Conduction current and Displacement current are zero

Answer: (B)

10. A particle is moving in a central force field given by  is the unit vector pointing away from the center of the field. The potential energy of the particle is given by

(A)  k/r2

(B)  k/2r2

(C)  −k/r2

(D)  −k/2r2

Answer: (D)

11. Choose the correct statement related to the Fermi energy (EF) and the chemical potential (μ) of a metal.

(A)  μ = EF only at 0 K

(B)  μ = EF at finite temperature

(C)  μ < EF at 0 K

(D)  μ > EF at finite temperature

Answer: (A)

12. Consider a diatomic molecule formed by identical atoms. If EV and Ee represent the energy of the vibrational nuclear motion and electronic motion respectively, then in terms of the electronic mass m and nuclear mass M, EV/Ee is proportional to

(A)  (m/M)1/2

(B)  m/M

(C)  (m/M)3/2

(D)  (m/M)2

Answer: (A)

13. Which one of the following relations determines the manner in which the electric field lines are refracted across the interface between two dielectric media having dielectric constants ε1 and ε2 (see figure) ?

(A)  ε1 sinθ1 = ε2 sinθ2

(B)  ε1 cosθ1 = ε2 cosθ2

(C)  ε1 tanθ1 = ε2 tanθ2

(D)  ε1 cotθ1 = ε2 cotθ2

Answer: (D)

14. If  are the electric and magnetic fields respectively, then  is

(A)  odd under parity and even under time reversal

(B)  even under parity and odd under time reversal

(C)  odd under parity and odd under time reversal

(D)  even under parity and even under time reversal

Answer: (C)

15. A small disc is suspended by a fiber such that it is free to rotate about the fiber axis (see figure). For small angular deflections, the Hamiltonian for the disc is given by

                                                             

where I is the moment of inertia and α is the restoring torque per unit deflection. The disc is subjected to angular deflections (θ) due to thermal collisions from the surrounding gas at temperature T and p0 is the momentum conjugate to θ. The average and t he root-mean-square angular deflection, θavg and θrms, respectively are

Answer: (B)

16. As shown in the figure, an ideal gas is confined to chamber A of an insulated container, with vacuum in chamber B. When the plug in the wall separating the chambers A and B is removed, the gas fills both the chambers. Which one of the following statements is true?

(A)  The temperature of the gas remains unchanged

(B)  Internal energy of the gas decreases

(C)  Temperature of the gas decreases as it expands to fill the space in chamber B

(D)  Internal energy of the gas increases as its atoms have more space to move around

Answer: (A)

17. Particle A with angular momentum j = 3/2 decays into two particles B and C with angular momenta j1 and j2, respectively. If  the value of α is ______.

Answer: (1 to 1)

18. Far from the Earth, the Earth’s magnetic field can be approximated as due to bar magnet of magnetic pole strength 4 × 1014 Assume this magnetic field is generated by a current carrying loop encircling the magnetic equator. The current required to do so is about 4 × 10n A, where n is an integer. The value of n is _____.

(Earth’s circumference: 4 × 107 m)

Answer: (7 to 7)

19. The number of distinct ways the primitive unit cell can be constructed for the two dimensional lattice as shown in the figure is_____.

Answer: (5 to 5)

20. A hydrogenic atom is subjected to a strong magnetic field. In the absence of spin-orbit coupling, the number of doubly degenerated states created out of the d-level is _______.

Answer: (3 to 3)

21. A particle Y undergoes strong decay Y → π + π. The isospin of Y is ______.

Answer: (2 to 2)

22. For a complex variable z and the contour c: |z| = 1 taken in the counter clockwise direction, _______.

Answer: (-2 to -2)

23. Let p be the momentum conjugate to the generalized coordinate q. If the transformation

Q = √2qm cos p

P = √2qm sin p

is canonical, then m = ______.

Answer: (0.5 to 0.5)

24. A conducting sphere of radius 1 m is placed in air. The maximum number of electrons that can be put on the sphere to avoid electrical breakdown is about 7 × 10n, where n is integer. The value of n is ______.

Assume:

Breakdown electric field strength in air is   

Permittivity of free space ε0 = 8.85 × 1012 F/m

Electron charge e = 1.60 × 1019 C

Answer: (14 to 15)

25. If a particle is moving along a sinusoidal curve, the number of degrees of freedom of the particle is ______.

Answer: (1 to 1)

Q26 – Q55 carry two marks each.

26. The product of eigenvalues of  is

(A)  −1

(B)  1

(C)  0

(D)  2

Answer: (A)

27. Let  Let ℝ3 denote the three-dimensional real vector space. Which one of the following is correct?

(A)  S is an orthonormal set

(B)  S is a linearly dependent set

(C)  S is a basis for ℝ3

(D)

Answer: (C)

28. denotes the spin operator defined as  Which one of the following is correct?

(A)  The eigenstates of spin operator  

(B)  The eigenstates of spin operator  

(C)  In the spin state upon the measurement of , the probability for obtaining  

(D)  In the spin state  upon the measurement of , the probability for obtaining  

Answer: (D)

29. The input voltage (Vin) to the circuit shown in the figure is 2cos(100t) V. The output voltage (Vout) is  If R = 1 kΩ, the value of C (in μF) is

(A)  0.1

(B)  1

(C)  10

(D)  100

Answer: (C)

30. Consider a 4-bit counter constructed out of four flip-flops. It is formed by connecting the J and K inputs to logic high and feeding the Q output to the clock input of the following flip-flop (see the figure). The input signal to the counter is series of square pulses and the change of state is triggered by the falling edge. At time t = t0 the outputs are in logic low state (Q0 = Q1 = Q2 = Q3 = 0). Then at t = t1, the logic state of the outputs is

(A)  Q0 = 1, Q1 = 0, Q2 = 0 and Q3 = 0

(B)  Q0 = 0, Q1 = 0, Q2 = 0 and Q3 = 1

(C)  Q0 = 1, Q1 = 0, Q2 = 1 and Q3 = 0

(D)  Q0 = 0, Q1 = 1, Q2 = 1 and Q3 = 1

Answer: (B)

31. Consider the Lagrangian  where a, b and c are constants. If px and py are the momenta conjugate to the coordinates x and y respectively, then the Hamiltonian is

Answer: (A)

32. Which one of the following matrices does NOT represent a proper rotation in a plane?

Answer: (D)

33. A uniform magnetic field  exists in an inertial frame K. A perfect conducting sphere moves with a constant velocity  with respect to this inertial frame. The rest frame of the sphere is K’ (see figure). The electric and magnetic fields in K and K’ are related as

The induced surface charge density on the sphere (to the lowest order in v/c) in the frame K’ is

(A)  maximum along z’

(B)  maximum along y’

(C)  maximum along x’

(D)  uniform over the sphere

Answer: (A)

34. A charge q moving with uniform speed enters a cylindrical region in free space at t = 0 and exits the region at t = τ (see figure). Which one of the following options best describes the time dependence of the total electric flux φ(t), through the entire surface of the cylinder?

Answer: (D)

35. Consider a one-dimensional non-magnetic crystal with one atom per unit cell. Assume that the valence electrons (i) dot not interact with each other and (ii) interact weakly with the ions. If n is the number of valence electrons per unit cell, then at 0 K,

(A)  the crystal is metallic for any value of n

(B)  the crystal is non-metallic for any value of n

(C)  the crystal is metallic for even values of n

(D)  the crystal is metallic for odd values of n

Answer: (D)

36. According to the Fermi gas model of the nucleus, the nucleons move in a spherical volume of radius R(=R0A1/3, where A is the mass number and R0 is an empirical constant with the dimensions of length). The Fermi energy of the nucleus EF is proportional to

(A)  R02

(B)  1/R0

(C)  1/R02

(D)  1/R03

Answer: (C)

37. Consider a two dimensional crystal with 3 atoms in the basis. The number of allowed optical branches (n) and acoustic branches (m) due to the lattice vibrations are

(A)  (n, m) = (2, 4)

(B)  (n, m) = (3, 3)

(C)  (n, m) = (4, 2)

(D)  (n, m) = (1, 5)

Answer: (C)

38. The internal energy U of a system is given by U(S, V) = λV2/3S2, where λ is a constant of appropriate dimensions; V and S denote the volume and entropy, respectively. Which one of the following gives the correct equation of state of the system?

Answer: (A)

39. The potential energy of a particle of mass m is given by U(x) = a sin(k2x – π/2), a > 0, k2 > 0. The angular frequency of small oscillations of the particle about x = 0 is

Answer: (B)

40. The radial wave function of a particle in a central potential is given by  where A is the normalization constant and a is positive constant of suitable dimensions. If γa is the most probable distance of the particle from the force center, the value of γ is _________.

Answer: (4 to 4)

41. A free particle of mass M is located in a three-dimensional cubic potential well with impenetrable walls. The degeneracy of the fifth excited state of the particle is __________.

Answer: (6 to 6)

42. Consider the circuit given in the figure. Let the forward voltage drop across each diode be 0.7 V. The current I (in mA) through the resistor is ____________.

Answer: (8 to 8)

43. Let uu denote the 4-velocity of a relativistic particle whose squre uuuμ = 1. If εμvρσ is the Levi-Civita tensor then the value of εμvρσuμuvuρuσ is ______.

Answer: (0 to 0)

44. Consider a simple cubic monoatomic Bravias lattice which has a basis with vectors  a is the lattice parameter. The Bragg reflection is observed due to t he change in the wave vector between the incident and the scattered beam as given by  where  are primitive reciprocal lattice vectors. For n1 = 3, n2 = 3 and n3 = 2, the geometrical structure factor is _______.

Answer: (2 to 2)

45. A plane electromagnetic wave of wavelength λ is incident on a circular loop of conducting wire. The loop radius is a(a << λ). The angle (in degrees), made by the Poynting vector with t he normal to the plane of the loop to generate a maximum induced electrical signal, is _______.

Answer: (-270 to -270 or -90 to -90 or 90 to 90 or 270 to 270)

46. An electron in a hydrogen atom is in the state n = 3, l = 2, m = − Let  denote the y-component of the orbital angular momentum operator. If  the value of α is ________.

Answer: (1 to 1)

47. A sinusoidal voltage of the form V(t) = V0 cos(ωt) is applied across a parallel plate capacitor placed in vacuum. Ignoring the edge effects, the induced emf within the region between the capacitor plates can be expressed as a power series in ω. The lowest non-vanishing exponent in ω is ________.

Answer: (2 to 2)

48. If  for –π ≤ x ≤ π, the value of a2 is ______.

Answer: (-1 to -1)

49. Let  

The value of  is _______.

Answer: (0 to 0)

50. Consider the Hamiltonian  where

 is the time independent penturbation given by

  where k > 0. If the maximum energy eigenvalue of  is 3 eV corresponding to E = 2 eV, the value of k (rounded off to three decimal places) in eV is ______.

Answer: (0.706 to 0.708)

51. A hydrogen atom is in an orbital angular momentum state  lies on a cone which makes a half angle 30° with respect to the z-axis, the value of l is _______.

Answer: (3 to 3)

52. In the center of mass frame, two protons each having energy 7000 GeV, collide to produce protons and anti-protons. The maximum number of anti-protons produced is_______.

(Assume the proton mass to be 1 GeV/c2)

Answer: (6999 to 6999)

53. Consider a gas of hydrogen atoms in the atmosphere of the Sun where the temperature is 5800 K. If a sample from this atmosphere contains 6.023 × 1023 of hydrogen atoms in the ground state, the number of hydrogen atoms in the first excited state is approximately 8 × 10n, where n is an integer. The value of n is ______.

(Boltzmann constant: 8.617 × 10−5 eV/K)

Answer: (14 to 15)

54. For a gas of non-interacting particles, the probability that a particle has a speed v in the interval v to v + dv is given by

             

If E is the energy of a particle, then the maximum in the corresponding energy distribution in units of E/kBT occurs at _______ (rounded off to one decimal place).

Answer: (0.5 to 0.5)

55. The Planck’s energy density distribution is given by  At long wavelengths, the energy density of photons in thermal equilibrium with a cavity at temperature T varies as Tα, where α is _______.

Answer: (1 to 1)

GATE Exam 2020 Petroleum Engineering (PE) Question Paper With Answer Key

GATE-2020

PE-Petroleum Engineering

GA – General Aptitude

Q1 – Q5 carry one mark each.

1. He is known for his unscrupulous ways. He always sheds______ tears to deceive people.

(A)  fox’s

(B)  crocodile’s

(C)  crocodile

(D)  fox

Answer: (C)

2. Jofra Archer, the England fast bowler, is ____ than accurate.

(A)  more fast

(B)  faster

(C)  less fast

(D)  more faster

Answer: (A)

3. Select the word that fits the analogy:

Build : Building :: Grow : ______

(A)  Grown

(B)  Grew

(C)  Growth

(D)  Growed

Answer: (C)

4. I do not think you know the case well enough to have opinions. Having said that, I agree with your other point.

What does the phrase “having said that” mean in the given text?

(A)  as opposed to what I have said

(B)  despite what I have said

(C)  in addition to what I have said

(D)  contrary to what I have said

Answer: (B)

5. Define [x] as the greatest integer less than or equal to x, for each x ∈ (−∞, ∞). If y = [x], then area under y for x ∈ [1, 4] is _______.

(A)  1

(B)  3

(C)  4

(D)  6

Answer: (D)

Q6 – Q10 carry two marks each.

6. Crowd funding deals with mobilization of funds for a project from a large number of people, who would be willing to invest smaller amounts through web-based platforms in the project.

Based on the above paragraph, which of the following is correct about crowd funding?

(A)  Funds raised through unwilling contributions on web-based platforms.

(B)  Funds raised through large contributions on web-based platforms.

(C)  Funds raised through coerced contributions on web-based platforms.

(D)  Funds raised through voluntary contributions on web-based platforms.

Answer: (D)

7. P, Q, R and S are to be uniquely coded using α and β. If P is coded as αα and Q as αβ, then R and S, respectively, can be coded as ______.

(A)  βα and αβ

(B)  ββ and αα

(C)  αβ and ββ

(D)  βα and ββ

Answer: (D)

8. The sum of the first n terms in the sequence 8, 88, 888, 8888, … is ______.

(A)   

(B)   

(C)   

(D)   

Answer: (D)

9. Select the graph that schematically represents BOTH y = xm and y = x1/m properly in the interval 0 ≤ x ≤ 1, for integer values of m, where m > 1.

Answer: (A)

10. The bar graph shows the data of the students who appeared and passed in an examination for four schools P, Q, R and S. The average of success rates (in percentage) of these four schools is ________.

(A)  58.5%

(B)  58.8%

(C)  59.0%

(D)  59.3%

Answer: (C)

PE: Petroleum Engineering

Q1 – Q25 carry one mark each.

1. Consider a vector field,    are the unit vectors along the x,  y, and z directions, respectively. The divergence of A at the point (1, 1, 1) is equal to

(A)  0

(B)  2

(C)  3

(D)  4

Answer: (D)

2. Inverse Laplace transform of the function,  is given by

(A)  1 – et

(B)  1 + et

(C)  1 – et

(D)  1 + et

Answer: (C)

3. The solution of the differential equation,  with the condition y = 1 at x = 1, is given by

Answer: (D)

4. Two complex numbers are given as,  where, I = √−1, and θ1 and θ2 are the principal arguments, Given, θ1 ≠ θ2 and |θ1 – θ2| ≠ π. If  which one of the following conditions is correct?

(A)  2 < m < 3

(B)  0 < m < 2

(C)  m = 2

(D)  m = 0

Answer: (B)

5. Match the following

(A)  P-I, Q-II, R-III

(B)  P-II, Q-I, R-III

(C)  P-I, Q-III, R-II

(D)  P-III, Q-I, R-II

Answer: (D)

6. Shear stress versus shear rate plots for four different fluids are given in the Figure. Which curve represents a pseudoplastic fluid?

(A)  I

(B)  II

(C)  III

(D)  IV

Answer: (C)

7. Which one of the following is NOT a desired function of a hydraulic fracturing fluid additive?

(A)  Oxygen scavenging to prevent attack on polymers.

(B)  Increasing viscosity of fracturing fluid during flow back.

(C)  Work as bactericide.

(D)  Work as surfactant to facilitate post treatment clean-up.

Answer: (B)

8. Formation damage could be a result of

(i) scale formation near the wellbore

(ii) coke formation due to in-situ combustion

(iii) precipitation of asphaltene

(iv) condensate banking

Which one of the following options is correct?

(A)  (i) and (iv) only

(B)  (i) and (iii) only

(C)  (i), (ii), and (iii) only

(D)  (i), (ii), (iii) and (iv)

Answer: (D)

9. Which of the following statement(s) about gas and water coning in the reservoir is/are correct?

(i) Gas and water coning is characterized by downward movement of water and  upward movement of gas near the producing wellbore.

(ii) Gas and water coning is characterized by downward movement of gas and upward movement of water near the producing wellbore.

(iv) Gas and water coning is caused when gravitational forces dominate over viscous forces.

(A)  (i) and (iv) only

(B)  (ii) only

(C)  (ii), (iii), and (iv) only

(D)  (iv) only

Answer: (B)

10. Given the figure

Which one of the following options represents the correct combination of the trajectory number and the corresponding drilling type?

(A)  i → Build and Hold, ii → S-Type, iii → Modified S-Type, iv → Continuous Build

(B)  i → Continuous Build, ii → Build and Hold, iii → Modified S-Type, iv → S-Type

(C)  i → Continuous Build, ii → S-Type, iii → Modified S-Type, iv → Build and Hold

(D)  i → Build and Hold, ii → Modified S-Type, iii → S-Type, iv → Continuous Build

Answer: (B)

11. A stable geothermal gradient (approx. 25°C/km) in the earth’s crust will suddenly increase to a higher gradient value, when

(A)  there is excessive erosion or upliftment

(B)  there is excessive subsidence or deposition

(C)  there is excessive subsidence and  upliftment simultaneously

(D)  there is excessive subsidence and erosion simultaneously

Answer: (A)

12. A drawdown test is conducted at a constant flow rate in an oil well for a reservoir with constant compressibility. Which one of the following is valid for semi steady state condition?

(A)  Rate of pressure change at the wellbore is less than that at the boundary.

(B)  The effect of the outer boundary of the reservoir is felt at the wellbore.

(C)  Reservoir permeability does not affect the wellbore pressure.

(D)  Pressure in the reservoir does not change with time.

Answer: (B)

13. Formation volume factor (B0) versus Pressure (P) plot for an oil is given in the Figure.

Match the following with the corresponding pressure given in the Figure.

(I) Bubble point

(II) Saturated oil

(III) Under-saturated oil

(A)  I-P1, II-P2, III-P3

(B)  I-P1, II-P3, III-P2

(C)  I-P2, II-P1, III-P3

(D)  I-P2, II-P3, III-P1

Answer: (C)

14. Which one of the following statements is NOT correct?

(A)  Flash point of gasoline is lower than that of diesel.

(B)  Pour point is the temperature at which oil creases to flow.

(C)  Higher the Diesel Index of a fuel, higher is its cetane number.

(D)  Higher the aromatic content of diesel, higher is its aniline point.

Answer: (D)

15. Which one of the following additives is commonly added to drilling fluids to remove hydrogen sulfide?

(A)  Sodium chloride

(B)  Calcium chloride

(C)  Zinc carbonate

(D)  Bentonite

Answer: (C)

16. Two rigid spherical particles of the same density, with a diameter ratio D1 : D2 = 1:2, settle freely through a pool of liquid. The terminal settling velocity is given by the Stoke’s law. What is the ratio of their terminal settling velocities, V1 : V2 ?

(A)  1:2

(B)  2:1

(C)  1:4

(D)  4:1

Answer: (C)

17. Which one of the following options best represents the correct order of increasing thermal conductivity of the subsurface formations?

(A)  Coal < Shale < Dolomite < Evaporite

(B)  Evaporite < Shale < Coal < Dolomite

(C)  Coal < Shale < Evaporite < Dolomite

(D)  Shale < Coal < Evaporite < Dolomite

Answer: (A)

18. Which one of the following options is the correct combination of kerogen Type and the source from which it is derived?

(A)  Type I-Lacustrine, Type II-Terrestrial, Type III-Marine, Type IV-Varied

(B)  Type I-Lacustrine, Type II- Marine, Type III- Terrestrial, Type IV-Varied

(C)  Type I-Lacustrine, Type II- Varied, Type III- Marine, Type IV- Terrestrial

(D)  Type I- Marine, Type II- Terrestrial, Type III- Varied, Type IV- Lacustrine

Answer: (B)

19. The number of power outages in a city in a given time interval is a Poisson random variable with a mean of 2 power outages per month. The Poisson distribution is given by, 

The probability of exactly 2 power outages in 2 months (rounded off to two decimal places) is _______.

Answer: (0.12 to 0.18)

20. Anhydrous sodium hydroxide is added to 10 litre of water to raise its pH from 7.0 to 9.0. The molar mass of sodium hydroxide is 40 g/mol. Assuming complete dissociation of sodium hydroxide and zero volume change of mixing, the amount of sodium hydroxide added (rounded off to two decimal places) is ______ mg.

Answer: (3.85 to 4.10)

21. Consider unidirectional, laminar flow of water through a homogeneous porous media as shown in the Figure. Here, H = 100 m, W = 500 m, L = 500 m, permeability of the porous media is 1012 m2, and the driving pressure drop (across length L) is 106 Use the viscosity of water as 103 Pa.s.

At steady state, the volumetric flow rate of water (rounded off to two decimal places) is given by __________ m3/s.

Answer: (0.09 to 0.12)

22. A dry gas well is producing a gas stream of the following molar composition: 95% methane and 5% carbon dioxide. The molar mass of methane is 16 g/mol and that of carbon dioxide is 44 g/mol. Assuming ideal gas behavior, gas constant R = 8.31 J mol1 K1, the gas stream density at 107 Pa and 350 K (rounded off to one decimal place) is _______ kg/m3.

Answer: (59.00 to 61.00)

23. Consider fluid flow through the annular space between two cylindrical tubes. The outer diameter of the inner tube is 40 mm and the inner diameter of the outer tube is 50 mm. The hydraulic mean diameter for fluid flow calculations (rounded off to one decimal place) is _______ mm.

Answer: (9.0 to 10.5)

24. A build up test was performed on a well after 1000 hours of oil production. During the shut-in period, the Horner’s approximation is valid which results in the following equation relating the shut-in well pressure (P­ws) to the shut-in time:

Here, k is the reservoir permeability, h is the reservoir thickness, Pi is the initial reservoir pressure, q is the flow rate during production, μ is the oil viscosity, tD is the dimensionless production time, PD(tD) is the dimensionless pressure at tD, γ is a constant, and X is dependent on the shut-in time and the production time.

The value of X after 5 hours of the shut-in (rounded off to one decimal place) is _______.

Answer: (200.5 to 201.5)

25. The initial oil production from on offshore well is 1000 STB/day, which decreased to 960 STB/day in 30 days. Using the ‘exponential decline model’, the daily production rate after 360 days from the start (rounded off to one decimal place) will be ________ STB/day.

Answer: (610.0 to 615.0)

Q26 – Q55 carry two marks each.

26. An incompressible fluid flows through a network of pipes as shown in the given Figure. The total pressure drop across points a and b is 2 kPa. The flow rates (in m3/s)in sections 1, 2, and 3 are q1, q2 and q3 The pressure drops (in kPa) are 4q1, 3q2, and 2q3 across sections 1, 2, and 3 respectively.

For a steady-state flow operation, the system of equations for flow rates is given by,

The correct option for the numeric value of X is

(A)  −0.50

(B)  −1.75

(C)  −1.00

(D)  −2.00

Answer: (B)

27. Match the following for Enhanced Oil Recovery operations

(A)  P-II, Q-I, R-III, S-IV

(B)  P-III, Q-I, R-IV, S-II

(C)  P-III, Q-II, R-IV, S-I

(D)  P-III, Q-I, R-II, S-IV

Answer: (B)

28. An compressible fluid is flowing through a tube of radius, R and length, L. The shear rate dependence of the fluid viscosity is given by the power law, is the scalar shear rate, k is a constant, and n is the flow behavior index. Assuming the flow to be steady, laminar, and fully developed, the velocity profile inside the tube for a pressure drop of ∆p applied across the tube is

Answer: (A)

29. The apparent permeability of a core measured using air is Ka, and its absolute permeability measured using an incompressible liquid is KL. If, Pm is the mean air pressure in the core during permeability measurement, and c is a positive constant linked to the pore geometry, then Ka and KL are related as

Answer: (C)

30. The plot of volume (V) versus pressure (P) for two reservoir fluids (I and II) obtained in a constant composition expansion (CCE) is shown in the Figure, Here, Vsat is saturation volume and Psat is saturation pressure. The measurements were carried out at constant temperature (the measured reservoir temperature) throughout the experiment. Which one of the following statements for the type of reservoir is correct ?

(A)  I is a gas condensate reservoir and II is an oil reservoir.

(B)  I is an oil reservoir and II is a gas condensate reservoir,

(C)  I is a light oil reservoir and II is heavy oil reservoir.

(D)  I is a dry gas reservoir and II is gas condensate reservoir.

Answer: (B)

31. The following primary and secondary porosity types are prevalent in the subsurface formations:

(1) Interparticle

(2) Intraparticle

(3) Fracture

(4) Solution

(5) Bedding plane voids

(6) Channel

Which one of the following options represents the correct combination?

(A)  Primary (1, 2, 3); Secondary (4, 5, 6)

(B)  Primary (1, 2, 5); Secondary (3, 4, 6)

(C)  Primary (1, 3, 6); Secondary (2, 4,5)

(D)  Primary (2, 4, 6); Secondary (1, 3, 5)

Answer: (B)

32. In the given Figure, which one of the following options represents the correct combination of drainage and imbibitions processes for a water wet rock in the subsurface, as indicated by number 1 to 4?

(A)  1-oil displacing water, 2-spontaneous brine imbibitions, 3-water displacing oil, 4-spontaneous oil imbibition

(B)  1- water displacing oil, 2- spontaneous oil imbibition, 3- spontaneous brine imbibition, 4- oil displacing water

(C)  1- spontaneous oil imbibition, 2- spontaneous brine imbibition, 3- water displacing oil, 4- oil displacing water

(D)  1- water displacing oil, 2- spontaneous brine imbibition, 3- oil displacing water, 4- spontaneous oil imbibition

Answer: (A)

33. The following notations are defined for a porous medium:

φ = porosity,

Sp = surface area of the pore per unit bulk volume of the core,

τ = tortuosity factor for the interconnected porous channel,

C = geometric factor of the pore.

The correct combination for the hydraulic radius (rh) and absolute permeability (k) of a porous medium is

Answer: (C)

34. Match the following

(A)  I-P, II-Q, III-R

(B)  I-R, II-Q, III-P

(C)  I-Q, II-P, III-R

(D)  I-R, II-P, III-Q

Answer: (D)

35. Select the correct combination of a floating vessel motion in a horizontal plane (P) and a vertical plane (Q)

(A)  P: (Surge, Sway, Yaw) and Q: (Heave, Roll, Pitch)

(B)  P: (Heave, Roll, Pitch) and Q: (Surge, Sway, Yaw)

(C)  P: (Surge, Roll, Pitch) and Q: (Heave, Sway, Yaw)

(D)  P: (Surge, Sway, Pitch) and Q: (Heave, Roll, Yaw)

Answer: (A)

36. The equation x3 – 3x – 5 = 0 is to be solved using the Newton-Raphson method. Starting with an initial guess of 2, the value of x after three iterations ( rounded off to three decimal places) is ______.

Answer: (2.276 to 2.281)

37. The axis of a cylinder of radius a and length L is along the z-axis with center of the flat surface at (0, 0, 0). An inextensible string of negligible thickness is wound tightly as a right-handed helix around the curved surface of the cylinder. The two ends of the string are at (a, 0, 0) and (a, 0, L).

The parametric equation of the right-handed helix is given by,

   r(θ) = [a cos θ, a sin θ, cθ],

where r, is the position vector and θ is in radian.

Given  the total length of the string (rounded off to two decimal places) is ________ cm.

Answer: (8.80 to 9.20)

38. A data set containing n ( = 10) independent measurements (xi, yi) is to be fitted by a simple linear regression model. The last square estimates of regression coefficients are obtained and the regression estimate is given by 

where, Cov(x, y) is the sample covariance and Var(x) is the sample variance.

The values are given below:

For the given data set, the unbiased variance for the error   (round off to two decimal places) is ________.

Answer: (0.16 to 0.20)

39. A porous medium of 10 cm length is made of three horizontal, cylindrical capillaries of inside diameters 2 μm, 4 μm, and 6 μm as shown in the Figure (not to scale).

Oil is being injected in this porous medium that was initially filled completely with water. The interfacial tension between oil and water is 0.025 N/m. Consider water as the completely wetting phase, i.e., contact angle is 0°. When the pressure drop across the porous medium is 20 kPa, the maximum saturation of oil in the porous medium is 0.643.

When the pressure drop is increased to 30 kPa, the maximum oil saturation (rounded off to two decimal places) will be ________ (in fraction).

Answer: (0.90 to 0.95)

40. A unidirectional, immiscible displacement of an oil is carried out with water in a cylindrical reservoir core sample (Buckley-Leverett theory is applicable). The connate water saturation is 0.25. A fractional flow of water (fw) vs. water saturation (Sw) curve is drawn for the process. A line drawn from a point (Sw = 0.25, fw = 0) on the fractional flow curve is tangent at the point (Sw = 0.8, fw = 0.8) on the curve.

The average water saturation  in the core at the time of breakthrough (rounded off to two decimal places) is _______ (in fraction).

Answer: (0.90 to 0.96)

41. In a hydrate reservoir, the porosity of the porous medium is 0.3 and the solid hydrate saturation is 0.5. Assume that the permeability (in mD) in a porous medium is given by  where, ϕe is the effective porosity available for the fluids. The permeability of the hydrate bearing porous medium (rounded off to two decimal places) is ______ mD.

Answer: (23.00 to 28.00)

42. The slip velocity for a gas-liquid flow in a vertical production well is 0.1 m/s. The superficial velocity of each of the phases is 0.1 m/s. The fractional hold-up of the gas phase (rounded off to two decimal places) is ______.

Answer: (0.36 to 0.40)

43. A three stage reciprocating compressor is to compress 4 mol/s of methane from 1 bar absolute to 60 bar absolute pressure. The gas temperature is 303 K at the suction. The compression ratio in each stage is equal and the compression is isentropic. The gas behaves as an ideal gas and the ratio of specific heat capacities (Cp/Cv) is 1.4. Take gas constant, R = 8.31 J mol1 K1.

The minimum work rate of compression required for the gas (rounded off to two decimal places) is _______ kJ/s.

Answer: (48.00 to 52.00)

44. In a 1-1 counter flow shell and tube heat exchanger, a liquid process stream (Cp = 2.1 kJ kg1 K1) is cooled from 430 K to 330 K using water (Cp = 4.2 kJ kg1 K1) having an inlet temperature of 280 K. The process stream flows on the shell side at a rate of 1 kg/s and the water on the tube side at a rate of 2.5 kg/s. The overall heat transfer coefficient is 600 W m2 K1. Neglecting the heat loss to the surroundings, the required heat transfer area (rounded off to two decimal places) is ______ m2.

Answer: (3.80 to 4.50)

45. A pre-flush of 15 wt% HCl solution (density = 1070 kg/m3) is used to dissolve dolomite in a sandstone reservoir. The molecular formula, molar mass, and density of dolomite are CaMG(CO3)2, 184.3 g/mol, and 2840 kg/m3, respectively. The molar mass of HCl is 36.5 g/mol.

If the pre-flush has to remove all the dolomite, the volumetric dissolving power of the pre-flush (rounded off to three decimal places) is _______ (m3 of dolomite/m3 of 15 wt% HCl solution).

Answer: (0.068 to 0.074)

46. A cuboidal wooden block of density 750 kg/m3, with horizontal dimensions of 2.0 m ×0 m and vertical height of 0.8 m, floats in water (density = 1000 kg/m3). The acceleration due to gravity is 9.81 m/s2. The distance between center of gravity and metacenter of the block (rounded off to two decimal places) is _______ m.

Answer: (0.03 to 0.05)

47. It is desired to determine the radius of investigation (rinv) of a low-permeability and low-pressure gas reservoir which produces under a constant flow rate. Use the following data:

Absolute permeability (k) = 0.01 mD,

Porosity (φ) = 0.05,

Total isothermal compressibility (Ct) = 200 × 106 psia1, and

Viscosity (μ) = 0.05 cP.

Assuming transient flow conditions are valid, the radius of investigation (rinv) after 200 hours of gas production (rounded off to one decimal place) is ______ ft.

Answer: (55.0 to 67.0)

48. Well stimulation is carried out in a homogeneous formation. The well is stimulated up to a radial of 54 inch from the surface of the wellbore. The diameter of the wellbore is 12 inch. The permeability enhancement in the stimulated region is found to be 10 times that of the unstimulated region. Assuming steady-state radial flow, the skin factor after stimulation (rounded off to two decimal places) is _______.

Answer: (-2.20 to -1.95)

49. A gas reservoir has a permeability of 1.0 mD, which is to be fractured hydraulically to create a 600 m long and 0.30 cm wide fracture of 2 × 105 mD permeability around the center of damage area. The fracture conductivity for the well (rounded off to two decimal places) is _______.

Answer: (1.80 to 2.20)

50. A producing oil well with the drainage to wellbore radius ratio of 2981 is found to have a skin factor of 8. Assume steady state operation and negligible pressure drop in the tubing.

The ratio of production rate of the ‘damaged’ to the ‘undamabged’ well (rounded off to two decimal places) is _______.

Answer: (0.45 to 0.55)

51. It is desired to prepare a Class H cement slurry having a density of 2100 kg/m3 using hematite as an additive. The water requirement for the Class H cement is 20 litre/50 kg cement and that for hematite is 3 litre/1000 kg hematite.

Given:

Density of class H cement = 3125 kg/m3

Density of hematite = 5000 kg/m3

Density of water = 1000 kg/m3

Weight of one sack of cement = 50.0 kg

Assuming zero volume change of mixing, the amount of hematite that should be blended with one sack of cement (rounded off to two decimal places) is _______ kg.

Answer: (9.00 to 10.50)

52. A gas reservoir without aquifer is at 300 bar (absolute) and 90° The GIIP (gas initial in place) is 107 m3 (at surface conditions). Neglect formation and water compressibility.

Given:

Surface pressure = 1 bar (absolute)

Surface temperature = 25°C

Gas compressibility factor, Z (at surface condition) = 1

Z (at 300 bar (absolute) and 90°C) = 0.88

Z (at 100 bar (absolute) and 90°C) = 0.83

If the reservoir pressure reduces to 100 bar (absolute) under isothermal conditions, the total volume of gas (at surface conditions) produced from the reservoir (rounded off to two decimal places) is _______ × 106 m3.

Answer: (6.00 to 7.00)

53. Given the following data of a shale gas formation:

WTOC (weight fraction of total organic carbon (TOC)) = 0.10

SwT (total water saturation) = 0.25

ρTOC (density of TOC) = 1.10 g/cm3

ρm(density of matrix) = 2.65 g/cm3

ρg (density of gas) = 0.35 g/cm3

ρw (density of water) = 1.00 g/cm3

ρb (formation bulk density) = 2.00 g/cm3

Consider that only water and gas are present in the formation and the following equations apply,

                                                   

where, ρf is the fluid density, φT is the total porosity, and VTOC is the volume fraction of TOC.

The volume fraction of TOC (rounded of to two decimal places) is _______.

Answer: (0.15 to 0.20)

54. It is desired to drill a deviated well with ‘build and hold type’ trajectory. The kickoff point is at a vertical depth of 1500 ft from the surface and the rate of build is 2°/100 ft. At a true vertical depth (TVD) of 7500 ft, the net horizontal departure to the target is 2500 ft. The total measured depth is ______ ft.

Answer: (8000 to 8050)

55. A cylindrical core sample of 4 inch diameter and 20 inch length is obtained from a consolidated reservoir sand. At the reservoir temperature, the formation water resistivity (Rw) is 0.15 ohm-m whereas, the resistance of the core, which is 100% saturated with brine, is 100 ohm. Use the generalized form of the Archie’s formula relating Formation Resistivity Factor (FR) and the porosity (φ). Assume, a(tortuosity factor) = 1, and m (cementation factor) = 2.

The porosity (in fraction) of the core (rounded off to two decimal places) is _______.

Answer: (0.28 to 0.34)

GATE Exam 2020 Metallurgical Engineering (MT) Question Paper With Answer Key

GATE-2020

MT-Metallurgical Engineering

GA – General Aptitude

Q1 – Q5 carry one mark each.

1. He is known for his unscrupulous ways. He always sheds______ tears to deceive people.

(A)  fox’s

(B)  crocodile’s

(C)  crocodile

(D)  fox

Answer: (C)

2. Jofra Archer, the England fast bowler, is ____ than accurate.

(A)  more fast

(B)  faster

(C)  less fast

(D)  more faster

Answer: (A)

3. Select the word that fits the analogy:

Build : Building :: Grow : ______

(A)  Grown

(B)  Grew

(C)  Growth

(D)  Growed

Answer: (C)

4. I do not think you know the case well enough to have opinions. Having said that, I agree with your other point.

What does the phrase “having said that” mean in the given text?

(A)  as opposed to what I have said

(B)  despite what I have said

(C)  in addition to what I have said

(D)  contrary to what I have said

Answer: (B)

5. Define [x] as the greatest integer less than or equal to x, for each x ∈ (−∞, ∞). If y = [x], then area under y for x ∈ [1, 4] is _______.

(A)  1

(B)  3

(C)  4

(D)  6

Answer: (D)

Q6 – Q10 carry two marks each.

6. Crowd funding deals with mobilization of funds for a project from a large number of people, who would be willing to invest smaller amounts through web-based platforms in the project.

Based on the above paragraph, which of the following is correct about crowd funding?

(A)  Funds raised through unwilling contributions on web-based platforms.

(B)  Funds raised through large contributions on web-based platforms.

(C)  Funds raised through coerced contributions on web-based platforms.

(D)  Funds raised through voluntary contributions on web-based platforms.

Answer: (D)

7. P, Q, R and S are to be uniquely coded using α and β. If P is coded as αα and Q as αβ, then R and S, respectively, can be coded as ______.

(A)  βα and αβ

(B)  ββ and αα

(C)  αβ and ββ

(D)  βα and ββ

Answer: (D)

8. The sum of the first n terms in the sequence 8, 88, 888, 8888, … is ______.

(A)   

(B)   

(C)   

(D)   

Answer: (D)

9. Select the graph that schematically represents BOTH y = xm and y = x1/m properly in the interval 0 ≤ x ≤ 1, for integer values of m, where m > 1.

Answer: (A)

10. The bar graph shows the data of the students who appeared and passed in an examination for four schools P, Q, R and S. The average of success rates (in percentage) of these four schools is ________.

(A)  58.5%

(B)  58.8%

(C)  59.0%

(D)  59.3%

Answer: (C)

MT: Metallurgical Engineering

Q1 – Q25 carry one mark each.

1. The general solution to the following homogeneous ODE,  is  The values of λ1 and λ2 are:

(A)  −1 and −3

(B)  −3 and −3

(C)  1 and −3

(D)  1 and 3

Answer: (A)

2. The number of independent elastic constants of an isotropic material is:

(A)  1

(B)  2

(C)  3

(D)  4

Answer: (B)

3. A slip system consist of a slip plane and a slip direction. Which one of the following is NOT a valid slip system in a FCC copper crystal?

Answer: (B)

4. A dielectric material is:

(A)  Electrical conductor

(B)  Metallic magnet

(C)  Two coupled electrical conductors

(D)  Electrical insulator

Answer: (D)

5. Which one of the following processes is an example of an electrolytic cell?

(A)  Corrosion of a metal rod in ambient atmosphere

(B)  Charging of a rechargeable battery

(C)  Discharging of a rechargeable battery

(D)  Sacrificial cathodic protection system

Answer: (B)

6. Which one of the following statements regarding selective leaching of a binary alloy is TRUE?

(A)  The lower atomic weight element is leached.

(B)  The element having higher diffusivity is leached.

(C)  The more electronegative element is leached.

(D)  The element with lower density is leached.

Answer: (C)

7. In green sand casting, which one of the following is NOT a part of the gating system?

(A)  Runner

(B)  Sprue

(C)  Riser

(D)  Pouring basin

Answer: (MTA)

8. For a material to exhibit superplasticity, one of the requirements is:

(A)  Coarse-grained microstructure

(B)  High strain-rate sensitivity

(C)  Low strain-hardening exponent

(D)  High modulus of elasticity

Answer: (B)

9. The dye penetrant test for detecting flaws is based on:

(A)  Magnetism

(B)  Sound propagation

(C)  X-ray absorption

(D)  Capillary action

Answer: (D)

10. When 1 mole of C3H8 at 300 K is burnt with stoichiometric amount of oxygen at 300 K to form CO2 and H2O, the adiabatic flame temperature is 5975 K. If C3H8 is burnt under the same conditions but with excess oxygen, the adiabatic flame temperature will be

(A)  equal to 5975 K irrespective of the amount of excess oxygen.

(B)  higher than 5975 K irrespective of the amount of excess oxygen.

(C)  lower than 5975 K irrespective of the amount of excess oxygen.

(D)  higher or lower than 5975 K depending on the amount of excess oxygen.

Answer: (C)

11. Two solid spheres X and Y of identical diameter are made of different materials having thermal diffusivities 100 × 106 m2s1 and 25 × 106 m2s1 Both spheres are heated in a furnace maintained at 1000 K. If the center of the sphere X reaches 800 K in 1 hour, time required for the center of sphere Y to reach 800 K is

(A)  1 hour.

(B)  2 hours.

(C)  4 hours.

(D)  16 hours.

Answer: (C)

12. Select the correct spectra (shown on a log-log scale in the figures) for emission from a gray surface and a black body, both maintained at 1000 K.

Answer: (D)

13. Given the three vectors X = −i – j + k, Y = −i + 2j + k and Z = i + k, which one of the following statements is TRUE?

(A)  X, Y and Z are mutually perpendicular.

(B)  X, Y and Z are coplanar.

(C)  X makes an angle of 30° with the normal to the plane containing Y and Z.

(D)  Z makes an angle of 60° with the normal to the plane containing X and Y.

Answer: (A)

14. Angle between two neighboring tetrahedral bonds is Si having a diamond cubic structure is:

(A)  102.5°

(B)  109.5°

(C)  120°

(D)  135.5°

Answer: (B)

15. The sequence of precipitation during aging of Al – 4 wt.% Cu alloy is:

(A)  GP zone → θ” → θ’ → θ

(B)  GP zone → θ → θ’ → θ”

(C)  GP zone → θ’ → θ” → θ

(D)  θ” → θ’ → GP zone → θ

Answer: (A)

16. The indenter used in Rockwell hardness measurements on C scale is

(A)  diamond cone.

(B)  10 mm steel ball.

(C)  diamond pyramid.

(D)  1/16-in. steel ball

Answer: (A)

17. For the function y = ax, the derivative  at x = 1 is:

(A)  1

(B)  a

(C)  a2

(D)  a ln a

Answer: (D)

18. Cupola is a furnace used to produce

(A)  cast irons.

(B)  plain carbon steels.

(C)  copper alloys.

(D)  aluminium alloys.

Answer: (A)

19. The function y = ex and y = ex intersect at the point:

(A)  (1, 3)

(B)  (−2, 2)

(C)  (0, 1)

(D)  (−1, −1)

Answer: (C)

20. A heavily cold-worked metal will

(A)  yield a coarser recrystalized grain size.

(B)  possess a lower driving force for recrystallization.

(C)  have a higher energy barrier for nucleation of recrystallized grains.

(D)  recrystallize at lower temperatures.

Answer: (D)

21. For the function f(x) given in the figure, the value of  is _______ (round off to one decimal place).

Answer: (0.5 to 0.5)

22. A component subjected to tensile stress in a mechanical device is monitored periodically for cracks by NDT. The NDT technique can only detect cracks (both surface and internal) which are larger than 1 mm. Keeping a 10% margin of safety, the maximum allowed tensile stress on the component will be ______ MPa (round off to the nearest integer).

Given, fracture toughness KIC = 30 MPa m1/2 and assume crack geometry factor of unity.

Answer: (480 to 494)

23. An iron plate with a total exposed surface area of 50 cm2 undergoes atmospheric corrosion. If 200 g of weight is lost over a period of 10 years, then the corrosion rate is _______ kg.m2.year1 (round off to the nearest integer).

Answer: (4 to 4)

24. In cold-rolling, for the sheet to be drawn into rolls, the angle of contact (or angle of bite) should be less than or equal to _______ degree (round off to one decimal place).

Given, the coefficient of friction between sheet and roll is 0.1

Answer: (5.6 to 5.8)

25. The number of atoms per unit area in (100) plane of Pb is _______ nm2 (round off to the nearest integer).

Given, crystal structure and atomic radius of Pb are FCC and 0.175 nm respectively.

Answer: (7 to 9)

Q26 – Q55 carry two marks each.

26. In the edge dislocation configuration given in the figure, dislocations X and Y are fixed and separated by a distance 2h on the same slip plane. Dislocation Z is free to glide on a parallel slip plane. The two slip planes are separated by distance h. Which one of the following statements is TRUE regarding the stability of dislocation Z at positions 1, 2 and 3?

(A)  Position 1: unstable equilibrium; Position 2: unstable; Position 3: unstable

(B)  Position 1: stable equilibrium; Position 2: unstable; Position 3: unstable

(C)  Position 1: unstable equilibrium; Position 2: stable; Position 3: unstable

(D)  Position 1: stable equilibrium; Position 2: unstable; Position 3: stable

Answer: (A)

27. Which one of the following dislocation reactions is NOT feasible in a FCC crystal?

Answer: (B)

28. A galvanic cell is formed by connecting  and  wires immersed in their respective ion solutions. The cell discharges spontaneously with a voltage of 0.5 V. The ratio of the concentration of [Fe2+] to [Zn2+] ions in the cell is of the order of:

Given, R = 8.314 J.mol1.K1, F = 96500 C.mol1, T = 298 K

(A)  106

(B)  105

(C)  106

(D)  107

Answer: (C)

29. The divergence of the vector field (x3 + y3)i + 3xy2j + 3zy2k is;

(A)  3y2 + 6xy + 6x2

(B)  3x2 + 6y2 + 9xy + 6yz

(C)  12xyz

(D)  3(x + y)2

Answer: (D)

30. Match the products in Column I with the manufacturing processes in Column II.

(A)  P-1, Q-2, R-3, S-4

(B)  P-2, Q-3, R-1, S-4

(C)  P-4, Q-1, R-2, S-3

(D)  P-4, Q-2, R-1, S-3

Answer: (D)

31. f(x) = x ln(x) + (1 – x) ln(1 – x) + 3x(1 – x) has _______ at x = 0.5

(A)  a local minimum

(B)  a local maximum

(C)  a point of inflection

(D)  a non-zero slope

Answer: (B)

32. Match the process in Column I with the most appropriate mechanisms in Column II.

(A)  P-1, Q-, R-2, S-3

(B)  P-3, Q-1, R-2, S-4

(C)  P-3, Q-4, R-2, S-1

(D)  P-1, Q-2, R-3, S-4

Answer: (C)

33. Match the reactors in column I with the corresponding products in Column II.

(A)  P-1, Q-3, R-2, S-4

(B)  P-3, Q-4, R-2, S-1

(C)  P-3, Q-1, R-2, S-4

(D)  P-3, Q-1, R-4, S-2

Answer: (C)

34. X-ray diffraction pattern from an elemental metal with a FCC crystal structure shows the first peak at a Bragg angle θ = 24.65°. The lattice parameter of this metal is _____nm.

Given, wavelength of the X-ray used is 0.1543 nm.

(A)  0.185

(B)  0.262

(C)  0.320

(D)  0.370

Answer: (C)

35. Match the materials in Column I with their common applications in Column II.

(A)  P-1, Q-3, R-2, S-4

(B)  P-4, Q-2, R-1, S-3

(C)  P-2, Q-1, R-4, S-3

(D)  P-2, Q-3, R-1, S-4

Answer: (D)

36. The Mg-Sn phase diagram exhibits two eutectics on either side of the high melting intermetallic line compound, Mg2Sn, as given below.

At 561°C: L (36.9 wt.% Sn) → α (14.48 wt.% Sn) + Mg2Sn

At 203°C: L(97.87 wt.% Sn) → β-Sn (almost 100 wt. % Sn) + Mg2Sn

After the eutectic reaction has gone to completion and equilibrium has been attained at a temperature just below 561°C, the amount of eutectic constituent present in the alloy, Mg-50 wt.% Sn, is approximately_______(in wt.%).

Given, atomic weight of Sn is 118.7 and Mg is 24.3

(A)  25

(B)  38

(C)  62

(D)  75

Answer: (C)

37. Determine the correctness or otherwise of the following Assertion [a] and the Reason [r]

Assertion [a]: Low-alloy steels used for medium-temperature creep resistance often have additions of strong carbide-forming elements.

Reason [r]: During creep deformation, the particles with higher misfit with the matrix, lose coherency.

(A)  Both [a] and [r] are true and [r] is the correct reason for [a].

(B)  Both [a] and [r] are true but [r] is not the correct reason for [a].

(C)  Both [a] and [r] are false.

(D)  [a] is true but [r] is false.

Answer: (B)

38. Determine the correctness or otherwise of the following Assertion [a] and the Reason [r]

Assertion [a]: The rate of homogenization in a dilute substitutional solid solution of B in A is controlled by the diffusivity of B.

Reason [r]: Atomic migration cannot occur along dislocations and grain boundaries.

(A)  Both [a] and [r] are true and [r] is the correct reason for [a]

(B)  Both [a] and [r] are true but [r] is not the correct reason for [a]

(C)  Both [a] and [r] are false

(D)  [a] is true but [r] is false

Answer: (D)

39. Match the elements in Column I with their electronic behaviour given in Column II.

(A)  P-1, Q-2, R-3, S-4

(B)  P-3, Q-4, R-1, S-2

(C)  P-4, Q-1, R-2, S-3

(D)  P-4, Q-3, R-1, S-2

Answer: (C)

40. Radius of the largest interstitial atom that can be accommodated in an octahedral void in BCC iron without distorting the lattice is _______nm (round off to three decimal places).

Assume hard sphere model and radius of Fe atom as 0.124 nm.

Answer: (0.018 to 0.020)

41. The production process of cylindrical pipes results in a statistical scatter in their diameter which is modeled by a normal distribution with a mean value of 10 mm. If the area under the normal curve between 9 mm and 10 mm is 0.35, then the probability of producing pipes of diameter greater than 11 mm is __________ (round off to two decimal

Answer: (0.14 to 0.16 or 14 to 16)

42. The solution (using trapezoidal rule) of the integral  by dividing the range 0 to 1 into two equal intervals is _______ (round off to two decimal places).

Answer: (0.71 to 0.75)

43. Iron is corroding in fresh water which has dissolved oxygen concentration of 15 mM. The anodic current density at an overpotential of 120 mV is _____ A.cm2 (round off to three decimal places).

Given:

(1) Anodic Tafel slope is 0.06 V.

(2) Diffusion coefficient of oxygen is 2.42 × 105 cm2.s1.

(3) Diffusion layer thickness is 0.06 cm.

Answer: (0.190 to 0.238)

44. A metal oxidizes at 1200 K with a parabolic rate constant of 3 × 106 g2.cm4.s1. Time taken for the oxide film to grow to thickness of 2 μm is ________ s (round off to two decimal places).

Given, density of oxide is 6.5 g.cm3.

Answer: (0.54 to 0.58)

45. Two plates of composition, Fe−10 wt.% Ni and Fe−20 wt.% Ni are fusion-welded using a filler rod of composition 20 wt.% Ni-80 wt.% Cr. Contribution to dilution of the weld pool is 20% from each plate. The Ni content in the weld pool is ______wt.% (round off to the nearest integer).

Answer: (15 to 15)

46. Figure shows schematic of a venturimeter. The cross sectional area is 100 mm2 at A and is 50 mm2 at B. If air is flowing through the venturimeter at a flow rate of 103m3.s1, the height H in the air-over-water manometer is ________ mm (round off to the nearest integer).

Assume.

(1) Incompressible flow with no friction losses.

(2) Density of air is 1 kg m3.

(3) Density of water is 1000 kg m3.

(4) Acceleration due to gravity is 9.8 ms2.

Answer: (14 to 16)

47. For effective communition in a ball mill, it is desired that the balls travelling along the mill wall leave the wall at point C and travel freely in air along the path CDA, as shown in the figure. If ∠BOC is 120°, the rotational speed of the mill is ________ rpm (rounded off to one decimal place) by performing suitable force balance at point C.

Assume:

(1) There is no slip between the ball and mill wall.

(2) O is the rotational axis of the mill and OB is parallel to the vector g.

(3) Inner diameter of ball mill is 3.26 m.

(4) Acceleration due to gravity g is 9.8 m s2.

Answer: (15.6 to 17.6)

48. If liquid copper is cooled to 1353 K, magnitude of the driving force for liquid to transform to solid is _______ J. mol1 (round off to one decimal place).

Given, melting temperature and enthalpy of melting of copper are 1356 K and 13 kJ.mol1 resepctively.

Answer: (28.6 to 29.0)

49. 1000 kg of liquid steel containing 0.03 wt.% S needs to be desulphurized using a slag to bring the sulphur content down to 0.015 wt.%. The quantity of slag needed is _______ kg(round off to the nearest integer).

Assume:

(1) Thermodynamic equilibrium

(2) No sulphur in the slag prior to desulphurization treatment

Given the equilibrium sulphur partition ratio between slag and steel,   is 50.

Answer: (19 to 21)

50. Zone refining of Si results in residual P content of 0.1 parts per billion by weight. The electrical conductivity of this zone refined Si is ________ Ω1 .m1 (round off to two decimal places.)

Given:

(1) Avogadro number is 6.02 × 1023.

(2) Density of Si is 2.33 g.cm3.

(3) Atomic weight of P is 30.97.

(4) Charge of electron is 1.6 × 1019 A.s

(5) Mobility of electron is 0.2 m2.V1.s1

Answer: (0.14 to 0.16)

51. The steady state creep rate of a material increases by a factor of 20 when the temperature is increased from 890 K to 980 K. The creep rate at a temperature of ________ K (round off to the nearest integer) will be 5 times the creep rate at 890 K.

Answer: (933 to 939)

52. Crack growth is being continuously measured in test specimen subjected to constant amplitude cyclic stress with a mean stress of zero. The crack growth rate is related to the stress intensity range, ∆K as  where, a is the crack length and N is the number of cycles. When the crack length increases by a factor of two, the crack growth rate will increase by a factor of _______ (round off to one decimal place).

Answer: (2.6 to 3.0)

53. In a top gated mold, liquid metal enters the mold cavity as a freely falling steam under gravity from a height of 0.5 m. Ignore the fluid friction due to viscosity and the drag due to changes in the direction of flow. If the volume of the mold cavity is 10 m3, then the time required to fill the mold is ________ s (round off to nearest integer).

Given:

(1) Acceleration due to  gravity is 9.8 m.s2.

(2) Cross-sectional area of gate is 0.2 m2.

Answer: (14 to 18)

54. A Basic Oxygen Furnace operator, at the end of oxygen blow, measures the dissolved oxygen content in the steel as 0.03 wt.% and the steel temperature as 1800 K. The carbon content [C] in the steel is ________ wt.% (round off to two decimal places).

Assume:

(1) Equilibrium between dissolved carbon [C], dissolved oxygen [O], and CO (gas) at 1 atmosphere.

(2) Henry’s law is valid for both [C] and [O]

Given:

Answer: (0.06 to 0.08)

55. M and N are 3 × 3 matrices. If the det(M) is −9 and the det(N) is −14, then the det(NM) is _______ (round off to the nearest integer).

Answer: (126 to 126)

GATE Exam 2020 Mining Engineering (MN) Question Paper With Answer Key

GATE-2020

MN-Mining Engineering

GA-General Aptitude

Q1-Q5 carry one mark each.

1. Rajiv Gandhi Khel Ratna Award was conferred _____Mary Kom, a six-time world champion in boxing, recently in a ceremony _____ the Rashtrapati Bhawan (the President’s official residence) in New Delhi.

(A)  with, at

(B)  on, in

(C)  on, at

(D)  to, at

Answer: (C)

2. Despite a string of a poor performances, the changes of K. L. Rahul’s selection in the team are ______.

(A)  slim

(B)  bright

(C)  obvious

(D)  uncertain

Answer: (B)

3. Select the word that fits the analogy:

Cover : Uncover :: Associate : _______

(A)  Unassociate

(B)  Inassociate

(C)  Missassociate

(D)  Dissociate

Answer: (D)

4. Hig by floods, he kharif (summer sown) crops in various parts of the county have been affected. Officials believe that the loss in production of the kharif crops can be recovered in the output of the rabi (winter sown) crops so that the country can achieve its food-grain production target of 291 million tons in the crop year 2019-20 (July-June). They are hopeful that good rains in July-August will help the soil retain moisture for a longer period, helping winter sown crops such as wheat and pulses during the November-February period.

Which of the following statements can be inferred from the given passage?

(A)  Officials declared that the food-grain production target will be met due to good rains.

(B)  Officials want the food-grain production target to be met by the November-February period.

(C)  Officials feel that the food-grain production target cannot be met due to floods.

(D)  Officials hope that the food-grain production target will be met due to a good rabi produce.

Answer: (D)

5. The difference between the sum of the first 2n natural numbers and the sum of the first n odd natural numbers is ______.

(A)  n2 – n

(B)  n2 + n

(C)  2n2 – n

(D)  2n2 + n

Answer: (B)

Q6 – Q10 carry two marks each.

6. Repo rate is the at which Reserve Bank of India (RBI) lends commercial banks, and reverse repo rate is the rate at which RBI borrows money from commercial banks.

Which of the following statements can be inferred from the above passage?

(A)  Decrease in repo rate will increase cost of borrowing and decrease lending by commercial banks.

(B)  Increase in repo rate will decrease cost of borrowing and increase lending by commercial banks.

(C)  Increase in repo rate will decrease cost of borrowing and decrease lending by commercial banks.

(D)  Decrease in repo rate will decrease cost of borrowing and increase lending by commercial banks.

Answer: (D)

7. P, Q, R, S, T, U, V, and W are seated around a circular table.

(I) S is seated opposite to W.

(II) U is seated at the second place to the right of R.

(III) T is seated at the third place to the left of R.

(IV) V is a neighbor of S.

Which of the following must be true?

(A)  P is a neighbor of R.

(B)  Q is a neighbor of R.

(C)  P is not seated opposite to Q.

(D)  R is the left neighbor of S.

Answer: (C)

8. The distance between Delhi and Agra is 233 km. A car P started travelling from Delhi to Agra and another car Q started from Agra to Delhi along the same road 1 hour after the car P started. The two cars crossed each other 75 minutes after the car Q started. Both cars were travelling at constant speed. The speed of car P was 10 km/hr more than the speed of car Q. How many kilometers the car Q had travelled when the cars crossed each other?

(A)  66.6

(B)  75.2

(C)  88.2

(D)  116.5

Answer: (B)

9. For a matrix M = [mij], i. j= 1, 2, 3, 4, the diagonal elements are all zero and mij = −mij. The minimum number of elements required to fully specify the matrix is_______.

(A)  0

(B)  6

(C)  12

(D)  16

Answer: (B)

10. The profit shares of two companies P and Q are shown in the figure. If the two companies have invested a fixed and equal amount every year, then the ratio of the total revenue of company P to the total revenue of company Q, during 2013-2018 is ______.

(A)  15 : 17

(B)  16 : 17

(C)  17 : 15

(D)  17 : 16

Answer: (B)

MN: Mining Engineering

Q1 – Q25 carry one mark each.

1. The eigenvalues of the matrix  are

(A)  6, 4

(B)  4, 5

(C)  −2, 5

(D)  −4, 5

Answer: (C)

2. For the electric delay detonator shown in the figure. the components P, Q and R, respectively, are

(A)  fuse head, delay element, priming charge

(B)  fuse head, priming charge, delay element

(C)  priming charge, delay element, fuse head

(D)  delay element, fuse head, priming charge

Answer: (A)

3. Match the following safety arrangement for a surface-to-underground shaft hoist with their corresponding safety functions.

(A)  P-1, Q-2, R-3

(B)  P-3, Q-1, R-2

(C)  P-2, Q-3, R-1

(D)  P-1, Q-3, R-2

Answer: (D)

4. The fore bearings and back bearings of the lines of an open compass traverse are given below.

The stations that are free from local attraction are

(A)  P and Q

(B)  Q and R

(C)  R and S

(D)  S and T

Answer: (B)

5. The plane stress condition is given by

(A)  εzz = 0, γyz = 0, γzx = 0

(B)  σzz = 0, τyz = 0, τzx = 0

(C)  σzz ≠ 0, τyz ≠ 0, τzx ≠ 0

(D)  εzz ≠ 0, γyz ≠ 0, γzx ≠ 0

Answer: (B)

6. Match the following metals with their corresponding minerals.

(A)  P-4, Q-2, R-1, S-3

(B)  P-4, Q-1, R-3, S-2

(C)  P-3, Q-4, R-1, S-2

(D)  P-2, Q-3, R-4, S-1

Answer: (C)

7. Match the wire rope types with their corresponding cross-sectional diagrams.

(A)  P-3, Q-2, R-4, S-1

(B)  P-3, Q-1, R-2, S-4

(C)  P-1, Q-3, R-4, S-2

(D)  P-4, Q-2, R-3, S-1

Answer: (A)

8. The ‘ratchet and pawl’ mechanism in a jack hammer drill

(A)  forces down the piston

(B)  provides a twisting force to the drill steel

(C)  engages rifle bar with rifle nut

(D)  prevents reverse rotation of rifle bar

Answer: (D)

9. Post-pillar method of stoping is a variant of

(A)  cut and fill stoping

(B)  sublevel stoping

(C)  vertical crater retreat method

(D)  sublevel caving

Answer: (A)

10. ‘Cross-measure borehole method’ is used for

(A)  rock slope monitoring

(B)  methane drainage

(C)  connecting tow drifts

(D)  subsidence monitoring

Answer: (B)

11. The dry and wet bulb temperatures at the inlet of the airstream are 30°C and 25°C, respectively. The corresponding values at the outlet of the airstream are 26°C and 25°C, respectively. The psychrometric process that occurs in the airstream is described as

(A)  latent cooling

(B)  sensible cooling

(C)  condensation

(D)  evaporative cooling

Answer: (D)

12. For a mixture of inflammable gases, the lower and upper explosibility limits can be computed using

(A)  Dalton’s law

(B)  Graham’s law

(C)  Le Chatelier relation

(D)  Boyle’s law

Answer: (C)

13. The code for the lowest category of mineral resources under United Nations Framework Classification (UNFC) system is

(A)  444

(B)  123

(C)  334

(D)  111

Answer: (C)

14. Match the following sampling patterns with the corresponding sampling types.

(A)  P-1, Q-3, R-2, S-4

(B)  P-1, Q-3, R-4, S-2

(C)  P-1, Q-2, R-3, S-4

(D)  P-4, Q-2, R-1, S-3

Answer: (B)

15. In the context of gas testing using flame safety lamp, the correct statement is

(A)  Each accumulation test has to be necessarily followed by percentage test

(B)  Accumulation test is always done after percentage test

(C)  Either percentage test or accumulation test can be done first

(D)  Percentage test is done only in the event of accumulation test giving negative result

Answer: (D)

16. For x in the range of [−3, 3], the maximum value for the function f(x) = x3 – 6x2 + 9x + 15 is _________ (round off to 1 decimal place).

Answer: (18.0 to 20.0)

17. A rock sample has coefficient of thermal diffusivity 1.282 × 106 m2/s specific heat 900 J/(kg°C) and density 2600 kg/m3. The coefficient of thermal conductivity of the rock sample in W/(m°C) is ________ (round off to 1 decimal place).

Answer: (2.9 to 3.1)

18. A random variable X has the following probability mass function.

The expected value of the variable X is _______ (round off to 1 decimal place).

Answer: (5.4 to 5.6)

/bg_collapse]

19. A random sample has five observations as shown below.

The coefficient of variation of the sample is ________ (round off to 3 decimal places).

Answer: (0.120 to 0.125)

20. Rock strata has unit weight of 25 kN/m3 and Poisson’s ratio 1/3. At a depth of 200 m, the horizontal stress in MPa is _______(round off to 1 decimal place).

Answer: (2.4 to 2.6)

21. A shovel of bucket capacity 4.2 m3 makes 900 passes per day with a fill factor of 0.8. If the swell factor of the rock is 1.4, then in-situ volume handled b the shovel in a month of 24 working days in m3 is ________ (round off to 1 decimal place).

Answer: (51830.0 to 51850.0)

22. A cylindrical sample of granular material has the following measurements:

Length: 10 cm

Diameter: 5 cm

Weight: 350 g

Assume the sample is completely dry with specific gravity of solid grains 2.8. The void ratio of the  sample is ________ (round off to 3 decimal places).

Answer: (0.560 to 0.580)

23. The following consecutive readings were taken at uniform intervals with a level and a leveling staff on continuously sloping ground.

0.405,     1.035,  1.654,  0.240,  0.615,  1.125,  0.800,  1.125

The number of change points is _______.

Answer: (2 to 2)

24. The cost of a slurry pump is Rs. 50,000 and it has an estimated life of 7 years. If the salvage value is Rs. 8,000, the annual depreciation following straight-line depreciation method, in Rupees, is ________(round of to 1 decimal place).

Answer: (5990.0 to 6010.0)

25. In a mine bench, the shovel loading time follows exponential distribution with a mean loading time of 5 min per dumper. The arrival rate of dumpers that are identical in capacity, follows Poisson distribution with a mean arrival rate of 8 per hour. The probability that the shovel remains idle is _______ (round off to 2 decimal places).

Answer: (0.32 to 0.34)

Q26 – Q55 two marks each.

26. For  the value of   is

Answer: (B)

27. For the differential equation  assuming the constant of integration to be C, the general solution is

Answer: (D)

28. A belt-drive used for power transmission between two parallel shafts has a belt of mass 1.2 kg/m, and the maximum allowable belt tension is 2250 N. If the centrifugal tension is one third of the maximum allowable belt tension, the speed at which maximum power is transmitted by the belt, in m/s, is

(A)  46.48

(B)  37.73

(C)  25.00

(D)  35.36

Answer: (C)

29. An air receiver of a compressor, having volume 0.5 m3, supplies air for charging ANFO in drill holes. During the charging process the absolute pressure of the air receiver falls from 900 kPa to 700 kPa. Assuming the entire process is isothermal, the volume of air supplied by the receiver at 100 kPa ambient pressure, in m3 is

(A)  1.00

(B)  0.39

(C)  0.64

(D)  4.48

Answer: (A)

30. A conveyor belt consumes 60 kW power while running at a speed of 3.0 m/s. The angle of lap is 180° and the coefficient of friction between belt and pulley is 0.2. The maximum tension (kN) in the belt is

(A)  21.7

(B)  61.7

(C)  82.9

(D)  42.9

Answer: (D)

31. A linear programming problem is stated below.

Maximize Z = 3x1 + 5x2

   subject to, 2x1 + x2 ≤ 8

6x1 + 8x2 ≤ 30

x1, x2 ≥ 0

The objective function has

(A)  infinite number of solutions

(B)  unbounded solution

(C)  unique solution

(D)  infeasible solution

Answer: (C)

32. An explosive mixture has 80 g of ammonium nitrate (NH4NO3) and 14 g of fuel oil (C10H20). The oxygen balance in the mixture is

(A)  surplus by 32 g

(B)  deficient by 24 g

(C)  surplus by 16 g

(D)  deficient by 32 g

Answer: (D)

33. A cylindrical sample of cross-sectional area A, length L, and Young’s Modulus E, is subjected to a constant uniaxial load P. Within the elastic limit of loading, the total strain energy stored is

Answer: (D)

34. A surface mine production system long with reliability of the individual components is shown below. The system reliability is ________ (round off to 3 decimal places).

Answer: (0.530 to 0.550)

35. The selling price of one rock bolt (in Rs.) is given as, S = (200 – 10n5), where n is the number of bolts produced. The manufacturing cost of rock bolts has a fixed components of Rs. 15000, and a variable components of Rs. 100 per bolt. The minimum break-even production, in number of rock bolts, is _______ (round off to 1 decimal place).

Answer: (148.0 to 154.0)

36. Two underground mines are separated by a horizontal coal barrier of 60 m thickness. One mine is inundated, whereas the other mine as a blind heading of dimensions 4.0 m wide and 2.5 m height terminating at the barrier. The overall in-situ shear strength of the coal mass is 500 kPa. Assume that the failure mode is in shear. At a factor of safety of 10, the maximum water head that can be withstood by the coal barrier, in m, is ________ (round off to 1 decimal place).

Answer: (385.0 to 410.0)

37. Air enters from ambient atmosphere into a level duct of uniform cross-section area 0.2 m2 at a flow rate of 2.0 m3/s and density of 1.2 kg/m3. The entry shock loss factor is 1.0 and the resistance of the duct per meter length is 1.0 Ns2/m8. The static pressure measured at a distance of 20 m from the duct entrance in Pa, is ______ (round off to 1 decimal place).

Answer: (-205.0 to -195.0)

38. A sealed-off area air sample consists of 16.0%O2, 3.0%CO2, and the rest is N2. Assume that the standard composition of atmospheric air is 21.0% O2 and 79.0% N2. The percentage of blackdamp in the air sample is ________ (round off to 2 decimal places).

Answer: (22.00 to 25.00)

39. A diesel loader produces 60 l/s of exhaust fumes containing 5000 ppm of CO. The incoming air has 10 ppm of CO. The minimum amount air flow in m3/s that is needed at the loader to dilute CO to a permissible level of 50 ppm is ________ (round off to 1 decimal place).

Answer: (7.2 to 7.7)

40. The equation for peak particle velocity (PPV) from blast induced ground vibration is given by

 where k and B are site constants.

In a field study, the following readings are recorded.

The value of B is _______ (round off to 3 decimal places).

Answer: (-1.255 to -1.165)

41. An underground copper mine sends 2500 tonnes of ore per day to the concentrator plant having an average grade of 1.2% Cu. The plant produces concentrate of 25.0% Cu with a recovery of 93.0%. The solids portion of tailings generated from the plant per day in tonnes is________ (round of to 1 decimal place).

Answer: (2380.0 to 2396.0)

42. A mine gallery is supported by regularly placed roof bolts of 100 kN allowable pull force per bolt, as shown below. Assuming unit weight of the immediate roof is 25 kN/m3, the factor of safety of the support system is _________ (round off to 2 decimal places).

Answer: (2.10 to 2.45)

43. The slip circle of radius 30.0 m in an overburden slope is shown with the centre of slip circle at point O. The tension crack is completely filled with water. For one meter width of the slope, the moment exerted by the force of water about O in kN-m is _______ (round off to 1 decimal place).

Answer: (3360.0 to 3480.0)

44. The function values at four points of x are shown in the table. The area under the function, using trapezoidal method, is _______ (round off to 1 decimal place).

Answer: (71.0 to 72.0)

45. A particle starting from rest moves in a straight line such that after time, t (seconds), the acceleration becomes (6 – t/4) cm/s2. When the acceleration becomes zero, the velocity of the particle in cm/s is _______ (round off to 1 decimal place).

Answer: (71.0 to 73.0)

46. A parallelepiped has edge vectors as shown below.

The volume of the parallelepiped is ______ round off to 1 decimal place).

Answer: (87.0 to 89.0)

47. For a development heading, the blasting parameters are

Cross-section: 6.0 m × 5.0 m

Total number of holes: 72

Number of trimmer holes: 30

Depth of each hole: 3.5 m

Charge per hole (except trimmers): 3.5 kg

Charge per trimmer hole: 1.8 kg

Pull per round: 90% of hole depth

The powder factor for the development round in m3/kg is _______ (round off to 2 decimal  places).

Answer: (0.44 to 0.50)

48. The following figure represents the observations from the level survey of an underground gallery.

If the reduced level of station A is 100.0 m, the reduced level of station D in m is ______ (round off to 1 decimal place).

Answer: (100.4 to 100.6)

49. A tracheometer was setup at station P and the following readings were taken at two stations A and B with the staff held vertical and the line of sight horizontal.

The additive and multiplying constants of the tracheometer are 0 and 100, respectively. The length of AB in m is ________ (round off to 1 decimal place).

Answer: (157.0 to 161.0)

50. The geometry of a simple planar curve (ADB) is shown below. The value of the mid-ordinate of the curve in m is ________ (round off to 1 decimal place).

Answer: (248.0 to 252.0)

51. A steel cube of side 50 mm is subjected to a uniform pressure of 200 MPa acting on each face. The Young’s modulus and Poisson’s ratio of the material are 200 GPa and 0.25, respectively. The decrease in the side of the cube in mm is _______ (round off to 3 decimal places).

Answer: (0.020 to 0.030)

52. In the figure shown below, the friction coefficient between the block and the inclined plane is 0.2, and all the pulleys are weightless. The weight of the block is 10 N. The minimum force P in Newtons that is needed to slide the block up the inclined plane is _______ (round off to 2 decimal places).

Answer: (67.00 to 69.00)

53. An assignment problem is given below with the cost of assignment as shown.

If only one task can be assigned to one group, the minimum cost of assignment is _______.

Answer: (18 to 18)

54. A vertical stopping bock of length 60.0 m, height 40.0 m, and average width 1.5 m has sharp boundary walls. However, the mining width of the stopping block is 2.0 m. The tonnage factors are 3.0 tonne/m3 for ore and 2.5 tonne/m3 for wall rocks. The average grade of ore is 10.0%. The overall grade of ore mined on account of dilution in percentage is _________ (round off to 2 decimal places).

Answer: (7.70 to 7.95)

55. A bauxite ore body has four boreholes as shown below. For each borehole, the grade of alumina, the thickness of the ore body, and the triangular area of influence are as shown.

The average grade of ore body in the region ABCD in percentage is ________ (round off to 1 decimal place).

Answer: (30.8 to 31.6)

GATE Exam 2020 Mechanical Engineering (ME-2) Question Paper With Answer Key

GATE-2020

ME-2: Mechanical Engineering

GA-General Aptitude

Q1 – Q5 carry one mark each.

1. While I agree _____ his proposal this time, I do not often agree ____him.

(A)  to, with

(B)  with, to

(C)  with, with

(D)  to, to

Answer: (A)

2. The recent measures to improve the output would ______ the level of production satisfaction.

(A)  increase

(B)  decrease

(C)  speed

(D)  equalise

Answer: (A)

3. Select the word that fits the analogy:

White : Whitening :: Light :______

(A)  Lightning

(B)  Lightening

(C)  Lighting

(D)  Enlightening

Answer: (B)

4. In one of the greatest innings ever seen in 142 years of Test history, Ben Stokes upped the tempo in a five-and-a-half hour long stay of 219 balls including 11 fours and 8 sixes that saw him finish on a 135 not out as England squared the five-match series.

Based on their connotations in the given passage, which one of the following meanings DOES NOT match?

(A)  upped = increased

(B)  squared = lost

(C)  tempo = enthusiasm

(D)  saw = resulted in

Answer: (B)

5. There are five levels {P, Q, R, S, T} in a linear supply chain before a product reaches customers, as shown in the figure.

At each of the five levels, the price of the product is increased by 25%. If the product is produced at level P at the cost of Rs. 120 per unit, what is the price paid (in rupees) by the customers?

(A)  187.50

(B)  234.38

(C)  292.96

(D)  366.21

Answer: (D)

Q6 – Q10 carry two marks each.

6. Climate change and resilience deal with two aspects – reduction of sources of non-renewable energy resources and reducing vulnerability of climate change aspects. The terms ‘mitigation’ and ‘adaptation’ are used to refer to these aspects, respectively.

Which of the following assertions is best supported by the above information?

(A)  Mitigation deals with consequences of climate change.

(B)  Adaptation deals with causes of climate change.

(C)  Mitigation deals with actions taken to reduce the use of fossils fuels.

(D)  Adaptation deals with actions taken to combat green-house gas emissions.

Answer: (C)

7. Find the missing element in the following figure.

(A)  d

(B)  e

(C)  w

(D)  y

Answer: (A)

8. It was estimated that 52 men can complete a strip in a newly constructed highway connecting cities P and Q in 10 days. Due to an emergency, 12 men were sent to another project. How many number of days, more than original estimate, will be required to complete the strip?

(A)  3 days

(B)  5 days

(C)  10 days

(D)  13 days

Answer: (A)

9. An engineer measures THREE quantities X, Y and Z in an experiment. She finds that they follow a relationship that is represented in the figure below: (the product of X and Y linearly varies with Z)

Then, which of the following statements is FALSE?

(A)  For fixed Z; X is proportional to Y

(B)  For fixed Y; X is proportional to Z

(C)  For fixed X; Z is proportional to Y

(D)  XY/Z is constant

Answer: (A)

10. The two pie-charts given below show the data of total students and only girls registered in different streams in a university. If the total number of students registered in the university is 5000, and the total number of the registered girls is 1500; then, the ratio of boys enrolled in Arts to the girls enrolled in Management is ______.

(A)  2 : 1

(B)  9 : 22

(C)  11 : 9

(D)  22 : 9

Answer: (D)

ME2: Mechanical Engineering

Q1 – Q25 carry one mark each.

1. The sum of two normally distributed random variables X and Y is

(A)  always normally distributed

(B)  normally distributed, only if X and Y are independent

(C)  normally distributed, only if X and Y have the standard deviation

(D)  normally distributed, only if X and Y have the same mean

Answer: (MTA)

2. A matrix P is decomposed into its symmetric part S and skew symmetric part V. If then matrix P is

Answer: (B)

3. Let  Then, I may also be expressed as

Answer: (C)

4. The solution of  which additionally satisfies  in the Laplace s-domain is

Answer: (A)

5. An attempt is made to pull a roller of weight W over a curb (step) by applying a horizontal force F as shown in the figure.

The coefficient of static friction between the roller and the ground (including the edge of the step) is μ. Identify the correct free body diagram (FBD) of the roller when the roller is just about to climb over the step.

Answer: (B)

6. A circular disk of radius r is confined to roll without slipping at P and Q as shown in the figure.

(A)  v/r

(B)  r/2r

(C)  2v/3r

(D)  3v/2r

Answer: (D)

7. The equation of motion of a spring-mass-damper system is given by 

                                                                 

The damping factor for the system is

(A)  0.25

(B)  0.5

(C)  2

(D)  3

Answer: (B)

8. The number of qualitatively distinct kinematic inversions possible for a Grashof chain with four revolute pairs is

(A)  1

(B)  2

(C)  3

(D)  4

Answer: (C)

9. The process, that uses a tapered horn to amplify and focus the mechanical energy for machining of glass, is

(A)  electrochemical machining

(B)  electrical discharge machining

(C)  ultrasonic machining

(D)  abrasive jet machining

Answer: (C)

10. Two plates, each of 6 mm thickness, are to be butt-welded. Consider the following processes and select the correct sequence in increasing order of size of the heat affected zone.

(1) Arc welding

(2) MIG welding

(3) Laser beam welding

(4) Submerged are welding

(A)  1-4-2-3

(B)  3-4-2-1

(C)  4-3-2-1

(D)  3-2-4-1

Answer: (D)

11. Which one of the following statements about a phase diagram is INCORRECT?

(A)  It indicates the temperature at which different phases start to melt

(B)  Relative amount of different phases can be found under given equilibrium conditions

(C)  It gives information on transformation rates

(D)  Solid solubility limits are depicted by it

Answer: (C)

12. The figure below shows a symbolic representation of the surface texture in a perpendicular lay orientation with indicative values (I through VI) marking the various specifications whose definitions are listed below.

P: Maximum Waviness Height (mm); Q: Maximum Roughness Height (mm);

Q: Minimum Roughness Height (mm); S: Maximum Waviness Width (mm);

T: Maximum Roughness Width (mm); U: Roughness Width Cutoff (mm).

The correct match between the specifications and the symbols (I to VI) is

(A)  I-R, II-Q, III-P, IV-S, V-U, VI-T

(B)  I-R, II-P, III-U, IV-S, V-T, VI-Q

(C)  I-U, II-S, III-Q, IV-T, V-R, VI-P

(D)  I-Q, II-U, III-R, IV-T, V-S, VI-P

Answer: (A)

13. In materials Requirement Planning, if the inventory holding cost is very high and the setup cost is zero, which one of the following lot sizing approaches should be used?

(A)  Economic Order Quantity

(B)  Lot-for-Lot

(C)  Base Stock Level

(D)  Fixed Period Quantity, for 2 periods

Answer: (B)

14. Which of the following conditions is used to determine the stable equilibrium of all partially submerged floating bodies?

(A)  Centre of buoyancy must be above the centre of gravity

(B)  Centre of buoyancy must be below the centre of gravity

(C)  Metacentre must be a higher level than the centre of gravity

(D)  Metacentre must be at a lower level than the centre of gravity

Answer: (C)

15. In the space above the mercury column in a barometer tube, the gauge pressure of the vapour is

(A)  positive, but more than one atmosphere

(B)  negative

(C)  zero

(D)  positive, but less than one atmosphere

Answer: (B)

16. A closed vessel contains pure water, in thermal equilibrium with its vapour at 25°C (Stage # 1), as shown.

The vessel in this stage is then kept inside an isothermal over which is having an atmosphere of hot air maintained at 80°C. The vessel exchanges heat with the oven atmosphere and attains a new thermal equilibrium (Stage # 2). If the Valve A is now opened inside the oven, what will happen immediately after opening the valve?

(A)  Water vapor inside the vessel will come out of the Valve A

(B)  Hot air will go inside the vessel through Valve A

(C)  Nothing will happen-the vessel will continue to remain in equilibrium

(D)  All the vapor inside the vessel will  immediately condense

Answer: (B)

17. For an air-standard Diesel cycle,

(A)  heat addition is at constant volume and heat rejection is at constant pressure

(B)  heat addition is at constant pressure and heat rejection is at constant pressure

(C)  heat addition is at constant pressure and heat rejection is at constant volume

(D)  heat addition is at constant volume and heat rejection is at constant volume

Answer: (C)

18. The values of enthalpies at t he stator inlet and rotor outlet of a hydraulic turbomachine stage are h1 and h3 The enthalpy at t he stator outlet (or, rotor inlet) is h2. The condition (h2 – h1) = (h3 – h2) indicates that the degree of reaction of this stage is

(A)  zero

(B)  50%

(C)  75%

(D)  100%

Answer: (B)

19. Let I be a 100 dimensional identity matrix and E be the set of its distinct (no value appears more than once in E) real eigenvalues. The number of elements in E is _______.

Answer: (1 to 1)

20. A beam of negligible mass is hinged at support P and has a roller support Q as shown in the figure.

A point load of 1200 N is applied at point R. The magnitude of the reaction force at support Q is ______ N.

Answer: (1500 to 1500)

21. A machine member is subjected to fluctuating stress σ = σ0 cos(8πt). The endurance limit of the material is 350 MPa. If the factor of safety used in the design is 3.5 then the maximum allowable value of σ0 is ________MPa (round off to 2 decimal places).

Answer: (99.98 to 100.02)

22. A bolt head has to be made at the end of a rod of diameter d = 12 mm by localized forging (upsetting) operation. The length of the unsupported portion of the rod is 40 mm. To avoid buckling of the rod, a closed forging operation has to be performed with a maximum die diameter of _____ mm.

Answer: (18 to 19.5)

23. Consider the following network of activities, with each activity named A-L, illustrated in the nodes of the network.

The number of hours required for each activity is shown alongside the nodes. The slack on the activity L, is _______ hours.

Answer: (2 to 2)

24. In a furnace, the inner and outer sides of the brick wall (k1 = 2.5 W/m.K) are maintained at 1100°C and 700°C, respectively as shown in figure.

The brick wall is covered by an insulating material of thermal conductivity k2. The thickness of the insulation is 1/4th of the thickness of the brick wall. The outer surface of the insulation is at 200°C. The heat flux through the composite walls is 2500 W/m2.

The value of k2 is _____W/m.K (round off to one decimal place).

Answer: (0.5 to 0.5)

25. If a reversed Carnot cycle operates between the temperature limits of 27°C and −3°C, then the ratio of the COP of a refrigerator to that of a heat pump (COP of refrigerator / COP of heat pump) based on the cycle is________ (round off to 2 decimal places).

Answer: (0.89 to 0.91)

Q26 – Q55 carry two marks each.

26. The directional derivative of f(x, y, z) = xyz at point (−1, 1, 3) in the direction of vector  is

(A)  

(B)  −7/3

(C)  7/3

(D)  7

Answer: (C)

27. The function f(z) of complex variable z = x + iy, where i = √−1, is given as f(z) = (x3 – 3xy2) + i v(x, y). For this function to be analytic, v(x, y) should be

(A)  (3xy2 – y3) + constant

(B)  (3x2y2 – y3) + constant

(C)  (x3 – 3x2y) + constant

(D)  (3x2y – y3) + constant

Answer: (D)

28. A cantilever of length l, and flexural rigidity EI, stiffened by a spring of stiffness k, is loaded by a transverse force P, as shown.

The transverse deflection under the load is

Answer: (D)

29. The sun (S) and the planet (P) of an epicyclic gear train shown in the figure have identical number of teeth.

If the sun (S) and the outer ring (R) gears are rotated in the same direction with angular speed ωS and ωR. Respectively, then the angular speed of the arm AB is

Answer: (A)

30. A thin-walled cylinder of radius r and thickness t is open at both ends, and fits snugly between two rigid walls under ambient conditions, as shown in the figure.

The material of the cylinder has Young’s modulus E, Poisson’s ratio v, and coefficient of thermal expansion α. What is the minimum rise in temperature ∆T of the cylinder (assume uniform cylinder temperature with no buckling of the cylinder) required to prevent gas leakage if the cylinder has to store the gas at an internal pressure of p above the atmosphere?

Answer: (C)

31. A helical spring has spring constant k. If the wire diameter, spring diameter and the number of coils are all doubled then the spring constant of the new spring becomes

(A)  k/2

(B)  k

(C)  8k

(D)  16k

Answer: (B)

32. Two rollers of diameters D1(in mm) and D2 (in mm) are used to measure the internal taper angle in the V-groove of a machined component. The heights H1 (in mm) and H2 (in mm) are measured by using a height gauge after inserting the rollers into the same V-groove as shown in the figure.

Which one of the following is the correct relationship to evaluate the angle α as shown in the figure?

Answer: (A)

33. The forecast for the monthly demand of a product is given in the table below.

The forecast is made by using the exponential smoothing method. The exponential smoothing coefficient used in the forecasting the demand is

(A)  0.10

(B)  0.40

(C)  0.50

(D)  1.00

Answer: (A)

34. One kg of air in a closed system undergoes an irreversible process from an initial state of p1 = 1 bar (absolute) and T1 = 27°C, to a final state of p2 = 3 bar (absolute) and T2 = 127° If the gas constant of air is 287 J/kg.K and the ratio of the specific heats γ = 1.4, then the change in the specific entropy (in J/kg.K) of the air in the process is

(A)  −26.3

(B)  28.4

(C)  172.0

(D)  indeterminate, as the process is irreversible

Answer: (A)

35. For the integral  the absolute percentage error in numerical evaluation with the Trapezoidal rule, using only the end points, is ________ (round off to one decimal place).

Answer: (5.1 to 5.5)

36. A fair coin is tossed 20 times. The probability that ‘head’ will appear exactly 4 times in the first ten tosses, and ‘tail’ will appear exactly 4 times in the next ten tosses is ________ (round off to 3 decimal places).

Answer: (0.041 to 0.043)

37. A hollow spherical ball of radius 20 cm floats in still water, with half of its volume submerged. Taking the density of water as 1000 kg/m3, and the acceleration due to gravity as 10 m/s2, the natural frequency of small oscillations of the ball, normal to the water surface is ________ radian/s (round off to 2 decimal places).

Answer: (8.60 to 8.70)

38. Uniaxial compression test data for a solid metal bar of length 1 m is shown in the figure.

The bar material has a linear elastic response from O to P followed by a nonlinear response. The point P represents the yield point of the material. The rod is pinned at both the ends. The minimum diameter of the bar so that it does not buckle under axial loading before reaching the yield  point is _______mm (round off to one decimal place).

Answer: (55.0 to 58.0)

39. The turning moment diagram of a flywheel fitted to a fictitious engines is shown in the figure.

The mean turning moment is 2000 Nm. The average engine speed is 1000 rpm. for fluctuation in the speed to be within ±2% of the average speed, the mass moment o inertia of the flywheel is ______ kg.m2.

Answer: (3.55 to 3.65)

40. A rigid block of mass m1 = 10 kg having velocity v0 = 2 m/s strikes a stationary block of mass m2 = 30 kg after traveling 1 m along a frictionless horizontal surface as shown in the figure.

The two masses stick together and jointly move by a distance of 0.25 m further along the same frictionless surface, before they touch the mass-less buffer that is connected to the rigid vertical wall by means of a linear spring having a spring constant k = 105 N/m. The maximum deflection of the spring is _________ cm (round off to 2 decimal places).

Answer: (0.98 to 1.02)

41. A steel spur pinion has a module (m) of 1.25 mm, 20 teeth and 20° pressure angle. The pinion rotates at 1200 rpm and transmits power to a 60 teeth gear. The face width (F) is 50 mm, Lewis form factor Y = 0.322 and a dynamic factor Kv = 1.26. The bending stress (σ) induced in a tooth can be calculated by using the Lewis formula given below.

If the maximum bending stress experienced by the pinion is 400 MPa, the power transmitted is _______ kW (round off to one decimal place).

Lewis formula:   where Wt is the tangential load acting on the pinion.

Answer: (9.8 to 10.8)

42. A mould cavity of 1200 cm3 volume has to be filled through a sprue of 10 cm length feeding a horizontal runner. Cross-sectional area at the base of the sprue is 2 cm2. Consider acceleration due to gravity as 9.81 m/s2. Neglecting frictional losses due to molten metal flow, the time taken to fill the mould cavity is ________ seconds (round of to 2 decimal places).

Answer: (4 to 9)

43. A cylindrical bar with 200 mm diameter is being turned with a tool having geometry 0° – 9° – 7° – 8° – 15°- 30° – 0.05 inch (Coordinate system, ASA) resulting in a cutting force Fc1. If the tool geometry is changed to 0° – 9° – 7° – 8° – 15° – 0° – 0.05 inch (Coordinate system, ASA) and all the other parameters remain unchanged, the cutting force changes to Fc2. Specific cutting energy (in J/mm3) is Uc = U0(t1)4, where U0 is the specific energy coefficient, and t1 is the uncut thickness in mm. The value of percentage change in cutting force Fc2, i.e.  is ________ (round off to one decimal place).

Answer: (-5.8 to -5.5)

44. There are two identical shaping machines S1 and S2. In machine S2, the width of the workpiece is increased by 10% and the feed is decreased by 10%, with respect to that of S1. If all other conditions remain the same then the ratio of total time per pass in S1 and S2 will be ________ (round of to one decimal place).

Answer: (0.7 to 0.9)

45. Bars of 250 mm length and 25 mm diameter are to be turned on a lathe with a feed of 0.2 mm/rev. Each regrinding of the tool costs Rs. 20. The time required for each tool change is 1 min. Tool life equation is given as VT2 = 24 (where cutting speed V is in m/min and tool life T is in min). The optimum tool cost per piece for maximum production rate is Rs. _______ (round off to 2 decimal places).

Answer: (26.00 to 28.00)

46. A point ‘P’ on a CNC controlled XY-stage is moved to another point ‘Q’ using the coordinate system shown in the figure below and rapid positioning command (G00).

A pair of stepping motors with maximum sped of 800 rpm, controlling both the X and Y motion of the stage, are directly coupled to a pair of lead screws, each with a uniform pitch of 0.5 mm. The time needed to position the point ‘P’ to the point ‘Q’ is _________ minutes (round off to 2 decimal places).

Answer: (1.40 to 1.60)

47. For a single item inventory system, the demand is continuous, which is 10000 per year. The replenishment is instantaneous and backorders (S units) per cycle are allowed as shown in the figure.

As soon as the quantity (Q units) ordered from the supplier is received, the backordered quantity is issued to the customers. The ordering cost is Rs. 300 per order. The carrying cost is Rs. 4 per unit per year. The cost of backordering is Rs. 25 per unit per year. Based on the total cost minimization criteria, the maximum inventory reached in the system is __________ (round off to nearest integer).

Answer: (1130 to 1140)

48. Consider a flow through a nozzle, as shown in the figure below.

The air flow is steady, incompressible and inviscid. The density of air is 1.23 kg/m3. The pressure difference (p1 – patm) is _______ kPa (round off to 2 decimal places).

Answer: (1.50 to 1.55)

49. Water (density 1000 kg/m3) flows through an inclined pipe of uniform diameter. The velocity, pressure and elevation at section A are VA = 3.2 m/s, pA = 186 kPa and zA = 24.5 m, respectively, and those at section B are VB = 3.2 m/s, pB = 260 kPa and zB = 9.1 m, respectively. If acceleration due to gravity is 10 m/s2 then the head lost due to friction is _________ m (round off to one decimal place).

Answer: (7.9 to 8.1)

50. The spectral distribution of radiation from a black body at T1 = 3000 K has a maximum at wavelength λmax. The body cools down to a temperature T2. If the wavelength corresponding to the maximum of the spectral distribution at T2 is 1.2 times of the original wavelength λmax, then the temperature T2 is ________ K (round off to the nearest integer).

Answer: (2499 to 2501)

51. Water flows through a tube of 3 cm internal diameter and length 20 m. The outside surface of the tube is heated electrically so that it is subjected to uniform heat flux circumferentially and axially. The mean inlet and exit temperatures of the water are 10°C and 70°C, respectively. The mass flow rate of the water is 720 kg/h. Disregard the thermal resistance of the tube wall. The internal heat transfer coefficient is 1697 W/m2.K. Take specific heat Cp of water as 4.179 kJ/kg.K. The inner surface temperature at the exit section of the tube is ________ °C (round off to one decimal place).

Answer: (85.1 to 86.1)

52. Air is contained in a frictionless piston-cylinder arrangement as shown in the figure.

The atmospheric pressure is 100 kPa and the initial pressure of air in the cylinder is 105 kPa. The area of piston is 300 cm2. Heat is now added and the piston moves slowly from its initially position until it reaches the stops. The spring constant of the linear spring is 12.5 N/mm. Considering the air inside the cylinder as the system, the work interaction is _________ J (round off to the nearest integer).

Answer: (543 to 545)

53. Moist air at 105 kPa, 30°C and 80% relative humidity flows over a cooling coil in an insulated air-conditioning duct. Saturated air exits the duct at 100 kPa and 15° The saturation pressures of water at 30°C and 15°C are 4.24 kPa and 1.7 kPa respectively. Molecular weight of water is 18 g/mol and that of air is 28.94 g/mol. The mass of water condensing out from the duct is ______ g/kg of dry air (round off to the nearest integer).

Answer: (10 to 10)

54. In a stream power plant, superheated steam at 10 MPa and 500°C, is expanded isentropically in a turbine until it becomes a saturated vapour. It is then reheated at constant pressure to 500° The steam is next expanded isentropically in another turbine until it reaches the condenser pressure of 20 kPa. Relevant properties of steam are given in the following two tables. The work done by both the turbines together is _______ kJ/kg (round off to the nearest integer).

Answer: (1500 to 1525)

55. Keeping all other parameters identical, the Compression Ratio (CR) of an air standard diesel cycle is increased from 15 to 21. Take ratio of specific heats = 1.3 and cut-off ratio of the cycle rc = 2.

The difference between the new and the old efficiency values,  in percentage.

new|CR=21) – (ηold|CR=15) = __________% (round off to one decimal place).

Answer: (4.6 to 4.9)

GATE Exam 2020 Mechanical Engineering (ME-1) Question Paper With Answer Key

GATE-2020

ME-1: Mechanical Engineering

GA – General Aptitude

Q1 – Q5 carry one mark each.

1. He is known for his unscrupulous ways. He always sheds______ tears to deceive people.

(A)  fox’s

(B)  crocodile’s

(C)  crocodile

(D)  fox

Answer: (C)

2. Jofra Archer, the England fast bowler, is ____ than accurate.

(A)  more fast

(B)  faster

(C)  less fast

(D)  more faster

Answer: (A)

3. Select the word that fits the analogy:

Build : Building :: Grow : ______

(A)  Grown

(B)  Grew

(C)  Growth

(D)  Growed

Answer: (C)

4. I do not think you know the case well enough to have opinions. Having said that, I agree with your other point.

What does the phrase “having said that” mean in the given text?

(A)  as opposed to what I have said

(B)  despite what I have said

(C)  in addition to what I have said

(D)  contrary to what I have said

Answer: (B)

5. Define [x] as the greatest integer less than or equal to x, for each x ∈ (−∞, ∞). If y = [x], then area under y for x ∈ [1, 4] is _______.

(A)  1

(B)  3

(C)  4

(D)  6

Answer: (D)

Q6 – Q10 carry two marks each.

6. Crowd funding deals with mobilization of funds for a project from a large number of people, who would be willing to invest smaller amounts through web-based platforms in the project.

Based on the above paragraph, which of the following is correct about crowd funding?

(A)  Funds raised through unwilling contributions on web-based platforms.

(B)  Funds raised through large contributions on web-based platforms.

(C)  Funds raised through coerced contributions on web-based platforms.

(D)  Funds raised through voluntary contributions on web-based platforms.

Answer: (D)

7. P, Q, R and S are to be uniquely coded using α and β. If P is coded as αα and Q as αβ, then R and S, respectively, can be coded as ______.

(A)  βα and αβ

(B)  ββ and αα

(C)  αβ and ββ

(D)  βα and ββ

Answer: (D)

8. The sum of the first n terms in the sequence 8, 88, 888, 8888, … is ______.

(A) 

(B)   

(C)   

(D)   

Answer: (D)

9. Select the graph that schematically represents BOTH y = xm and y = x1/m properly in the interval 0 ≤ x ≤ 1, for integer values of m, where m > 1.

Answer: (A)

10. The bar graph shows the data of the students who appeared and passed in an examination for four schools P, Q, R and S. The average of success rates (in percentage) of these four schools is ________.

(A)  58.5%

(B)  58.8%

(C)  59.0%

(D)  59.3%

Answer: (C)

ME1: Mechanical Engineering

Q1 – Q25 carry one mark each.

1. Multiplication of real valued square matrices of same dimension is

(A)  associative

(B)  commutative

(C)  always positive definite

(D)  not always possible to compute

Answer: (A)

2. The value of  is

(A)  c

(B)  c + 1

(C)   

(D)   

Answer: (C)

3. The Laplace transform of a function f(t) is  Then, f(t) is

(A)   

(B)   

(C)   

(D)   

Answer: (C)

4. Which of the following function f(z), of the complex variable z, is NOT analytic at all the points of the complex plane?

(A)  f(z) = z2

(B)  f(z) = ez

(C)  f(z) = sin z

(D)  f(z) = log z

Answer: (D)

5. The members carrying zero force (i.e. zero-force members) in the truss shown in the figure, for any load P > 0 with no appreciable deformation of the truss (i.e. with no appreciable change in angles between the members), are

(A)  BF and DH only

(B)  BF, DH and GC only

(C)  BF, DH, GC, CD and DE only

(D)  BF, DH, GC, FG and GH only

Answer: (C)

6. A single-degree-freedom oscillator is subjected to harmonic excitation F(t) = F0 cos(ωt) as shown in the figure.

The non-zero value of ω, for which the amplitude of the force transmitted to the ground will be F0, is

Answer: (C)

7. The stress state at a point in a material under plane stress condition is equi-biaxial tension with a magnitude of 10 MPa. If one unit on the σ – τ plane is 1 MPa, the Mohr’s circle representation of the state-of-stress is given by

(A)  a circle with a radius equal to principal stress and its center at the origin of the σ – τ plane

(B)  a point on the σ axis at a distance of 10 units from the origin

(C)  a circle with radius of 10 units of the σ – τ plane

(D)  a point on the τ axis at a distance of 10 units from origin

Answer: (B)

8. A four bar mechanism is shown below.

For the mechanism to be a crank-rocker mechanism, the length of the link PQ can be

(A)  80 mm

(B)  200 mm

(C)  300 mm

(D)  350 mm

Answer: (A)

9. A helical gear with 20° pressure angle and 30° helix angle mounted at the mid-span of a shaft that is supported between two bearings at the ends. The nature of the stress induced in the shaft is

(A)  normal stress due to bending only

(B)  normal stress due to bending in one plane and axial loading; shear stress due to torsion

(C)  normal stress due to bending in two planes and axial loading; shear stress due to torsion

(D)  normal stress due to bending in two planes; shear stress due to torsion

Answer: (A OR C)

10. The crystal structure of γ iron (austenite phase) is

(A)  BCC

(B)  FCC

(C)  HCP

(D)  BCT

Answer: (B)

11. Match the following.

(A)  P-2, Q-3, R-4, S-1

(B)  P-1, Q-1, R-3, S-2

(C)  P-3, Q-3, R-1, S-3

(D)  P-4, Q-3, R-2, S-1

Answer: (A)

12. The base of a brass bracket needs rough grinding. For this purpose, the most suitable grinding wheel grade specification is

(A)  C30Q12V

(B)  A50G8V

(C)  C90J4B

(D)  A30D12V

Answer: (A)

13. In the Critical Path Method (CPM), the cost-time slope of an activity is given by

Answer: (D)

14. Froude number is the ratio of

(A)  buoyancy forces to viscous forces

(B)  inertia forces to viscous forces

(C)  buoyancy forces to inertia forces

(D)  inertia forces to gravity forces

Answer: (D)

15. Match the following non-dimensional numbers with the corresponding definitions:

(A)  P-1, Q-3, R-2, S-4

(B)  P-3, Q-1, R-2, S-4

(C)  P-4, Q-3, R-1, S-2

(D)  P-3, Q-1, R-4, S-2

Answer: (D)

16. The velocity filed of an incompressible flow in a Cartesian system is represented by  Which one of the following expressions for v is valid?

(A)  −4xz + 6xy

(B)  −4xy − 4xz

(C)  4xz – 6xy

(D)  4xy + 4xz

Answer: (B)

17. For an ideal gas, the value of the Joule-Thomson coefficient is

(A)  positive

(B)  negative

(C)  zero

(D)  indeterminate

Answer: (C)

18. For an ideal gas, a constant pressure line and a constant volume line intersect at a point, in the Temperature (T) versus specific entropy (s) diagram. CP is the specific heat at constant pressure and C­V is the specific heat at constant volume. The ratio of the slopes of the constant pressure and constant volume lines at the point of intersection is

(A)  

(B)  CP/CV

(C)   

(D)  CV/CP

Answer: (D)

19. For three vectors  and  are unit vectors along the axes of a right-handed rectangular/Cartesian coordinate system, the value of  is _______.

Answer: (6 to 6)

20. A flywheel is attached to an engine to keep its rotational speed between 100 rad/s and 110 rad/s. If the energy fluctuation in the flywheel between these two speeds is 1.05 kJ then the moment of inertia of the flywheel is _______ kg.m2 (round off to 2 decimal places).

Answer: (0.98 to 1.02)

21. A balanced rigid disc mounted on a rigid roto has four identical point masses, each of 10 grams, attached to four points on the 100 mm radius circle shown in the figure.

The rotor is driven by a motor at uniform angular speed of 10 rad/s. If one of the masses gets detached then the magnitude of the resultant unbalance force on the rotor is _______ N (round off to 2 decimal places).

Answer: (0.09 to 0.11)

22. A sheet metal with a stock hardness of 250 HRC has to be sheared using a punch and a die having a clearance of 1 mm between them. If the stock hardness of the sheet metal increases to 400 HRC, the clearance between the punch and the die should be _______mm.

Answer: (1.0 to 1.3)

23. A company is hiring to fill four managerial vacancies. The candidates are five men and three women. If every candidate is equally likely to be chosen then the probability that at least one woman will be selected is _______ (round off to 2 decimal places).

Answer: (0.90 to 0.95)

24. The compressor of a gas turbine plant, operating on an ideal intercooled Brayton cycle, accomplishes an overall compression ratio of 6 in two-stage compression process. Intercooling is used to cool the air coming out from the first stage to the inlet temperature of the first stage, before its entry to the second stage. Air enters the compressors at 300 K and 100 KPa. If the properties of gas are constant, the intercooling pressure for minimum compressor work is ________kPa (round off to 2 decimal places).

Answer: (MTA)

25. In a concentric tube counter-flow heat exchanger, hot oil enters at 102°C and leaves at 65° Cold water enters at 25°C and leaves at 42°C. The log mean temperature difference (LMTD) is __________°C (round of to one decimal place)

Answer: (48.8 to 49.8)

Q26 – Q55 carry two marks each.

26. The evaluation of the definite integral  by using Simpson’s 1/3rd (one-third) rule with step size h = 0.6 yields

(A)  0.914

(B)  1.248

(C)  0.581

(D)  0.592

Answer: (D)

27. A vector field is defined as

where  are unit vectors along the axes of right-handed rectangular /Cartesian coordinate system. The surface integral  is an elemental surface area vector) evaluated over the inner and outer surfaces of a spherical shell formed by two concentric spheres with origin as the center, and internal and external radii of 1 and 2, respectively, is

(A)  0

(B)  2π

(C)  4π

(D)  8π

Answer: (A)

28. Bar of square and circular cross-section with 0.5 m length are made of a material with shear strength of 20 MPa. The square bar cross-section dimension is 4 cm × 4 cm and the cylindrical bar cross-section diameter is 4 cm. The specimens are loaded as shown in the figure.

Which specimen(s) will fail due to the applied load as per maximum shear stress theory?

(A)  Tensile and compressive load specimens

(B)  Torsional load specimen

(C)  Bending load specimen

(D)  None of the specimens

Answer: (A)

29. The 2 kg block shown in figure (top view) rests on a smooth horizontal surface and is attached to massless elastic cord that has a stiffness 5 N/m.

The cord hinged at O is initially unstretched and always remains elastic. The block is given a velocity v of 1.5 m/s perpendicular to the cord. The magnitude of velocity in m/s of the block at the instant the cord is stretched by 0.4 m is

(A)  0.83

(B)  1.07

(C)  1.36

(D)  1.50

Answer: (C)

30. The truss shown in the figure has four members of length l and flexural rigidity EI, and one member of length l√2 and flexural rigidity 4EI. The truss is loaded by a pair of forces of magnitude P, as shown in the figure.

The smallest value of P, at which any of the truss members will buckle is

Answer: (C)

31. A rigid mass-less rod of length L is connected to a disc (pulley) of mass m and radius r = L/4 through a friction-less revolute joint. The other end of that rod is attached to a wall through a friction-less hinge. A spring of stiffness 2k is attached to the rod at its mid-span. An inextensible rope passes over half the disc periphery and is securely tied to a spring of stiffness k at point C as shown in the figure. There is no slip between the rope and the pulley. The system is in static equilibrium in the configuration shown in the figure and the rope is always taut.

Neglecting the influence of gravity, the natural frequency of the system for small amplitude vibration is

Answer: (C)

32. A strip of thickness 40 mm is to be rolled to a thickness of 20 mm using a two-high mill having rolls of diameter 200 mm. Coefficient of friction and arc length in mm, respectively are

(A)  0.45 and 38.84

(B)  0.39 and 38.84

(C)  0.39 and 44.72

(D)  0.45 and 44.72

Answer: (D)

33. For an assembly line, the production rate was 4 pieces per hour and the average processing time was 60 minutes. The WIP inventory was calculated. Now, the production rate is kept the same, and the average processing time is brought down by 30 percent. As a result of this change in the processing time, the WIP inventory

(A)  decreases by 25%

(B)  increases by 25%

(C)  decreases by 30%

(D)  increases by 30%

Answer: (C)

34. A small metal bead (radius 0.5 mm), initially at 100°C, when placed in a stream of fluid at 20°C, attains a temperature of 28°C in 4.35 seconds. The density specific heat of the metal are 8500 kg/m3 and 400 J/kg.K, respectively. If the bead is considered as lumped system, the convective heat transfer coefficient (in W/m2.K) between the metal bead and the fluid steam is

(A)  283.3

(B)  299.8

(C)  149.9

(D)  449.7

Answer: (B)

35. Consider two exponentially distributed random variables X and Y, both having a mean of 0.50. Let Z = X + Y and r be the correlation coefficient between X and Y. If the variance of Z equals 0, then the value of r is ________ (round off to 2 decimal places).

Answer: (-1.00 to -0.98)

36. An analytic function of a complex variable z = x + iy(i = √−1) is defined as f(z) = x2 – y2 + iψ(x, y), where iψ(x, y) is a real function. The value of imaginary part of f(z) at z = (1+ i) is ________( round off to 2 decimal places).

Answer: (1.99 to 2.01)

37. In a disc-type axial clutch, the frictional contact takes place within an annular region with outer and inner diameters 250 mm and 50 mm, respectively. An axial force F1 is needed to transmit a torque by a new clutch. However, to transmit the same torque, one needs an axial force F2 when the clutch wears out. If contact pressure remains uniform during operation of a new clutch while the wear is assumed to be uniform for an old clutch, and the coefficient of friction does not change, then the ratio F1/F2 is _______ (round off to 2 decimal places).

Answer: (0.85 to 0.89)

38. A cam with a translating flat-face follower is desired to have the follower motion y(θ) = 4[2πθ – θ2], 0 ≤ θ ≤ 2π. Contact stress considerations dictate that the radius of curvature of the cam profile should not be less than 40 mm anywhere. The minimum permissible base circle radius is _______ mm (round off to one decimal place).

Answer: (47.9 to 48.1)

39. A rectangular steel bar of length 500 mm, width 100 mm, and thickness 15 mm is cantilevered to a 200 mm steel channel using 4 bolts, as shown.

For an external load of 10 kN applied at the ip of the steel bar, the resultant shear load on the bolt at B, is _______ kN (round off to one decimal place).

Answer: (15.9 to 16.1)

40. The barrier shown between two water tanks of unit width (1 m) into the plane of the screen is modeled as cantilever.

Taking the density of water as 1000 kg/m3, and the acceleration due to gravity as 10 m/s2, the maximum absolute bending moment developed in the cantilever is ______kN∙m (round off to the nearest integer).

Answer: (104 to 106)

41. The magnitude of reaction force at joint C o the hinge-beam shown in the figure is _______ kN (round off to 2 decimal places).

Answer: (19.95 to 20.05)

42. A slot of 25 mm × 25 mm is to be billed in a workpiece of 300 mm length using a side and face milling cutter of diameter 100 mm, width 25 mm and having 20 teeth.

For a depth of cut 5 mm, feed per tooth 0.1 mm, cutting speed 35 m/min and approach and over travel distance of 5 mm each, the time required for milling the slot is _____ minutes (round off to one decimal place).

Answer: (7 to 9)

43. The following data applies to basic shaft system:

tolerance for hole = 0.002 mm,

tolerance for shaft = 0.001 mm,

allowance = 0.003 mm,

basic size = 50 mm.

The maximum hole size is ____ mm (round off to 3 decimal places).

Answer: (50.005 to 50.005)

44. A steel part with surface area of 125 cm2 is to be chrome coated through an electroplating process using chromium acid sulphate as an electrolyte. An increasing current is applied to the part according to the following current time relation :

I = 12 + 0.2t

where, I = current (A) and t = time (minutes). The part is a submerged in the plating solution for a duration of 20 minutes for plating purpose. Assuming the cathode efficiency of chromium to be 15% and the plating constant of chromium acid sulphate to be 2.50 × 102 mm3/A∙s, the resulting coating thickness on the part surface is _______ μm (round off to one decimal place).

Answer: (0 to 0)

45. In a turning process using orthogonal tool geometry, a chip length of 100 mm is obtained for an uncut chip length of 250 mm.

The cutting conditions are: cutting speed = 30 m/min, rake angle = 20°.

The shear plane angle is _______ degrees (round off to one decimal place).

Answer: (22 to 25)

46. The thickness of a steel plate with material strength coefficient of 210 MPa, has to be reduced from 20 mm to 15 min in a single pass in two-high rolling mill with a roll radius of 450 mm and rolling velocity of 28 m/min. If the plate has a width of 200 mm and its strain hardening exponent, n is 0.25, the rolling force required for the operation is ______ kN (round off to 2 decimal places).

Answer: (1164 to 1168)

47. Two business owners Shveta and Ashok run their businesses in two different states. Each of them, independent of the other, produces two products A and B, sells them at Rs. 2,000 per kg and Rs. 3,000 per kg, respectively, and uses Linear Programming to determine the optimal quantity of A and B to maximize their respective daily revenue. Their constraints are as follows: i) for each business owner, the production process is such that the daily production of A has to be at least as much as B, and the upper limit for production of B is 10 kg per day, and (ii) the respective state regulations restrict Shveta’s production of A to less than 20 kg per day, and Ashok’s production of A to less than 15 kg per day. The demand of both A and B in both the states is very high and everything produced is sold.

The absolute value of the difference in daily (optimal) revenue of Shveta and Ashok is _________ thousand Rupees (round off to 2 decimal places).

Answer: (9.90 to 10.10)

48. Consider two cases as below.

Case 1: A company buys 1000 pieces per year of a certain part from vendor ‘X’. The changeover time is 2 hours and the price is Rs. 10 per piece. The holding cost rate per part is 10% per year.

Case 2: For the same part, another vendor ‘Y’ offers a design where the changeover time is 6 minutes, with a price of Rs. 5 per piece, and a holding cost rate per part of 100% per year. The order size is 800 pieces per year from ‘X’ and 200 pieces per year from ‘Y’.

Assume the cost of downtime as Rs. 200 per hour. The percentage reduction in the annual cost for Case 2, as compared to Case 1 is ______ (round off to 2 decimal places).

Answer: (8.19 to 8.23)

49. Consider steady, viscous, fully developed flow of a fluid through a circular pipe of internal diameter D. We know that the velocity profile forms a paraboloid about the pipe centre line, given by:   where C is a constant. The rate of kinetic energy (in J/s) at the control surface A-B, as shown in the figure, is proportional to Dn. The value of n is ______.

Answer: (8 to 8)

50. Air discharges steadily through a horizontal nozzle and impings one a stationary vertical plate as shown in figure.

The inlet and outlet area of the nozzle are 0.1 m2 and 0.02 m2, respectively. Take air density as constant and equal to 1.2 kg/m3. If the inlet gauge pressure of air is 0.36 kPa, the gauge pressure at point O on the plate is ______ kPa (round off to two decimal places)

Answer: (0.37 to 0.45)

51. Air (ideal gas) enters a perfectly insulated compressor at a temperature of 310 K. The pressure ratio of the compressor is 6. Specific heat at constant pressure for air is 1005 J/kg.K and ratio of specific heats at constant pressure constant volume is 1.4. Assume that specific heats of air are constant. If the isentropic efficiency of the compressor is 85 percent, the difference in enthalpies of air between the exit and the inlet of the compressor is ______ kJ/kg (round off to nearest integer).

Answer: (244 to 246)

52. One kg of air, initially at a temperature of 127°C, expands reversible at a constant pressure until the volume is doubled. If the gas constant of air is 287 J/kg.K, the magnitude of work transfer is ______ kJ (round off to 2 decimal places).

Answer: (114.6 to 115.0)

53. For an ideal Rankine cycle operating between pressures of 30 bar and 0.04 bar, the work output from the turbine is 903 kJ/kg and the work input to the feed pump is 3 kJ/kg. The specific steam consumption is _______ kg/kW.h (round of to 2 decimal places).

Answer: (3.98 to 4.02)

54. For a Kaplan (axial flow) turbine, the outlet blade velocity diagram at a section is shown in figure.

The diameter at this section is 3 m. The hub and tip diameters of the blade are 2 n and 4 m, respectively. The water volume flow rate is 100 m3/s. The rotational speed of the turbine is 300 rpm. The blade outlet angle β is _____ degrees (round off to one decimal place).

Answer: (11.0 to 14.0)

55. The indicated power developed by an engine with compression ratio of 8, is calculated using an air-standard Otto cycle (constant properties). The rate of heat addition is 10 kW. The ratio of specific heats at constant pressure and constant volume is 1.4. The mechanical efficiency of the engine is 80 percent.

The brake power output of the engine is ______kW (round off to one decimal place).

Answer: (4.4 to 4.6)

GATE Exam 2020 Mathematics (MA) Question Paper With Answer Key

GATE-2020

MA: Mathematics

GA-General Aptitude

Q1-Q5 carry one mark each.

1. Rajiv Gandhi Khel Ratna Award was conferred _____Mary Kom, a six-time world champion in boxing, recently in a ceremony _____ the Rashtrapati Bhawan (the President’s official residence) in New Delhi.

(A)  with, at

(B)  on, in

(C)  on, at

(D)  to, at

Answer: (C)

2. Despite a string of a poor performances, the changes of K. L. Rahul’s selection in the team are ______.

(A)  slim

(B)  bright

(C)  obvious

(D)  uncertain

Answer: (B)

3. Select the word that fits the analogy:

Cover : Uncover :: Associate : _______

(A)  Unassociate

(B)  Inassociate

(C)  Missassociate

(D)  Dissociate

Answer: (D)

4. Hig by floods, he kharif (summer sown) crops in various parts of the county have been affected. Officials believe that the loss in production of the kharif crops can be recovered in the output of the rabi (winter sown) crops so that the country can achieve its food-grain production target of 291 million tons in the crop year 2019-20 (July-June). They are hopeful that good rains in July-August will help the soil retain moisture for a longer period, helping winter sown crops such as wheat and pulses during the November-February period.

Which of the following statements can be inferred from the given passage?

(A)  Officials declared that the food-grain production target will be met due to good rains.

(B)  Officials want the food-grain production target to be met by the November-February period.

(C)  Officials feel that the food-grain production target cannot be met due to floods.

(D)  Officials hope that the food-grain production target will be met due to a good rabi produce.

Answer: (D)

5. The difference between the sum of the first 2n natural numbers and the sum of the first n odd natural numbers is ______.

(A)  n2 – n

(B)  n2 + n

(C)  2n2 – n

(D)  2n2 + n

Answer: (B)

Q6-Q10 carry two marks each.

6. Repo rate is the at which Reserve Bank of India (RBI) lends commercial banks, and reverse repo rate is the rate at which RBI borrows money from commercial banks.

Which of the following statements can be inferred from the above passage?

(A)  Decrease in repo rate will increase cost of borrowing and decrease lending by commercial banks.

(B)  Increase in repo rate will decrease cost of borrowing and increase lending by commercial banks.

(C)  Increase in repo rate will decrease cost of borrowing and decrease lending by commercial banks.

(D)  Decrease in repo rate will decrease cost of borrowing and increase lending by commercial banks.

Answer: (D)

7. P, Q, R, S, T, U, V, and W are seated around a circular table.

(I) S is seated opposite to W.

(II) U is seated at the second place to the right of R.

(III) T is seated at the third place to the left of R.

(IV) V is a neighbor of S.

Which of the following must be true?

(A)  P is a neighbor of R.

(B)  Q is a neighbor of R.

(C)  P is not seated opposite to Q.

(D)  R is the left neighbor of S.

Answer: (C)

8. The distance between Delhi and Agra is 233 km. A car P started travelling from Delhi to Agra and another car Q started from Agra to Delhi along the same road 1 hour after the car P started. The two cars crossed each other 75 minutes after the car Q started. Both cars were travelling at constant speed. The speed of car P was 10 km/hr more than the speed of car Q. How many kilometers the car Q had travelled when the cars crossed each other?

(A)  66.6

(B)  75.2

(C)  88.2

(D)  116.5

Answer: (B)

9. For a matrix M = [mij], i. j= 1, 2, 3, 4, the diagonal elements are all zero and mij = −mij. The minimum number of elements required to fully specify the matrix is_______.

(A)  0

(B)  6

(C)  12

(D)  16

Answer: (B)

10. The profit shares of two companies P and Q are shown in the figure. If the two companies have invested a fixed and equal amount every year, then the ratio of the total revenue of company P to the total revenue of company Q, during 2013-2018 is ______.

(A)  15 : 17

(B)  16 : 17

(C)  17 : 15

(D)  17 : 16

Answer: (B)

MA: Mathematics

Q1 – Q25 carry one mark each.

1. Suppose that ℑ1 and ℑ2 are topologies on X induced by metrics d1 and d2, respectively, such that ℑ1 ⊆ ℑ2. Then which of the following statements is TRUE?

(A)  If a sequence converges in (X, d2) then it converges in (X, d1)

(B)  If a sequence converges in (X, d1) then it converges in (X, d2)

(C)  Every open ball in (X, d1) is an open ball in (X, d2)

(D)  The map x → x from (X, d1) to (X, d2) is continuous

Answer: (A)

2. Let D = [−1, 1] × [−1, 1]. If the function f : D → ℝ is defined by

then

(A)  f is continuous at (0, 0)

(B)  both the first order partial derivatives of f exist at (0, 0)

(C)  ∬D |f(x, y)|1/2 dx dy is finite

(D)  ∬D |f(x, y)| dx dy is finite

Answer: (C)

3. The initial value problem y′ = y3/5, y(0) = b has

(A)  a  unique solution if b = 0

(B)  no solution if b = 1

(C)  infinitely many solutions if b = 2

(D)  a unique solution if b = 1

Answer: (D)

4. Consider the following statements:

I : log(|z|) is harmonic on ℂ\{0}

II: log(|z|) has a harmonic conjugate on ℂ\{0}

Then

(A)  both I and II are true

(B)  I is true but II is false

(C)  I is false but II is true

(D)  both I and II are false

Answer: (B)

5. Let G and H be defined by

G = ℂ\ {z =  x + iy ∈ ℂ : x ≤ 0, y = 0},

H = ℂ\ {z = x + iy ∈ ℂ : x ∈ ℤ, x ≤ 0, y = 0}.

Suppose  f: G → ℂ and g : H → ℂ are analytic functions. Consider the following statements:

I : ∫γ f dz is independent of pathys γ in G joining –i and i

II : ∫γ g dz is independent of paths γ in H joining –i and i

Then

(A)  both I and II are true

(B)  I is true but II is false

(C)  I is false but II true

(D)  both I and II are false

Answer: (B)

6. Let f (z) = e1/z, z ∈ ℂ \ {0} and let, for n ∈ ℕ,

If for a subset S of ℂ,  denotes the closure of S in ℂ, then

Answer: (A or D)

7. Suppose that

Then, with respect to the Eclidean metric on ℝ2,

(A)  both U and V are disconnected

(B)  U is disconnected but V is connected

(C)  U is connected but V is disconnected

(D)  both U and V are connected

Answer: (C)

8. If (D1) and (D2) denote the dual problems of the linear programming problems (P1) and P2), respectively, where

(P1) : minimize x1­ – 2x2 subject to –x1 +x2 = 10, x1, x2 ≥ 0,

(P2) : minimize x1 – 2x2 subject to –x1 + x2 = 10, x1 – x2 = 10, x1, x2 ≥ 0, then

(A)  both (D1) and (D2) are infeasible

(B)  (P2) is infeasible and (D2) is feasible

(C)  (D1) is infeasible and (D2) is feasible but unbounded

(D)  (P1) is feasible but unbounded and (D1) is feasible

Answer: (A)

9. If (4, 0) and (0, −1/2) are critical points of the function

f(x, y) = 5 – (α + β) x2 + βy2 + (α + 1) y3 + x3,

where α, β ∈ ℝ, then

(A)  (4, −1/2) is a point of local maxima of f

(B)  (4, −1/2) is saddle point of f

(C)  α = 4, β = 2

(D)  (4, −1/2) is a point of local minima of f

Answer: (B)

10. Consider the iterative scheme

with initial point x0 > 0. Then the sequence {xn}

(A)  converges only if x0 > 1

(B)  converges only if x0 < 3

(C)  converges for any x­0

(D)  does not converge for any x­0

Answer: (C)

11. Let C[0, 1] denote the space of all real-valued continuous functions on [0, 1] equipped with the supremum norm || ∙ ||. Let T : C[0, 1] → C[0, 1] be the linear operator defined by

Then

(A)  || T || = 1

(B)  I – T is not invertible

(C)  T is surjective

(D)  ||I + T|| = 1+ ||T||

Answer: (D)

12. Suppose that M is a 5 × 5 matrix with real entries and p(x) = det(xI – M). Then

(A)  p(0) = det(M)

(B)  every eigenvalue of M is real if p(1) + (p)2 = 0 = p(2) + p(3)

(C)  M1 is necessarily a polynomial in M of degree 4 if M is invertible

(D)  M is not invertible if M2 – 2M = 0

Answer: (C)

13. Let C[0, 1] denote the space of all real-valued continuous functions on [0, 1] equipped with the supremum norm || ∙ ||. Let f ∈[0, 1] be such that

|f(x) – f(y)| ≤ M |x – y|, for all x, y ∈ [0, 1] and for some M  > 0.

For n ∈ ℕ, let fn(x) = f(x1+1/n). If S = {fn : n ∈ ℕ}, then

(A)  the closure of S is compact

(B)  S is closed and bounded

(C)  S is bounded but not totally bounded

(D)  S is compact

Answer: (A)

14. Let K : ℝ × (0, ∞) → ℝ be a function such that the solution of the initial value problem  u(x, 0), = f(x), x ∈ ℝ, t > 0, is given by

for all bounded continuous functions f. Then the value of  is ______

Answer: (1 to 1)

15. The number of cyclic subgroups of the quaternion group  is _________

Answer: (5 to 5)

16. The number of elements of order 3 in the symmetric group S6 is _______

Answer: (80 to 80)

17. Let F be the field with 4096 elements. The number of proper subfields of F is ______

Answer: (5 to 5)

18. If (x­1*, x2*) is an optimal solution of the linear programming problem, minimize x1 + 2x2 subject to

4x1 – x2 ≥8

2x1 + x2 ≥ 10

−x1 + x2 ≤ 7

x1, x2 ≥ 0

and (λ1*, λ2*, λ3*) in an optimal solution of its dual problem, then  is equal to ______ (correct up to one decimal place)

Answer: (5.5 to 5.5)

19. Let a, b, c ∈ ℝ be such that the quadrature rule

is exact for al polynomials of degree less than or equal to 2. Then b is equal to ______ (rounded off to two decimal places)

Answer: (1.70 to 1.80)

20. Let f(x) = x4 and let p(x) be the interpolating polynomial of f at nodes 1, 2 and 3. Then p(0) is equal to _______

Answer: (36 to 36)

21. For n ≥ 2, define the sequence {xn} by

Then the sequence {xn} converges to ______ (correct up to two decimal places)

Answer: (0.25 to 0.25)

22. Let

equipped with the norm  and let T be the linear functional on L2[0, 10] given by

Then ||T|| is equal to ______

Answer: (3 to 3)

23. If {x13 x22, x23 = 10, x31, x32, x34} is the set of basic variable of a balanced transportation problem seeking to minimize cost of transportation from origins to destinations, where the cost matrix is,

and λ, μ ∈ ℝ, then x32 is equal to ______

Answer: (5 to 5)

24. Let ℤ225 be the ring of integers modulo 225. If x is the number of prime ideals and y is the number of nontrivial units in ℤ225, then x + y is equal to _______

Answer: (121 to 121)

25. Let u(x, t) be the solution of

where f is a twice continuously differential function. If f(−2) = 4, f(0) = 0, and u(2, 2) = 8, then the value of u(1, 3) is ________

Answer: (10 to 10)

Q26 – Q55 carry two marks each.

26. Let  be an orthonormal basis for a separable Hilbert space H with the inner product  Define

Then

(A)  the closure of the span {fn : n ∈ ℕ} equals H

(B)  f = 0 if  for all n ∈ ℕ

(C)   is an orthogonal subset of H

(D)  there does not exist nonzero f ∈ H such that 

Answer: (A)

27. Suppose V is a finite dimensional non-zero vector space over ℂ and T : V → V is a linear transformation such that Range (T) = Nullspace (T). Then which of the following statements is FALSE ?

(A)  The dimension of V is even

(B)  0 is the only eigenvalue of T

(C)  Both 0 and 1 are eigenvalues of T

(D)  T2 = 0

Answer: (C)

28. Let P ∈ Mm×n(ℝ). Consider the following statements:

I : If XPY = 0 for all X ∈ M1×m(ℝ), then P = 0.

II : If m = n, P is symmetric and P2 = 0, then P = 0.

Then

(A)  both I and II are true

(B)  I is true but II is false

(C)  I is false but II is true

(D)  both I and II are false

Answer: (A)

29. For n ∈ ℕ, let Tn : (l1, ||∙||1) → (l, ||∙||) and T : (l1, ||∙||) be the bounded linear operators defined by

and

T(x1, x2, …) = (x1, x2 …).

Then

(A)  ||Tn|| does not converge to ||T|| as n → ∞

(B)  ||Tn – T|| converges to zero as n → ∞

(C)  for all x ∈ l1, ||Tn(x) – T(x)|| converges to zero as n → ∞

(D)  for each non-zero x ∈ l1, there exists a continuous linear functional g on l such that g(Tn(x)) does not converge to g(T(x)) as n → ∞

Answer: (C)

30. Let P(ℝ) denote the power set of ℝ, equipped with the metric

where χU and χ­V denote the characteristic functions of the subjects U and V, respectively of ℝ. The set {{m} : m ∈ ℤ} in the metric space (P(ℝ), d) is

(A)  bounded but not totally bounded

(B)  totally bounded but not compact

(C)  compact

(D)  not bounded

Answer: (A)

31. Let f : ℝ → ℝ be defined by

where χ(n n+1] is the characteristic function of the interval (n, n + 1]. for α ∈ ℝ, let Sα  = {x ∈ ℝ : f(x) > α}. Then

(A)  S1/2 is open

(B)  S5/2 is not measurable

(C)  S0 is closed

(D)  S1/3 is measurable

Answer: (D)

32. For n ∈ ℕ, let fn, gn­ : (0, 1) → ℝ be functions defined by fn(x) = xn and gn(x) = xn (1 – x).

Then

(A)  {fn} converges uniformly but {gn} does not converge uniformly

(B)  {gn} converges uniformly but {fn} does not converge  uniformly

(C)  both {fn} and {gn} converge uniformly

(D)  neither {fn} nor{gn} converge uniformly

Answer: (B)

33. Let u be a solution of the differential equation y′ + xy = 0 and let ϕ = uψ be a solution of the differential equation y′′ + 2xy′ + (x2 + 2) y = 0 satisfying ϕ(0) = 1 and ϕ′(0) = 0. Then ϕ(x) is

Answer: (B)

34. For n ∈ ℕ ⋃ {0}, let yn be a solution of the differential equation xy′′ + (1 – x)y′ + ny = 0 satisfying yn(0) = 1. For which of the following functions w(x), the integral  is equal to zero?

Answer: (B)

35. Suppose that

are metric spaces with metrics induced by the Euclidean metric of ℝ2. Let Band BY be the open unit balls around (0 0) in X and Y, respectively. Consider the following statements:

I : The closure of B­X in X is compact.

II : The closure of BY is compact.

Then

(A)  both I and II are true

(B)  I is true but II is false

(C)  I is false but II is true

(D)  both I and II are false

Answer: (C)

36. If f : ℂ \ {0} → ℂ is a function such that  and its restriction to the unit circle is continuous, then

(A)  f is continuous but not necessarily analytic

(B)  f is analytic but not necessarily a constant function

(C)  f is a constant function

(D) 

Answer: (A)

37. For a subset S of a topological space, let Int(S) and  denote the interior and closure of S, respectively. Then which of the following statements is TRUE?

(A)  If S is open, then 

(B)  If the boundary of S is empty, then S is open

(C)  If the boundary of S is empty, then S is open

(D)  If  is a proper subset of the boundary of S, then S is open

Answer: (B)

38. Suppose ℑ1, ℑ2, and ℑ3 are the smallest topologies on ℝ containing S1, S2 and S3, respectively, where

Then

(A)  ℑ3 ⊋ ℑ1

(B)  ℑ3 ⊋ ℑ2

(C)  ℑ1 = ℑ2

(D)  ℑ1 ⊋ ℑ2

Answer: (C)

39. Let  Consider the following statements:

I: There exists a lower triangular matrix L such that M = LLt, where Lt denotes transpose of L.

II : Gauss-Seidel method of Mx = b (b ∈ ℝ3) converges for any initial choice x0 ∈ ℝ3.

Then

(A)  I is not true when α > 9/2, β = 3

(B)  II is not true when α > 9/2, β =−1

(C)  II is not true when α = 4, β = 3/2

(D)  I is true when α = 5, β = 3

Answer: (D)

40. Let I and J be the ideals generated by {5, √10} and {4, √10} in the ring ℤ[√10] = {a+ b√10|a, b ∈ ℤ}, respectively. Then

(A)  both I and J are maximal ideals

(B)  I is a maximal ideal but J is not a prime ideal

(C)  I is not a maximal ideal but J is a prime ideal

(D)  neither I nor J is a maximal ideal

Answer: (B)

41. Suppose V is finite dimensional vector space over ℝ. If W1, W2 and W3 are subspaces of V, then which of the following statements is TRUE?

(A)  If W1  + W2 + W3 = V then span (W1 ⋃ W2) ⋃ span(W2 ⋃ W3) ⋃ span(W3 ⋃ W1) = V

(B)  If W1 ⋂ W2 = {0} and W1 ⋂ W3 = {0}, then W1 ⋂ (W2 + W3) = {0}

(C)  If W1 + W2 = W1 + W3, then W2 =  W3

(D)  If W1 ≠ V, then span(V\W1) = V

Answer: (D)

42. Let α, β ∈ ℝ, α ≠ The system

x1 – 2x2 + αx3 = 8

x1 – x2 + x4 = β

x1, x2, x­3, x4 ≥ 0

has NO basic feasible solution if

(A)  α < 0, β > 8

(B)  α > 0, 0 < β < 8

(C)  α > 0, β < 0

(D)  α < 0, β < 8

Answer: (D)

43. Let 0 < p < 1 and let

Then

(A)  |∙| defines a norm on X

(B)  |f + g|p ≤ |f|p + |g|p for all f, g ∈ X

(C)  |f +g|pp ≤ |f|pp + |g|pp for all, f, g ∈ X

(D)  if fn converges to f pointwise on ℝ, then 

Answer: (C)

44. Suppose the ϕ1 and ϕ2 are linearly independent solutions of the differential equation 2x2y′′ – (x + x2)y′ + (x2 – 2)y = 0, and ϕ1(0) = 0. Then the smallest positive integer n such that  is _______

Answer: (3 to 3)

45. Suppose that  z ∈ ℂ and γ(t) = e2it, t ∈[0, 2π]. If  then the value of α is equal to ________

Answer: (56 to 56)

46. If  t ∈ [0, 2] and  then β is equal to _______ (correct up to one decimal place)

Answer: (0.5 to 0.5)

47. Let  where ω is a primitive cube root of unity. Then the degree of extension of K over ℚ is _______

Answer: (4 to 4)

48. Let α ∈ ℝ. If (3, 0, 0, β) is an solution of the linear programming problem minimize x1 + x2 + x3 – αx4 subject to

2x1 – x2 + x3 = 6

−x1 + x2 + x4 = 3

x1, x2, x3, x4 ≥ 0

then the maximum value of β – α is _________

Answer: (7 to 7)

49. Suppose that T : ℝ4 → ℝ[x] is a linear transformation over ℝ satisfying T(−1, 1, 1, 1) = x2 + 2x4, T(1, 2, 3, 4) = 1 – x2, T(2, −1, −1, 0) = x3 – x4. Then the coefficient of x4 in T(−3, 5, 6, 6) is ________

Answer: (5 to 5)

50. Let and let S be the surface of the tetrahedron bounded by the planes x = 0, y = 0, z = 0 and x + y + z = 1.If is the unit outward normal to the tetrahedron, then the value of   is _______ (rounded off to two decimal places)

Answer: (1.30 to 1.40)

51. Let  and let S be the surface x2 + y2 + z = 1, z ≥ If  is a unit normal to S and  Then α is equal to _____

Answer: (2 to 2)

52. Let G be a non-cyclic group of order 57. Then the number of elements of order 3 in G is _______

Answer: (38 to 38)

53. The coefficient of (x – 1)5 in the Taylor expansion about x = 1 of the function  is _______ (correct up to two decimal places)

Answer: (0.04 to 0.04)

54. Let u(x, y) be the solution of the initial value problem  u(x, 0) = 1 + x2. Then the value of u(0, 1) is ______ (rounded off to three decimal places

Answer: (1.610 to 1.625)

55. The value of  is _______ (rounded off to three decimal places)

Answer: (2.710 to 2.725)

GATE Exam 2020 Instrumentation Engineering (IN) Question Paper With Answer Key

GATE-2020

IN: Instrumentation Engineering

GA-General Aptitude

Q1 – Q5 carry one mark each.

1. He is known for his unscrupulous ways. He always sheds ______ tears to deceive people.

(A)  fox’s

(B)  crocodile’s

(C)  crocodile

(D)  fox

Answer: (C)

2. Jofra Archer, the England fast bowler, is _____ than accurate.

(A)  more fast

(B)  faster

(C)  less fast

(D)  more faster

Answer: (A)

3. Select the word that fits the analogy:

Build : Building :: Grow :______

(A)  Grown

(B)  Grew

(C)  Growth

(D)  Growed

Answer: (C)

4. I do not think you know the case well enough to have opinions. Having said that, I agree with your other point.

What does the phrase “having said that” mean in the given text?

(A)  as opposed to what I have said

(B)  despite what I have said

(C)  in addition to what I have said

(D)  contrary to what I have said

Answer: (B)

5. Define [x] as the greatest integer less than or equal to x, for each x ∈ (−∞, ∞). If y = [x], then are under y for x ∈ [1, 4] is _____.

(A)  1

(B)  3

(C)  4

(D)  6

Answer: (D)

Q6 – Q10 carry two marks each.

6. Crowd funding deals with mobilization of funds for a project from a large number of people, who would be willing to invest smaller amounts through web-based platforms in the project.

Based on the above paragraph, which of the following is correct about crowd funding?

(A)  Funds raised through unwilling contributions on web-based platforms.

(B)  Funds raised through large contributions on web-based platforms.

(C)  Funds raised through coerced contributions on web-based platforms.

(D)  Funds raised through voluntary contributions on web-based platforms.

Answer: (D)

7. P, Q, R and S are to the uniquely coded using α and β. If P is coded as αα and Q as αβ, then R and S, respectively, can be coded as _______.

(A)  βα and αβ

(B)  ββ and αα

(C)  αβ and ββ

(D)  βα and ββ

Answer: (D)

8. The sum of the first n terms in the sequence, 8, 88, 888, …. is _______.

(A) 

(B) 

(C) 

(D) 

Answer: (D)

9. Select the graph that schematically represents BOTH y = xm and y = x1/m properly in the interval 0 ≤ x ≤ 1, for integer values of m, where m > 1.

Answer: (A)

10. The bar graph shows the data of the students who appeared and passed in an examination for four schools P, Q, R and S. Te average of success rates (in percentage) of these four schools is _______.

(A)  58.5%

(B)  58.8%

(C)  59.0%

(D)  59.3%

Answer: (C)

IN: Instrumentation Engineering

Q1 – Q25 carry one mark each.

1. The unit vectors along the mutually perpendicular x, y and z axes are,  Consider the plane z = 0 and two vectors  on that plane such that  for any scalar α. A vector perpendicular to both  is ______.

(A) 

(B) 

(C) 

(D) 

Answer: (A)

2. Consider the recursive equation Xn+1 = Xn – h(F(Xn) – Xn), with initial condition X0 = 1 and h > 0 being a very small valued scalar. This recursion numerically solves the ordinary differential equation_____

(A)  Ẋ = −F(X), X(0) = 1

(B)  Ẋ = −F(X) + X, X(0) = 1

(C)  Ẋ = F(X), X(0) =1

(D)  Ẋ = F(X) + X, X(0) = 1

Answer: (B)

3. A set of linear equations is given in the form Ax = b, where A is a 2 × 4 matrix with real number entries and b ≠ Will it be possible to solve for x and obtain a unique solution by multiplying both left and right sides of the equation by AT (the super script T denotes the transpose) and inverting the matrix ATA? Answer is _______

(A)  Yes, it is always possible to get a unique solution for any 2 × 4 matrix A.

(B)  No, it is not possible to get a unique solution for any 2 × 4 matrix A.

(C)  Yes, can obtain a unique solution provided the matrix ATA is well conditioned

(D)  Yes, can obtain a unique solution provided the matrix A is well conditioned

Answer: (B)

4. In the circuit shown below, the safe maximum value for the current I is ______

(A)  1.0 A

(B)  0.5 A

(C)  0.1 A

(D)  0.05 A

Answer: (C)

5. A differentiator has a transfer function whose

(A)  phase increases linearly with frequency

(B)  magnitude remains constant

(C)  magnitude increases linearly with frequency

(D)  magnitude decreases linearly with frequency

Answer: (C)

6. A phase lead network has the transfer function  The angular frequency at which the maximum phase shift for the network occurs is _____

(A)  10 rad/s

(B)  20 rad/s

(C)  100 rad/s

(D)  200 rad/s

Answer: (A)

7. If the diodes in the circuit shown are ideal and the breakdown voltage VZ of the Zener diode is 5 V, the power dissipated in the 100 Ω resistor (in watts) is _______

(A)  0

(B)  1

(C)  25/100

(D)  225/100

Answer: (A)

8. Given f(A, B, C, D) = ∑m(0, 1, 2, 6, 8, 9, 10, 11) + ∑d(3, 7, 14, 15) is a Boolean function, where m represents min-terms and d represents don’t-cares. The minimal sum of products expression for f is______

(A) 

(B) 

(C) 

(D) 

Answer: (B)

9. A Q meter is best suited for the measurement of the ______

(A)  Quality factor of a capacitance.

(B)  Distributed capacitance of a coil.

(C)  Quality factor of piezoelectric sensor.

(D)  Turns-ratio of a transformer

Answer: (B)

10. If I is the current flowing through a Hall effect sensor and B is the magnetic flux density perpendicular to the direction of the current (in the plane of the Hall effect sensor), the Hall voltage generated is______

(A)  Directly proportional to I and inversely proportional to B

(B)  Directly proportional to both I and B

(C)  Inversely proportional to both I and B

(D)  Inversely proportional to I and directly proportional to B

Answer: (B)

11. The Boolean expression for the shaded regions as shown in the figure is _______

(A) 

(B) 

(C) 

(D) 

Answer: (A)

12. The Boolea operation performed by the following circuit at the output O is ______

(A)  O = S1 ⊕ S0

(B)  

(C)  O = S1 + S0

(D) 

Answer: (A)

13. Consider the Signal x[n] = sin(2πn) u[n], where  The period of this signal x[n] is _____

(A)  4

(B)  3

(C)  2

(D)  1

Answer: (D)

14. The closed loop transfer function of a control system is given by  For the input r(t) = sin t, the steady state response c(t) is _____

(A)  1

(B) 

(C) 

(D) 

Answer: (D)

15. Let  The value of the integral ∮f(z) dz over a circle C with center (−a, 0) and radius R > 0 evaluated in the anti-clockwise direction is ____

(A)  0

(B)  2πi

(C)  −2πi

(D)  4πi

Answer: (B)

16. A player throws a ball at a basket kept at a distance. The probability that the ball falls into the basket in a single attempt is 0.1. The player attempts to throw the ball twice. Considering each attempt to be independent, the probability that this player puts the ball into the basket only in the second attempt (rounded off to two decimal places) is ____

Answer: (0.09 to 0.09)

17. Assuming ideal opamps, the output voltage at V1 in the figure shown (in volts) is ______

Answer: (7 to 7)

18. Three 400 Ω resistors are connected in delta and powered by a 400 V (rms), 50 Hz, balanced, symmetrical R-Y-B sequence, three-phase three-wire mains. The rms value of the line current (in amperes, rounded off to one decimal place) is _______

Answer: (1.7 to 1.8)

19. Consider the signal x(t) = e|t|. Let  be the Fourier transform of x(t). The value of X(j0) is _____

Answer: (2 to 2)

20. A second order system has closed loop poles located at s = −3 ± The time t at which the maximum value of the step response occurs (in seconds, rounded off to two decimal places)

Answer: (0.78 to 0.79)

21. Assume that the opamp in the circuit shown is ideal.

The value of Vx/Ix (in kΩ) is ____

Answer: (-4 to -4)

22. A sinusoid of 10 kHz is sampled at 15 k samples/s. The resulting signal is passed through an ideal low pass filter (LPF) with cut-off frequency of 25 kHz. The maximum frequency component at the output of the LPF (in kHz) is _____

Answer: (25 to 25)

23. A 200 mV full-scale dual-slope analog to digital converter (DS-ADC) has a reference voltage of 100 mV. The first integration time is set as 100 ms. The DS-ADC is operated in the continuous conversion mode. The conversion time of the DS-ADC for an input voltage of 123.4 mV (in ms, rounded off to one decimal place) is______

Answer: (223.3 to 223.5)

24. The capacitance Cx of a capacitive type sensor is (1000 x) pF, where x is the input to the sensor. As shown in the figure, the sensor is excited by a voltage 10 sin (100 πt) V. The other terminal of the sensor is tied to the input of a high input impedance amplifier through a shielded cable, with shield connected to ground. The cable capacitance is 100 pF. The peak of the voltage V­A at the input of the amplifier when x = 0.1 (in volts) is

Answer: (5 to 5)

25. Two 100 Ω resistors having tolerance 3% and 4% are connected in series. The effective tolerance of the series combination (in n% rounded off to one decimal place) is ______

Answer: (2.5 to 2.5 OR 3.5 to 3.5)

Q26 – Q55 carry two marks each.

26. Consider the matrix  One of the eigenvectors of M is

(A) 

(B) 

(C) 

(D) 

Answer: (D)

27. Consider the differential equation  with the initial condition x(0) = 0. The solution to this ordinary differential equation is _______

(A)  x(t) = 0

(B)  x(t) = sin(t)

(C)  x(t) = cos(t)

(D)  x(t) = sin(t) – cos(t)

Answer: (A)

28. A straight line drawn on an x-y plane intercepts the x-axis at −5 and the y-axis at 1. The equation that describes this line is _____

(A)  y = −0.5x + 1

(B)  y = x – 0.5

(C)  y = 0.5x – 1

(D)  y = 2x + 1

Answer: (D)

29. The loop transfer function of a negative feedback system is  The Nyquist plot for the above system____

(A)  encircles (−1 + j0) point once in the clockwise direction

(B)  encircles (−1 + j0) point once in the counterclockwise direction

(C)  does not encircle (−1 +  j0) point

(D)  encircles (−1 + j0) point twice in the counterclockwise direction

Answer: (A)

30. I1, I2, I3 in the figure below are mesh currents. The correct set of mesh equations for these currents, in matrix form, is ______

Answer: (A)

31. Consider the function f(x, y) = x2 + y2. The minimum value the function attains on the line x + y = 1 (rounded off to one decimal place) is ______

Answer: (0.5 to 0.5)

32. Consider two identical gas B1 and B2 each containing 10 balls o identical shapes and sizes. Bag B1 contains 7 Red and 3 Green balls, while bag B2 contains 3 Red and 7 Green balls. A bag is picked at random and a ball is drawn from it, which was found to be Red. The probability that the Red ball came from bag B1 (rounded off to ne decimal place) is ______

Answer: (0.7 to 0.7)

33. The rms of the phasor current I in the circuit shown (in amperes) is ______

Answer: (1 to 1)

34. In the circuit shown, the rms value of the voltage across the 100 Ω resistor (in volts) is _____

Answer: (115 to 116)

35. Let  and   Consider y[n] = h[n] ⊗ g[n], where ⊗ denotes the convolution operator. The value of y[2] is ______

Answer: (0 to 0)

36. The loop transfer function of a negative feedback system is given by  where K > 0. The value of K at the breakaway point of the root locus for the above system (rounded off to one decimal place) is ______

Answer: (5.0 to 5.1)

37. The system shown in Fig. (a) has a time response y(t) to an input r(t) = 10 u(t) as shown in Fig. (b), u(t) being the unit step input. Both K, τ are positive. The gain K of the system is ________

Answer: (4 to 4)

38. Assuming that the opamp used in the circuit shown in ideal, the reading of the 1 Hz bandwidth, permanent magnet moving coil (PMMC) type voltmeter (in volts) is ______

Answer: (1 to 1)

39. If the opamps in the circuit shown are ideal and Vx = 0.5 mV, the steady state value of VO (in volts, rounded off to two decimal places) is _____

Answer: (0.45 to 0.55)

40. Two T-flip flops are interconnected as shown in the figure. The present state of the flip flops are: A = 1, B = 1. The input x is given as 1, 0, 1 in the next three clock cycles. The decimal equivalent of (ABy)2 with A being the MSB and y being the LSB, after the 3rd clock cycle is ________

Answer: (7 to 7)

41. The address lines A9 …. A2 of a 10 bit, 1.023 V full-scale digital to analog converter (DAC) is connected to the data lines D7 to D0 of an 8-bit microprocessor, with A1 and A0 of the DAC grounded. Now, D7 … D0 is changed from 1010 1010 to 1010 1011. The corresponding change in the output of the DAC (in mV, rounded off to one decimal place) is _____

Answer: (3.5 to 4.5)

42. The real power drawn by a balanced load connected a 400 V, 50 Hz balanced, symmetrical 3-phase, 3-wire, RYB sequence mains is measured using the two-wattmeter method. Wattmeter W­1 is connected in the R line and wattmeter W2 is connected in the B line. The line current is measured as 1/√3 A. If the wattmeter W1 reads zero, the reading on W2(in watt) is ____

Answer: (199 to 201)

43. A 6½ digit timer-counter is set in the ‘time period’ mode of operation and the range is set as ‘ns’. For an input signal, the timer-counter displays 1000000. With the same input signal, the timer-counter is changed to ‘frequency’ mode of operation and the range is et as ‘Hz’. The display will show the number_____

Answer: (999 to 1001)

44. The circuit shown uses ideals opamp powered from a supply VCC = 5 V. If the charge qp generated by the piezoelectric sensor is o the form qp = 0.1 sin(10000πt) μC, the peak detector output after 10 cycles of qp (in volts, rounded off to one decimal place) is _____

Answer: (3.4 to 3.6)

45. A metallic strain gauge of resistance Rx with a gauge factor is 2 is bonded to a structure made of a metal with modulus of elasticity of 200 GN/m2. The value of R­x is 1 kΩ when no stress is applied. Rx is a part of quarter bridge with three identical fixed resistors of 1 kΩ The bridge is excited from a DC voltage of 4 V. The structure is subjected to a stress of 100 MN/m2. Magnitude of the output of the bridge (in mV, rounded off to two decimal places is _____

Answer: (0.95 to 1.05)

46. A laser beam of 10 mm beam diameter is focused onto an optical fibre using a thin biconvex lens as shown in the figure. The refractive index of the lens is 1.5. The refractive indices of the core and cladding of the fibre are 1.55 and 1.54 respectively. The minimum value of the focal length of the lens to attain the maximum coupling to the fibre (in mm, rounded off to one decimal place) is ______

Answer: (27.5 to 28.5)

47. As shown in the figure, a slab of finite thickness t with refractive index n2 = 1 .5, has air (n1 = 1) above and below it. Light of free space wavelength 600 nm is incident normally from air as shown. For a destructive interference to be observed at R, the minimum value of thickness of the slab t (in mm) is _____

Answer: (200 to 200)

48. Consider the finite sequence X = (1, 1, 1). The Inverse Discrete Fourier Transform (IDFT) of X is given as (x90), x(1), x(2)). The value of x(2) is ______

Answer: (0 to 0)

49. A circuit consisting of capacitors, DC voltage source and an amplifier having a voltage gain G = −5 is shown in the figure. The effective capacitance across the nodes A and B (in μF, rounded off to one decimal place) is _____

Answer: (14.5 to 15.0)

50. Consider the following state variable equations:

The initial conditions are x1(0) = 0 and x2(0) = 1. At t = 1 second, the value of x2(1) (rounded off to two decimal places) is ______

Answer: (-0.13 to -0.11)

51. Assume the diodes in the circuit shown are ideal. The current Ix flowing through the 3 kΩ resistor (in mA, rounded off to one decimal place) is ____

Answer: (1.8 to 1.8)

52. A 1000/1 A, 5VA, UPF bar-primary measuring current transformer has 1000 secondary turns. The current transformer exhibits a ratio error of −1% and a phase error of 3.438 minutes when the primary current is 1000 A. At this operating condition, the rms value of the magnetization current of the current transformer (in amperes, rounded off to two decimal places) is ____

Answer: (0.95 to 1.05)

53. The mutual inductances between the primary coil and the secondary coils of a linear variable differential transformer (LVDT) shown in the figure are M1 and M2. Assume that the self-inductances LS1 and LS2 remain constant and are independent of x. When x = 0, M1 = M2 = M0. When x is in the range ±10 mm, M1 and M2 change linearly with x. At x = +10 mm or −10 mm, the change in the magnitudes of M1 and M2 is 0.25 M0. For a particular displacement x =  D, the voltage across the detector becomes zero when |V2| = 1.25|V1|. The value of D (in mm, rounded off to one decimal place) is _____

Answer: (4.3 to 4.6)

54. In the Maxwell-Wien bridge shown, the detector D reads zero when C1 = 100 nF and R1 = 100 kΩ. The Q factor of the coil is____

Answer: (10 to 10)

55. The loop transfer function of a negative feedback system is  The phase margin of the system (in degrees, rounded off to one decimal place) is ____

Answer: (65.4 to 65.6)

GATE Exam 2020 Geology and Geophysics (GG) Question Paper With Answer Key

GATE-2020

GG-Geology  & Geophysics

GA: General Aptitude

Q 1 – Q5 Carry one mark each.

1. The untimely loss of life is a cause of serious global concern as thousands of people get killed ______ accidents every year while many other die ______ diseases like cardio vascular disease, cancer, etc.

(A)    in, of

(B)    from, of

(C)    during, from

(D)    from, from

Answer: (A)

2. He was not only accused of theft ______ of conspiracy.

(A)    rather

(B)    but also

(C)    but even

(D)    rather than

Answer: (B)

3. Select the word that fits the analogy:

Explicit : Implicit : : Express: _______

(A)    Impress

(B)    Repress

(C)    Compress

(D)    Suppress

Answer: (B)

4. The Canadian constitution requires that equal importance be given to English and French. Last year, Air Canada lost a lawsuit, and had to pay a six-figure fine to a French-speaking couple after they filed complaints about formal in-flight announcements is English lasting 15 seconds, as opposed to informal 5 second messages in French.

The French-speaking couple were  upset at ______.

(A)    the in-flight announcements being made in English.

(B)    the English announcements being clearer than the French ones.

(C)    the English announcements being longer than the French ones.

(D)    equal importance being given to English and French.

Answer: (C)

5. A superadditive function f (∙) satisfies the following property

f(x1 + x2) ≥ f(x1) + f(x2)

Which of the following functions is a superadditive function for x > 1?

(A)    ex

(B)    √x

(C)    1/x

(D)    ex

Answer: (A)

Q6 – Q10 carry two marks each.

6. The global financial crisis in 2008 is considered to be the most serious world-wide financial crisis, which started with the sub-prime lending crisis in USA in 2007. The sub-prime lending crisis led to the banking crisis in 2008 with the collapse of Lehman Brothers may have difficulties in repaying loans, and it arises because of excess liquidity following the East Asian crisis.

Which one of the following sequences shows the correct precedence as per the given passage?

(A)    East Asian crisis → subprime lending crisis → banking crisis global financial crisis.

(B)    Subprime lending crisis → global financial crisis → banking crisis → East Asian crisis.

(C)    Banking crisis → subprime lending crisis → global financial crisis → East Asian crisis.

(D)    Global financial crisis → East Asian crisis → Banking crisis → subprime lending crisis.

Answer: (A)

7. It is quarter past three in your watch. The angle between the hour hand and the minute hand is ________.

(A)    0°

(B)    7.5°

(C)    15°

(D)    22.5°

Answer: (B)

8. A circle with centre O is shown in the figure. A rectangle PQRS of maximum possible area is inscribed in the circle. If the radius of the circle is a, then the area of the shaded portion is ______.

(A)    πa2 – a2

(B)    πa2 – √2a2

(C)    πa2 – 2a2

(D)    πa2 – 3a2

Answer: (C)

9. a, b, c are real numbers. The quadratic equation ax2 – bx + c = 0 has equal roots, which is β, then

(A)    β = b/a

(B)    β2 = ac

(C)    β3 = bc/(2a2)

(D)    b2 ≠ 4ac

Answer: (C)

10. The following figure shows the data of students enrolled in 5 years (2014 to 2018) for two schools P and Q. During this period, the ratio of the average number of the students enrolled in school P to the average of the difference of the number of students enrolled in schools P and Q is ______ .

(A)    8 : 23

(B)    23 : 8

(C)    23 : 31

(D)    31 : 23

Answer: (B)

Part A- Compulsory Section For All Candidates

Q1 – Q25 carry cone mark each.

1. A plagioclase with  is

(A)  albite

(B)  anorthite

(C)  oligoclase

(D)  bytownite

Answer: (C)

2. Tillite is an important constituent of the

(A)  Talchir Formation

(B)  Barakar Formation

(C)  Pachmarhi Formation

(D)  Lameta Formation

Answer: (A)

3. If the ratio of gravity to total magnetic field at the equator of t he Earth is X, then the ratio of gravity to total magnetic field at the pole of the Earth will be close to

(A)  2X

(B)  X/2

(C)  4X

(D)  X/8

Answer: (B)

4. Which of the following is NOT a point group?

(A)  222

(B)  422

(C)  432

(D)  632

Answer: (D)

5. Mississippian is an Epoch within the

(A)  Permian Period

(B)  Carboniferous Period

(C)  Triassic Period

(D)  Jurassic Period

Answer: (B)

6. The given stereoplot of the axial plane and the axis of a fold represents an/a

(A)  upright fold

(B)  vertical fold

(C)  reclined fold

(D)  recumbent fold

Answer: (D)

7. A siliciclastic sedimentary rock with <5% matrix and QFL composition of 60% quartz, 30% rock fragments and 10% feldspar, is called

(A)  quartz wacke

(B)  lithic arenite

(C)  quartz arenite

(D)  feldspathic wacke

Answer: (B)

8. Which one of the following pairs of geophysical methods is most suitable to delineate chromite ore deposits occurring at a shallow depth in a granitic terrain?

(A)  Gravity and Electrical methods

(B)  Electrical and Electromagnetic methods

(C)  Seismic and Gravity methods

(D)  Seismic and Magnetic methods

Answer: (A)

9. The ratio of bridging to non-bridging oxygen atoms is zero in case of

(A)  nesosilicates

(B)  inosilicates

(C)  phyllosilicates

(D)  tectosilicates

Answer: (A)

10. Lahar is geomorphic feature associated with

(A)  wind activity

(B)  river activity

(C)  glacial activity

(D)  volcanic activity

Answer: (D)

11. Kepler’s second law of planetary motion follows the principle of conservation of

(A)  energy

(B)  momentum

(C)  angular momentum

(D)  moment of inertia

Answer: (C)

12. Which one of the following options shows the internal structural units of the Earth arranged in the CORRECT sequence of increasing volume?

(A)  Outer core < Inner core < Upper mantle < Lower mantle

(B)  Outer core < Inner core < lower mantle < Upper mantle

(C)  Inner core < Outer core < Upper mantle < Lower mantle

(D)  Inner core < Outer core < Lower mantle < Upper mantle

Answer: (C)

13. Which one of the following is NOT an earthquake intensity scale?

(A)  Richter scale

(B)  JMA scale

(C)  Modified Mercalli scale

(D)  Rossi-Forel scale

Answer: (A)

14. The dimension of transmissivity of an aquifer is

(A)  M0L1T1

(B)  M0L0T0

(C)  M1L1T2

(D)  M0L2T1

Answer: (D)

15. During ‘K-capture’ nuclear transmutation process

(A)  both atomic number atomic mass increase

(B)  atomic number decreases but atomic mass remains the same

(C)  atomic number increases but atomic mass remains the same

(D)  both atomic number and atomic mass decrease

Answer: (B)

16. Which one amongst the following logs has the maximum depth of investigation?

(A)  Neutron log

(B)  Natural Gamma-ray log

(C)  Lateral log

(D)  Density log

Answer: (C)

17. The scale factor of an aerial photo of a planar ground surface, taken vertically downwards by a camera with a focal length of 300 mm, from a flying height of 3000 m is ______.

Answer: (10000 to 10000 OR 0.0001 to 0.0001)

18. In a soil sample, specific gravity of soil particles is 2.5 and the void ratio is 0.5. The density of the soil sample when it is fully saturated with water is _______ kg/m3. (Assume density of water = 1000 kg/m3, and no volume change of the soil sample with saturation)

Answer: (2000 to 2000)

19. Nuclide A decays to nuclide B exclusively through α and β decay, such that the mass number is reduced by 32 and the atomic number is reduced by 10. The number of β particles emitted during the decay of nuclide A to nuclide B is ________.

Answer: (6 to 6)

20. A cylindrical specimen (diameter = 54.7 mm; length = 110 mm) of basalt shows linear elastic behavior under uniaxial compression. At an axial stress of 100 Mega-Pascal (MPa), the absolute value of the measured axial strain is 0.2%. The Young’s modulus is calculated to be __________ Giga-Pascal (GPa).

Answer: (50 to 50)

21. A Mid-Oceanic-Ridge has symmetric magnetic anomalies about the ridge axis as shown below. Using the information given in the figure, the average relative velocity between the Plates A and B is calculated to be _______ cm/year.

Answer: (10 to 10)

22. The transmission coefficient for the vertically incident seismic wave at the interface between Layer and Layer 2 given in the figure is __________. (Round off to 2 decimal places)

Answer: (0.71 to 0.81)

23. The ‘geometrical factor’ for the electrode configuration given below will be ________ m. (Round off to 2 decimal places) (Use π = 3.14)

(C1 and C2 are current electrodes; P1 and P2 are potential electrodes)

Answer: (186.00 to 192.00)

24. In an electromagnetic measurement, the resultant field shows a phase lag of 30° with respect to the primary field at the receiver coil. The ratio of Inphase to Quadrature component of the resultant field is _________. (Round off to 2 decimal places)

Answer: (1.70 to 1.75)

25. A 4 km-high plateau is isostatically compensated as shown in the figure. Assuming Pratt’s hypothesis of isostasy, the calculated density of the plateau is ________ kg/m3.

Answer: (2450 to 2450)

Part B (Section 1): For Geology Candidates Only

Q1 – Q30 carry two marks each.

1. “Point Group” in crystallography is characterized by a set of symmetry operations such that

(A)  all points in a crystal are affected by it

(B)  no point in a crystal is affected by bit

(C)  at least one point in a crystal is affected by it

(D)  at least one point in a crystal is unaffected by it

Answer: (D)

2. What are the Miller indices of a plane that intercepts each of the crystallographic axes X, Y and Z at 20 Å? (Assume a primitive unit-cell with the dimensions a = 5 Å, b = 2 Å and c = Å.)

(A)  (111)

(B)  (524)

(C)  (425)

(D)  (542)

Answer: (B)

3. Which one of the following processes is associated with the emission of X-rays?

(A)  alpha decay

(B)  beta decay

(C)  electron capture decay

(D)  positron decay

Answer: (C)

4. Which one of the following radioisotopes has the longest half-life?

(A)  87Rb

(B)  147Sm

(C)  232Th

(D)  238U

Answer: (B)

5. The given geological map represents

(A)  culmination of an antiformal anticline

(B)  culmination of an antiformal syncline

(C)  depression of a synformal anticline

(D)  culmination of a synformal syncline

Answer: (MTA)

6. On a fault plane, the net slip is parallel to the bedding trace. Then, the apparent movement will be recognizable

(A)  both in horizontal and vertical sections

(B)  in horizontal, but not in vertical section

(C)  in vertical, but not in horizontal section

(D)  neither in horizontal nor in vertical section

Answer: (D)

7. The CORRECT sequence of the given electromagnetic radiations in order of increasing wavelength is

(A)  Ultraviolet < Gamma Rays < Radiowave < Near-Infrared

(B)  Gamma Rays < Ultraviolet < Near-Infrared < Radiowave

(C)  Gamma Rays < Radiowave < Ultraviolet < Near-Infrared

(D)  Ultraviolet < Radiowave < Near-Infrared < Gamma Rays

Answer: (B)

8. Choose the CORRECT combination of foraminiferal tests and types of coiling.

(A)  Test 1 – Trochospiral, Test 2 – Planispiral, Test 3 – Milioline

(B)  Test 1 – Milioline, Test 2 – Planispiral, Test 3 – Trochospiral

(C)  Test 1 – Milioline, Test 2 – Trochospiral, Test 3 – Planispiral

(D)  Test 1 – Trochospiral, Test 2 – Milioline, Test 3 – Planispiral

Answer: (B)

9. The figure below represents an isobaric binary liquidus phase diagram, with the solid phases A, B and C. What are the degrees of freedom associated with equilibrium phase assemblages represented by the bulk compositions w, x, y and z, in the fields indicated in the figure?

(A)  w  = 2, x = 1, y = 1, z = 1

(B)  w = 2, x = 1, y = 0, z = 2

(C)  w = 1, x = 1, y = 0, z = 1

(D)  w = 1, x = 1, y = 1, z = 2

Answer: (A)

10. Match the basins (Group I) with the corresponding stratigraphic units (Group II).

(A)  P-3, Q-4, R-1, S-2

(B)  P-2, Q-4, R-1, S-3

(C)  P-3, Q-1, R-4, S-2

(D)  P-2, Q-3, R-4, S-1

Answer: (A)

11. In the metamorphic reaction Quartz + Muscovite = X + Sillimanite + Water, ‘X’ represents

(A)  Garnet

(B)  Staurolite

(C)  Orthoclase

(D)  Cordierite

Answer: (C)

12. The talc-kyanite assemblage can stabilize in

(A)  greenschist facies marly rocks

(B)  amphibolites facies mafic rocks

(C)  eclogite facies politic rocks

(D)  sanidinite facies ultramafic rocks

Answer: (C)

13. Which one of the following statements about igneous rocks is CORRECT ?

(A)  Tholeiitic and calc-alkaline rocks are both alkaline in nature.

(B)  Tholeiitic rocks are subalkline, but calc-alkaline rocks are alkaline in nature.

(C)  Tholeiitic rocks are subalkaline, but calc-alkaline rocks are alkaline in nature.

(D)  Tholeiitic and calc-alkaline rocks are both subalkaline in nature.

Answer: (D)

14. Based on the three statements given below, choose the CORRECT option.

Statement I: Barchans are crescent-shaped dunes that close in the downwind direction.

Statement II : Parabolic dunes are U-shaped dunes that close in the downwind direction.

Statement III: Barchanoid dunes are sinuous transverse ridges, the crestline sinuousity of successive bed forms are either in-phase or out-phase.

(A)  All the statements are correct

(B)  Statement I is correct, but statements II and III are incorrect

(C)  Statements I and II are correct, but statement III is incorrect

(D)  Statements II and III are correct, but statement I is incorrect

Answer: (D)

15. Based on the three statements given below, choose the CORRECT option.

Statement I: Barapasaurus is  known from the Jurassic Kota Formation.

Statement II: Morganucodon is known from the Tatrot Formation.

Statement III: Lystrosaurus is known from the Lameta Formation.

(A)  All the three statements are correct

(B)  Statement I is correct but statements II and III are incorrect

(C)  Statements I and II are correct but statement III is incorrect

(D)  Statements II and III are correct  but statement I is correct

Answer: (B)

16. Which one of the following assemblages of plant fossils is known from the Barakar Formation?

(A)  Glossopteris, Gangamopteris, Dicroidium

(B)  Glossopteris, Gangamopteris, Noeggerathiopsis

(C)  Glossopteris, Gangamopteris, Ptilophyllum

(D)  Schizoneura, Noeggerathiopsis, Ptilophyllum

Answer: (B)

17. Match the features (Group I) with the corresponding invertebrate genera (Group II).

(A)  P-3, Q-4, R-1, S-2

(B)  P-3, Q-4, R-2, S-1

(C)  P-4, Q-3, R-2, S-1

(D)  P-2, Q-1, R-4, S-3

Answer: (B)

18. If the orthogonal thickness is constant along a folded layer, as per Ramsay’s morphological classification of folds, it is a

(A)  Class IA fold

(B)  Class IB fold

(C)  Class 2 fold

(D)  Class 3 fold

Answer: (B)

19. If density of quartz is 2650 kg/m3 and that of orthoclase is 2550 kg/m3, the lithostatic pressure due to granite with 68 modal % orthoclase at a depth of 10 km will be _______ kbar. (Round off to 2 decimal places) (Acceleration due to gravity g = 9.8 m/s2

Answer: (2.56 to 2.58)

20. The unit-cell of an orthorhombic mineral was compressed during deformation from 5 Å to 4.5 Å along the c-axis, with the other two dimensions remaining unaffected. The absolute value of the shift in the position of the (001) peak in its XRD pattern is _______ °2θ. (Round off to 3 decimal places) (Wavelength of X-ray used = 1.5418 Å. For orthorhombic system: 1/d2 = h2/a2 + k2/b2 + l2/c2).

Answer: (1.98 to 2.00)

21. The grade of iron in an ore body containing 80 wt. % hematite and 20 wt. % gangue is _______ %. (Round off to 2 decimal places) (Atomic wt. of Fe = 55.85, atomic weight of O = 16)

Answer: (55.90 to 55.97)

22. The abundances of the isotopes 35C­l (atomic mass = 34.96885 amu) and 37Cl (atomic mass = 36.96590 amu) are 75.77% and 24.23%, respectively. The calculated atomic weight of Cl is ______ amu. (Round off to 3 decimal places)

Answer: (35.451 to 35.454)

23. A vertical profile perpendicular to the rest line of an asymmetrical ripple is given in the figure. The calculated Ripple Index is ________.

Answer: (4 to 4)

24. A source rock undergoes melting. Assuming batch melting, 5% partial melting and bulk distribution coefficient of 0.045, the enrichment factor (C1/C0) of Rb in the melt will be _______. (Round off to 2 decimal places)

Answer: (10.75 to 10.87)

25. If the ∆H of formation of CaSiO3, SiO and CaO from Ca, Si and O are respectively −1635, −911 and −635 kJ/mol, the enthalpy of formation of CaSiO3 from CaO and SiO­2 is ________ kJ/mol.

Answer: (-89 to -89)

26. The tip-line of an actively propagating thrust fault is located at a depth of 1 km from the horizontal ground surface. The average density of the material from the ground surface to this depth is assumed to be uniform and can be taken as 2700 kg/m3. The rock at this depth follows the failure criterion given by the equation: σ1 = 10 MPa + 3σ­3, where σ1 and σ3 are the maximum and minimum principal stresses. Considering Anderson’s theory of faulting, the calculated maximum principal stress at this depth is _______ Mega-Pascal (MPa). (Assume the acceleration due to gravity (g) to be 10 m/s2.)

Answer: (91 to 91)

27. During a rockslide, a 20 kg granite block gets dislodged from the top of a planar hill slope and starts sliding down the slope as shown in the figure. The slope angle is 30° with the horizontal. After travelling a distance of 40 m in the same direction on the slope, the block hits the road. Assuming zero cohesion and zero friction, and considering acceleration due to gravity (g) as 10 m/s2, the velocity with which the block hits the road is ________ m/s.

Answer: (20 to 20)

28. Liquid limit and plastic limit of a soil are 40% and 20%, respectively. If the natural (i.e. in situ) water content of the soil is 30%. the liquidity index is ___________.

Answer: (0.5 to 0.5)

29. A confined aquifer has a uniform area (‘A’) perpendicular to the water flow. The hydraulic gradient and coefficient of permeability are given as 0.005 and 2 m/day, respectively. The total daily flow of water is 250 m3. Using Darcy’s law, the calculated value of ‘A’ is ________m2.

Answer: (25000 to 25000)

30. The apparent dip amount of a sandstone bed is 45°. The angle between the true dip direction and the apparent dip direction is 60°. The true dip amount of the bed is ________ degree (°). (Round off to 2 decimal places)

Answer: (63.42 to 63.44)

PART B (Section 2): Fro Geophysics Candidates only)

Q1 – Q30 carry two marks each.

1. International gravity formula is based on which one of the following models?

(A)  Non-rotating homogeneous spherical Earth model

(B)  Non-rotating homogenous oblate spheroidal Earth model

(C)  Rotating homogeneous oblate spheroidal Earth model

(D)  Rotating inhomogeneous spherical Earth model

Answer: (C)

2. Heat flow equation  is valid when

[T = Temperature, z is coordinate along z-axis]

(A)  steady state heat conduction is considered in an isotropic medium without heat source

(B)  steady state heat conduction is considered in an isotropic medium with heat source

(C)  steady state heat convection is considered in an isotropic medium without heat source

(D)  steady state heat convection is considered in an isotropic medium with heat source

Answer: (A)

3. Assuming the inner core of the Earth to be one-third of its present size, which one of the following statements is CORRECT? (Radius of the Earth and outer core remain unchanged)

(A)  Shadow zone of P-wave increases but that of S-wave decreases

(B)  Shadow zone of P-wave increases and that S-wave remains unchanged

(C)  Shadow zone of P-wave increases and that S-wave increases

(D)  Shadow zone of P-wave decreases but that of S-wave remains unchanged

Answer: (MTA)

4. Match the following instruments (Group I) with their corresponding physical principle (Group II)

(A)  P-4, Q-1, R-2, S-3

(B)  P-4, Q-3, R-2, S-1

(C)  P-3, Q-1, R-4, S-2

(D)  P-3, Q-2, R-4, S-1

Answer: (C)

5. The sensitivity of LaCoste-Romberg gravimeter is proportional to the line period (T) of the spring as

(A)  T2

(B)  1/T2

(C)  √T

(D)  1/√T

Answer: (A)

6. Match the following gravity/magnetic data interpretation techniques (Group I) with the corresponding terms (Group II)

(A)  P-4, Q-1, R-2, S-3

(B)  P-3, Q-4, R-1, S-2

(C)  P-4, Q-2, R-1, S-3

(D)  P-3, Q-1, R-4, S-2

Answer: (B)

7. Assuming uncorrelated noise, the improvement in the signal to noise ratio in a reflection seismic survey with ‘n’ geophones spaced equally along the profile is proportional to

(A)  n

(B)  1/n

(C)  √n

(D)  1/√n

Answer: (C)

8. A waveform with amplitude spectrum A(ω) and phase spectrum ϕ(ω) is auto-correlated. Which one of the options given below correctly represents the information about t he original waveform that can be retrieved from the autocorrelated waveform?

(A)  A(ω) can be retrieved but not ϕ(ω)

(B)  ϕ(ω) can be retrieved but not A(ω)

(C)  Both A(ω) and ϕ(ω) can be retrieved

(D)  Both A(ω) and ϕ(ω) cannot retrieved

Answer: (A)

9. The convolution A(4, 2, −1, 2) with B(1, 0, −1) gives

(A)  {−4, 2, −5, 0, 1 2}

(B)  {4, 2, −5, 0, 1, −2}

(C)  {−4, −2, 5, 0, −1, −2}

(D)  4, 2, 5, 0, −1, 2}

Answer: (B)

10. Which one of the following does NOT contribute to the suppression of SP log response for a thin, shaly, gas-bearing sandstone formation? (Resistivity of mud filtrate > resistivity of formation water)

(A)  Increase in shale content

(B)  Increase in hydrocarbon content

(C)  Decrease in the thickness of the bed

(D)  Increase in the salinity of formation water

Answer: (D)

11. The crossover observed for a hydrocarbon-bearing sandstone formation in the plot of Neutron and Density porosity logs (∅n – Neutron Density porosity is due to

(A)  increase in ∅d and decrease in ∅n

(B)  decrease in ∅d and increase in ∅n

(C)  increase in both ∅d and ∅n

(D)  decrease in both ∅d and ∅n

Answer: (A)

12. In which one of the following electromagnetic methods are the amplitude ratio and relative phase difference measured between two receiver coils?

(A)  Fixed vertical loop method

(B)  Compensator method

(C)  TURAM method

(D)  Slingram method

Answer: (C)

13. If four impedance tensors Zxx, Zyy, Zxy and Zyx are computed for a 2D body in magneto-telluric method (x is the strike direction), then

(A)  Zxx = 0, Zyy ≠ 0, Zxy = Zyx

(B)  Zxx ≠ 0, Zyy ≠ 0, Zxy = Zyx

(C)  Zxx ≠ 0, Zyy ≠ 0, Zxy ≠ Zyx

(D)  Zxx = 0, Zyy = 0, Zxy ≠ Zyx

Answer: (D)

14. Match the inversion methods (Group I) with the associated terms (Group II)

(A)  P-3, Q-2, R-1, S-4

(B)  P-4, Q-3, R-1, S-2

(C)  P-2, Q-1, R-4, S-3

(D)  P-2, Q-3, R-4, S-1

Answer: (D)

15. Ten equispaced metal electrodes are arranged along a profile for multi-electrode 2D resistivity imaging survey. If Wenner array is used for data recording, the maximum number of observations will be

(A)  7

(B)  11

(C)  12

(D)  13

Answer: (C)

16. P and R are Jacobian matrices for two different geophysical inverse problems. If their generalized inverses are written as P1 = (PTP)1PT and R1 = RT(RRT)1, then

(A)  both P and R deal with over-determined problems

(B)  both P and R deal with under-determined problems

(C)  P deals with over-determined and R deals with under-determined problem

(D)  P deals with under-determined and R deals with over-determined problem

Answer: (C)

17. In a 3D seismic survey, there are 512 groups of receivers in one line of a patch. Eight groups are moved per line from one patch to the next along the swath. What is the inline fold?

(A)  32

(B)  16

(C)  8

(D)  4

Answer: (A)

18. The magnetic potential of a uniform vertically magnetized buried spherical body with uniform density is given as   Then, the vertical magnetic field Bz is proportional to

[I = intensity of magnetization, ρ = density, gz = vertical component of gravity field, G = Universal gravitational constant, μ0  = magnetic permeability of three space, coordinate of the center of the body is (0, z) and that of the observation point is (x, 0)]

(A) 

(B) 

(C) 

(D)

Answer: (A)

19. A sample of granite is observed to have a P-wave velocity of 5 km/s and density of 2600 kg/m3. The bulk modulus of the granite, assuming it to be a Poisson’s solid, is ________kilo-Pascal (kPa). (Round off to 2 decimal places)

Answer: (35000000 to 37000000)

20. The half-life of a parent radionuclide is 100 yrs. If the parent radionuclide decays to a daughter radionuclide which itself decays with a decay constant of 1/4th that of the parent radionuclide, then radioactive equilibrium will be reached after ______ (Round of to 2 decimal places) (Assume at time t = 0 the number of daughter radionuclide is zero)

Answer: (265.00 to 270.00)

21. Current and potential electrodes in resistivity survey over an inhomogeneous ground is shown in the figure below. If 100 mA current flow between C1 and C2 generates 50 mV potential difference between P1 and P2, then the apparent resistivity of the medium will be _______Ω (Round off to 2 decimal places) (Use π = 3.14)

Answer: (37.00 to 41.00)

22. Skin depths in homogeneous media of resistivity ρ1 and ρ2 are 100 m and 200 m, respectively, at 1000 Hz frequency. The ratio ρ12 will be _______.(Round of to 2 decimal places)

Answer: (0.23 to 0.27)

23. The mean resistivity of a horizontally stratified cuboid rock sample is 100 Ωm and coefficient of electrical anisotropy is 1.15. The transverse resistivity of the rock sample is _______ Ω (Round off to 2 decimal places)

Answer: (113.00 to 117.00)

24. A seismic reflection survey is carried out over a 1500 m thick horizontal layer with a P-wave velocity of 2000 m/s. The travel time of a reflected wave at a surface detector placed 1000 m from a surface source is ________ milliseconds.

Answer: (1576 to 1586)

25. A seismic reflection survey is carried out using a 10 milliseconds seismic wavelet over a subsurface medium having an average P-wave velocity of 1600 m/s. The best resolution which is obtained on the basis of Rayleigh criteria is _______ m. (Assume seismic wavelet contains one cycle)

Answer: (4 to 4)

26. To detect a 0.01 nT change in magnetic field using a proton precession magnetometer, the sensitivity required in the frequency measurement of the instrument is ______ × 104 (Round off to 2 decimal places) (Assume gyromagnetic ratio of proton as 2.67515 × 108 s1T1)

Answer: (4.24 to 4.28)

27. A micro-gravity survey with appropriate station spacing is performed to detect a subsurface spherical cavity in a bedrock of density 2500 kg/m3. The depth to the center of the cavity is 4 m from the surface and the elevation measurement accuracy of the surveying instrument is 0.1 m. The smallest cavity that can be detected by the survey must have a radius greater than _______m. (Round of to 1 decimal place) (Assume G = 6.673 × 1011 m3kg1s2)

Answer: (1.6 to 2.1)

28. The gravity anomaly over a spherical ore body is shown in the figure below. The calculated excess mass due to the ore body will be ________ × 1010 (Round off to 1 decimal place) (Assume z = 1.3 × x1/2; G = 6.673 × 1011 m3kg1s2)

Answer: (2.1 to 2.5)

29. A scalar potential field in 3D space is expressed as U(x, y, z) = x2 + yz2. The magnitude of the maximum rate of change in U(x, y, z) at a point (1, 1, 2) is ________.

Answer: (6 to 6)

30. A 10 Hz seismic wave propagates for 40 km through a material with a P-wave velocity of 5 km/s and quality factor (Q) of 100. The percentage of the initial amplitude retained in the attenuated wave is _______. (Round off to 1 decimal place) (Use π = 3.14)

Answer: (7.9 to 8.3)

GATE Exam 2020 Ecology and Evolution (EY) Question Paper With Answer Key

GATE-2020

EY-Ecology and Evolution

GA- General Aptitude

Q1 – Q5 carry one mark each.

1. This book, including all its chapters, ______ interesting. The students as well as the instructor ______ in agreement about it.

(A)  is, was

(B)  are, are

(C)  is, are

(D)  were, was

Answer: (C)

2. People were prohibited ______ their vehicles near the entrance of the main administrative building.

(A)  to park

(B)  from parking

(C)  parking

(D)  to have parked

Answer: (B)

3. Select the word that fits the analogy:

Do : Undo :: Trust : ________

(A)  Entrust

(B)  Intrust

(C)  Distrust

(D)  Untrust

Answer: (C)

4. Stock markets ________ at the news of the coup.

(A)  poised

(B)  plunged

(C)  plugged

(D)  probed

Answer: (B)

5. If P, Q, R, S are four individuals, how many teams of size exceeding one can be formed, with Q as a member ?

(A)  5

(B)  6

(C)  7

(D)  8

Answer: (C)

Q6 – Q10 carry two marks each.

6. Non-performing Assets (NPAs) of a bank in India is defined as an asset, which remains unpaid by a borrower for a certain period of time in terms of interest, principal, or both. Reserve Bank of India (RBI) has changed the definition of NPA thrice during 1993-2004, in terms of the holding period of loans. The holding period was reduced by one quarter each time. In 1993, the holding period was four quarters (360 days).

Based on the above paragraph, the holding period of loans in 2004 after the third revision was ______ days.

(A)  45

(B)  90

(C)  135

(D)  180

Answer: (B)

7. Select the next element of the series: Z, WV, RQP, ______

(A)  LKJI

(B)  JIHG

(C)  KJIH

(D)  NMLK

Answer: (C)

8. In four-digit integer numbers from 1001 to 9999, the digit group “37” (in the same sequence) appears ______ times.

(A)  270

(B)  279

(C)  280

(D)  299

Answer: (C)

9. Given a semicircle with O as the centre, as shown in the figure, the ratio is ______. where  are chords.

(A)  √2

(B)  √3

(C)  2

(D)  3

Answer: (A)

10. The revenue and expenditure of four different companies P, Q, R and S in 2015 are shown in the figure. If the revenue of company Q in 2015 was 20% more than that in 2014, and company Q had earned a profit of 10% on expenditure in 2014, then its expenditure (in million rupees) in 2014 was _______.

(A)  32.7

(B)  33.7

(C)  34.1

(D)  35.1

Answer: (C)

EY: Ecology and Evolution

Q1 – Q25 carry one mark each.

1. Who among the following was a strong public supporter of Darwin’s theory of evolution by natural selection?

(A)  Jean-Baptiste Lamarck

(B)  Carl Linnaeus

(C)  Thomas Huxley

(D)  Gregor Mendel

Answer: (C)

2. Analysis of variance (ANOVA) can be used to compare multiple groups of samples. Select the correct option that reflects the principle behind ANOVA.

(A)  The sum of the squares of the variances is calculated for the groups being compared.

(B)  The variance ratio is calculated by subtracting each value from the overall mean, squaring the difference, and summing the resulting squared deviations.

(C)  The variation between groups is compared with the variation within groups.

(D)  The F value I statistically significant if the mean values between the groups are the same.

Answer: (C)

3. Which of the following information is provided by phylogenetic tree?

(A)  The topology and the branch lengths of the related taxa.

(B)  The topology and sequence length of the gene.

(C)  The sequence length of the gene and tree length.

(D)  The sequence type ad sequence variations within each taxa.

Answer: (A)

4. Myrmecochory refers to seed dispersal by which of the following agents?

(A)  Bats

(B)  Ants

(C)  Lizards

(D)  Birds

Answer: (B)

5. Which of the following is NOT capable of photosynthesis ?

(A)  Diatoms

(B)  Phytoplankton

(C)  Pteridophytes

(D)  Ascomycetes

Answer: (D)

6. Which of the following sensory mechanisms do most frugivorous bats primarily use while foraging?

(A)  Olfaction

(B)  Electromagnetism

(C)  Vibration

(D)  Echolocation

Answer: (A)

7. What direct effect does Follicle Stimulating Hormone (FSH) have in vertebrates?

(A)  It causes follicles of the duodenum to contract.

(B)  It causes follicles o the ovaries to grow.

(C)  It causes follicles of the liver to enlarge.

(D)  It causes follicles of muscles to contract.

Answer: (B)

8. Juvenile rhesus macaques, who have never seen a leopard before, can learn to show fear response if they see an adult react fearfully to a leopard. What kind of behavioural response is this?

(A)  Imprinting

(B)  Instinct

(C)  Cultural transmission

(D)  Mimicry transmission

Answer: (C)

9. In a tropical rainforest during the day, which of the following factors does NOT affect the spectral irradiance at the forest floor?

(A)  Angle of the sun on the horizon.

(B)  Weather conditions of the atmosphere.

(C)  Structure of the canopy vegetation.

(D)  Spectral reflectance of the leaf litter.

Answer: (D)

10. What would evolutionary biologist hypothesize as the ultimate cause for the presence of colouful dewlaps in lizards?

(A)  Colour of dewlaps are formed by pigments in the skin.

(B)  Colourful dewlaps are formed by folds in the skin.

(C)  Colourful dewlaps increase mating success.

(D)  Colourful dewlaps are regions where motor neurons control head movement.

Answer: (C)

11. Which of the following is true about comparisons between herbivorous and carnivorous mammals?

(A)  Herbivores have longer digestive tracts and smaller caecum for the given body size than carnivores.

(B)  Carnivores have longer digestive tracts and smaller caecum for a given body size than herbivores.

(C)  Herbivores have shorter digestive tracts and smaller caecum for a given body size than carnivores.

(D)  Carnivores have shorter digestive tracts and smaller caecum for a given body size than herbivores.

Answer: (D)

12. Two isolated populations X and Y have 100 ad 10000 individuals respectively. Both populations have the same starting allele frequencies of p = 0.5 and q = 0.5. After 1000 generations of genetic drift, which of the following statements is true about the heterozygosity at this locus in these two populations?

(A)  The heterozygosity of population X will be more than population Y.

(B)  The heterozygosity of  population Y will be more than in population X.

(C)  The heterozygosity of both the populations will be identical.

(D)  The heterozygosity of these populations will not depend on their population sizes.

Answer: (B)

13. Which of the following criteria is used to define species under the biological species concept?

(A)  Niche partitioning

(B)  Reproductive isolation

(C)  Morphological divergence

(D)  Genetic distance

Answer: (B)

14. The abundances of three species (P, Q and R) were measured along a resource gradient. The resultant pattern in summarized in the figure. Which of the following statements can be inferred from niche theory?

(A)  P is generalist; Q and R are specialists.

(B)  Q is a generalist; P and R specialists.

(C)  P and Q are generalists; R is a specialist.

(D)  P and R are generalists; Q is a specialist.

Answer: (B)

15. In the last 3 to 4 decades, the average CO2 concentration in the Earth’s atmosphere has increased from_______.

(A)  3 ppm to 4 ppm

(B)  30 ppm to 40 ppm

(C)  300 ppm to 400 ppm

(D)  3000 ppm to 4000 ppm

Answer: (C)

16. What effect does myelination have on neurons?

(A)  It increases the transmembrane resistance.

(B)  It increases the membrane capacitance.

(C)  It changes the direction of signal propagation.

(D)  It protects synapse from damage.

Answer: (A)

17. Which of the following processes contributes to an increase in genetic variation?

(A)  Genetic drift

(B)  Directional selection

(C)  Inbreeding

(D)  Immigration

Answer: (D)

18. What characteristic do the plant species, sundew (Drosera capensis) and Venus fly trap (Dionaea muscipla), share?

(A)  They are thigmonastic.

(B)  They are nyctinastic.

(C)  They are epiphytic.

(D)  They are endophytic.

Answer: (A)

19. What is the study of fish known as?

(A)  Malacology

(B)  Herpetology

(C)  Physiology

(D)  Ichthyology

Answer: (D)

20. On one side of a house, a 50 watt bulb attracts moths at a rate of 30 individuals/minute. On the other side of the house, a 15 watt bulb attracts moths at a rate of 10 individuals/minute. There are 20 bats in the area who are foraging for these moths. According to Ideal Free Distribution, the number of bats near the 15 watt bulb should be _______.

Answer: (5 to 5)

21. The Simpson’s index of diversity is expressed as:

Where pi is the proportion of the ith species, and n is the total number of species.

The Simpson’s index of diversity for this dataset is ____ (round off to three decimal places).

Answer: (0.440 to 0.454)

22. In a botanical garden, tree species P had an average height of 1.5 m, while tree species Q had n average height of 1.8 m Pooled together, these two tree species had an average height of 1.7 m. From this, one can infer that the number of trees of species Q in the garden was _______ times the number of trees of species P.

Answer: (2 to 2)

23. A fragment of double stranded DNA has 30% Adenine. The % GC content in this fragment is _______.

Answer: (40 to 40)

24. A researcher traps rodents in a small, isolated forest patch. In the first trapping session she captures 24 mice ad marks them by notching their ears. In the second trapping session she captures 16 mice, of which 8 are already marked. Assuming that the population is closed (no immigration, emigration, birth, or death), the estimated number mice in the patch is_______.

Answer: (48 to 48)

25. A raptor sitting on a tree sees a rodent on the ground below as shown in the figure (not to scale). If the raptor views the rodent from a height of 10 metres, and the rodent subtends a visual angle of 45° on the raptor’s eye, the straight line distance from the raptor to the rodent in metres is ______ (round off to two decimal places).

Answer: (14.00 to 14.28)

Q26 – Q55 carry two marks each.

26. Beetles of one species have the option of eating plant species X or Y in their environment. Plant species X and Y have the same nutritional quality. When beetle diets comprise a greater proportion of a plant X, the population size of beetles increase faster than when beetle diets are dominated by plant Y. Which of the following is NOT a probable explanation for this outcome?

(A)  Xenobiotics in X are physiologically easier for the beetles to detoxify than t hose in Y.

(B)  Sequestration of xenobiotics from Y by the beetles confers greater protection from bird predators than those from Y.

(C)  X attracts parasites of the beetles while Y does not.

(D)  X provides greater protection from bird predators during foraging than does Y.

Answer: (C)

27. A researcher was documenting the number of tree species in landscape. Within each of three forest types (P, Q and R), she laid 100 quadrats and documented all the species found in each quadrat. She then plotted the cumulative species richness for these forest types as shown in the figure. Which of the following statements is FALSE?

(A)  There are more species of trees in P than in R.

(B)  There are more species of trees in Q than in R.

(C)  More quadrats are required to estimate species richness in P than in Q.

(D)  More quadrats are required to estimate species richness in Q than in R.

Answer: (C)

28. Which of the following ha an endemic species represented in the Andaman and Nicobar Islands for all taxonomic groups in that column?

(A)  Column P

(B)  Column Q

(C)  Column R

(D)  Column S

Answer: (D)

29. The theory of island biogeography predicts that the number of species o islands is determined by: (i) the rate of colonization, which depends on the distance of the island from the mainland and (ii) the rate of extinction which depends on the size of the island. A researcher surveyed two islands with similar habitats and geological history, and found that both islands have the same number of species. Which of the following statement(s) can explain this observation?

(P) The islands are of the same size and are at the same distance from the mainland.

(Q) The islands are of different sizes and are at the same distance from the mainland.

(R) The islands are of the same size and are at different distance from the mainland.

(S) The  islands are of different sizes and are of different distances from t he mainland.

(A)  Only P.

(B)  Only Q.

(C)  Both P and S.

(D)  Both Q and R.

Answer: (C)

30. In marine fauna, Pelagic Larval Duration (PLD) or the amount of time that larvae spend swimming or drifting in the water column affects their dispersal distance. Successful establishment of the larvae on a substrate also depends on finding a suitable habitat after dispersal, which is influenced by whether that habitat is patchy or continuous. Which of the following species will have the lowest population genetic structure (FST) across the same spatial scale?

(A)  Species with high PLD inn patchy habitats.

(B)  Species with high PLD in continuous habitats.

(C)  Species with low PLD in patchy habitats.

(D)  Species with low PLD in continuous habitats.

Answer: (B)

31. Every lake in Wakanda has three species of fish: P, Q and R. Species P is a bottom dweller and substrate feeder, species Q is a mid-column dweller and herbivore, and species R is a surface dweller and piscivore. Which of the following processes best explains this distribution pattern?

(A)  Speciation within one lake followed by dispersal to other lakes.

(B)  Dispersal between lakes followed by speciation within lakes.

(C)  Independent speciation events within all the takes.

(D)  Speciation within a lake with no dispersal between lakes.

Answer: (A)

32. Group living can have both benefits (such as protection from predators) and costs (such as competition for resources). The figure depicts net benefit to individuals as a function of group size. Consider a population with more than hundred individuals, where groups do not split, and individuals can choose to either join a group or remain solitary. Given this information, what is the typical group size predicted?

(A)  Less than 6

(B)  Equal to 6

(C)  Between 6 and 12

(D)  Greater than 12

Answer: (C)

33. Two species of snails, P and Q, are found in rivers across a range of temperatures that vary from upstream a downstream. An experiment was conducted in which P was removed from a river and the distribution of Q was measured after a few weeks. In another similar river, the reciprocal experiment was conducted I which Q was removed, and the distribution of P was measured after a few weeks. In the graph below, the filled bars plot the distribution of P and Q when both species are present in a river. The open bars plot the distribution of P when Q is removed, and Q when P is removed. Which of the following statements is true?

(A)  The realized niche of P is smaller than its fundamental niche.

(B)  The realized niche of Q is smaller than its fundamental niche.

(C)  The realized niche is smaller than the fundamental niche for both species.

(D)  The realized niches of both species are equal.

Answer: (A)

34. In plants with bisexual flowers (hermaphrodites), mate-choice by females is expected to be important under which of the following cases?

(i) Seed set is pollen-limited rather than resource-limited.

(ii) Obligate self-pollination is present.

(iii) Seed set is resource-limited rather than pollen-limited.

(iv) Obligate cross-pollination is present.

(A)  (i) and (ii)

(B)  (ii) and (iii)

(C)  (iii) and (iv)

(D)  (i) and (iv)

Answer: (C)

35. Match the breeding system of the plants with their pollen : ovule ratio.

(A)  P-i, Q-iii, R-ii, S-iv

(B)  P-ii, Q-i, R-iv, S-iii

(C)  P-iv, Q-iii, R-i, S-ii

(D)  P-iii, Q-iv, R-i, S-ii

Answer: (C)

36. A particular gene sequence from two different species shows molecular clock-like evolution. Which of the following statements is consistent with this observation?

(A)  The two sequences will show a linear decrease in their genetic distance with time.

(B)  The genetic distance between the two sequences remains constant over time.

(C)  The rate of evolution for this gene sequence is not constant over time.

(D)  The two sequences will show a linear increase in their genetic distance with time.

Answer: (D)

37. An internal parasite of a mammal does not generate its own heat and yet it can maintain a constant body temperature. Which characteristics describe this parasite?

(A)  Homeothermic ectotherm

(B)  Homeothermic endotherm

(C)  Poikilothermic ectotherm

(D)  Poikilothermic endotherm

Answer: (A)

38. In a population of infinite size, the frequency of two alleles. A1 and A2 at a neutral locus are the same. What are the expected genotype frequencies (A1A1, A1A2, A2A2) after 100 generations of random mating?

(A)  0.25, 0.5, 0.25

(B)  0.5, 0.25, 0.25

(C)  0.25, 0.25, 0.5

(D)  0.05, 0.5, 0.45

Answer: (A)

39. A certain rodent species shows territoriality, competes for space and food, and their population is at carrying capacity. In the figures, the area within the rectangle (i) to (iv) represents a completely homogenous habitat where resources are distributed throughout, and the grey polygons represent rodent home ranges. Which of the following patterns best represents the expected distribution of home ranges of the rodent species if individuals vary in competitive ability?

(A)  (i)

(B)  (ii)

(C)  (iii)

(D)  (iv)

Answer: (D)

40. For a given gene there is 5% DNA sequence divergence between two species, however the protein coded by this gene has identical sequences in the two species. Which of the following types of mutations best explains this pattern in the DNA sequence?

(A)  Nonsense mutation

(B)  Synonymous substitution

(C)  Non-synonymous substitution

(D)  Frame-shift mutation

Answer: (B)

41. A researcher collects data on plant species composition in two habitats (P and Q) by using 10 quadrats each in both habitats. She calculates the average α-diversity and the β-diversity of each habitat from this data (shown below). Which of the following can be inferred about these habitats?

(A)  p and Q have the same total diversity (γ) and P is more heterogeneous than Q.

(B)  Q has lower total diversity (γ) and is more heterogeneous than P.

(C)  P has greater total diversity (γ) and is more heterogeneous than Q.

(D)  Q has greater total diversity (γ) and is more heterogeneous than P.

Answer: (C)

42. Among four related bird species P, Q, R and S, species P and Q are nectar feeding with long thin beaks, whereas species R and S are seed eaters with short thick beaks. Which o the following phylogenies unambiguously suggests that seed eating habit was the ancestral state?

(A)  (i)

(B)  (ii)

(C)  (iii)

(D)  (iv)

Answer: (B)

43. A population of unicorns is growing over time, but its rate of growth is declining. Which of the following graphs best represents this pattern of growth?

(A)  (i)

(B)  (ii)

(C)  (iii)

(D)  (iv)

Answer: (A)

44. In a polyploidization event, tetraploid progeny were formed by diploid parents. Hybridization between the tetraploid and diploid parent gave rise to sterile triploids. Which of the following best explains why these triploids were sterile?

(A)  Many mutations during polyploidization have no phenotypic effect.

(B)  Some chromosomes are without homologs during meiosis.

(C)  Some chromosomes are without homologs during mitosis.

(D)  All chromosomes have homologs during mitosis.

Answer: (B)

45. Blood tests are often used to screen for potential diseases. For a particular disease:

(i) Of 1000 persons who tested negative (T’), 1 person had the disease (D); Pr(D|T’).

(ii) of 10 persons who tested positive (T’), 1 person had the disease (D); Pr(D|T’).

From this we can calculate  

What can be inferred from t his value ?

(A)  100 people have the disease but they will not test positive.

(B)  1 in 100 people have the disease and they will test positive.

(C)  100 in 1000 people have the disease and their blood tests will be inconclusive.

(D)  People with positive tests are 100 time more likely t have the disease than people with negative tests.

Answer: (D)

46. The relative frequency distributions of vales of a trait in two samples, P and Q, are shown in figure. Which of the following statements is consistent with the figure?

(A)  P has higher mean than Q; Q has higher variance than P.

(B)  P has a higher mean than Q: P has a higher variance than Q.

(C)  P and Q have the same mean; Q has higher variance than P.

(D)  P and Q  have the same mean; P has higher variance than Q.

Answer: (C)

47. Consider the function f(x) = |(eβx)/(β)|

Which of the following graphs represents the relationship between x and f(x)?

(A)  (i)

(B)  (ii)

(C)  (iii)

(D)  (iv)

Answer: (A)

48. In a nocturnal moth species (X), individuals with longer antennae have smaller eyes. This indicates a sensory trade-off. In a closely related but diurnal moth species (Y), individuals exhibit the same obligate sensory trade-off. Which of the following figures describes the relationship between these two traits in moth species Y, if it relies more on vision than olfaction?

(A)  (i)

(B)  (ii)

(C)  (iii)

(D)  (iv)

Answer: (A)

49. A researcher compared grass species richness in a 10 m × 10 m plot immediately before (T0) and 100 days after (T100) a fire. She observed that species richness was higher at T100 than at T­0. Which of the following data is required for her to conclude that species richness increased because of the fire?

(A)  Grass species richness of a 10 m × 10 m plot in a nearby area that also burned.

(B)  Grass species richness of a 10 m × 10 m plot only at T100 in nearby area that did not burn.

(C)  Grass species richness of a 10 m × 10 m plot at both T0 and T100 in a nearby area that did not burn.

(D)  Grass species richness of the same 10 m × 10 m  plot every 10 days after the fire until T100.

Answer: (C)

50. Following a gene duplication event, the duplicated copy often loses function and is called a pseudogene. In the absence of positive selection, which of the following is true about these genes?

(A)  The functional gene will accumulate mutations more rapidly than the pseudogene.

(B)  The pseudogene will accumulate mutations more rapidly than the functional gene.

(C)  Both the functional gene and pseudogene will accumulate mutations at the same rate.

(D)  The pseudogene will not accumulate mutations.

Answer: (B)

51. Which of the figures represents the expected relationship between parental care and the number of offspring produced across taxa?

(A)  (i)

(B)  (ii)

(C)  (iii)

(D)  (iv)

Answer: (C)

52. Match the following plant traits with the correct plant group/family:

(A)  P-iv, Q-v, R-i, S-ii

(B)  P-ii, Q-iv, R-iii, S-i

(C)  P-i, Q-ii, R-iii, S-iv

(D)  P-iii, Q-v, R-i, S-ii

Answer: (D)

53. In a population of 100 individuals of a diploid organism with 1:1 sex ratio, the probability of fixation of a new neutral mutation is ______ (round off to three decimal places).

Answer: (0.005 to 0.005)

54. The binomial probability of obtaining exactly k successes in n trials, where the probability of success in a single trial is p, is given by :

Here,  rotation refers to number of combinations for k successes among n trials. With a fair and unbiased coin, the probability of getting 2 HEADS in a trial with 5 tosses is __________ (round off to two decimal places).

Answer: (0.30 to 0.35)

55. The petrified wood fossil was discovered with 8 g of 14 The decay of 14C over time is given by:

NT = N0e0.0001216T

I the half-life of 14C is 5700 years, and the fossil initially had 2 g of 14C, the age of the fossil in  years is ________.

Answer: (11300 to 11500)

GATE Exam 2020 Electrical Engineering (EE) Question Paper With Answer Key

GATE-2020

EE: Electrical Engineering

GA- General Aptitude

Q1 – Q5 carry one mark each.

1. This book, including all its chapters, ______ interesting. The students as well as the instructor ______ in agreement about it.

(A)  is, was

(B)  are, are

(C)  is, are

(D)  were, was

Answer: (C)

2. People were prohibited ______ their vehicles near the entrance of the main administrative building.

(A)  to park

(B)  from parking

(C)  parking

(D)  to have parked

Answer: (B)

3. Select the word that fits the analogy:

Do : Undo :: Trust : ________

(A)  Entrust

(B)  Intrust

(C)  Distrust

(D)  Untrust

Answer: (C)

4. Stock markets ________ at the news of the coup.

(A)  poised

(B)  plunged

(C)  plugged

(D)  probed

Answer: (B)

5. If P, Q, R, S are four individuals, how many teams of size exceeding one can be formed, with Q as a member ?

(A)  5

(B)  6

(C)  7

(D)  8

Answer: (C)

Q6 – Q10 carry two marks each.

6. Non-performing Assets (NPAs) of a bank in India is defined as an asset, which remains unpaid by a borrower for a certain period of time in terms of interest, principal, or both. Reserve Bank of India (RBI) has changed the definition of NPA thrice during 1993-2004, in terms of the holding period of loans. The holding period was reduced by one quarter each time. In 1993, the holding period was four quarters (360 days).

Based on the above paragraph, the holding period of loans in 2004 after the third revision was ______ days.

(A)  45

(B)  90

(C)  135

(D)  180

Answer: (B)

7. Select the next element of the series: Z, WV, RQP, ______

(A)  LKJI

(B)  JIHG

(C)  KJIH

(D)  NMLK

Answer: (C)

8. In four-digit integer numbers from 1001 to 9999, the digit group “37” (in the same sequence) appears ______ times.

(A)  270

(B)  279

(C)  280

(D)  299

Answer: (C)

9. Given a semicircle with O as the centre, as shown in the figure, the ratio  is ______. where  are chords.

(A)  √2

(B)  √3

(C)  2

(D)  3

Answer: (A)

10. The revenue and expenditure of four different companies P, Q, R and S in 2015 are shown in the figure. If the revenue of company Q in 2015 was 20% more than that in 2014, and company Q had earned a profit of 10% on expenditure in 2014, then its expenditure (in million rupees) in 2014 was _______.

(A)  32.7

(B)  33.7

(C)  34.1

(D)  35.1

Answer: (C)

EE: Electrical Engineering

Q1 – Q25 carry one mark each.

1. ax3 + bx2 + cx + d is a polynomial on real x over real coefficients a, b, c, d wherein a ≠ Which of the following statements is true?

(A)  d can be chosen to ensure that x = 0 is a root for any given set a, b, c.

(B)  no choice of coefficients can make all roots identical.

(C)  a, b, c, d can be chosen to ensure that all roots are complex.

(D)  c alone cannot ensure that all roots are real.

Answer: (A OR D)

2. Which of the following is true for all possible non-zero choices of integers m, n; m ≠ n, or all possible non-zero choices of real numbers p, q; p ≠ q, as applicable?

(A)

(B) 

(C) 

(D) 

Answer: (C)

3. Which of the following statements is true about the two sided Laplace transform?

(A)  It exists for every signal that may or may not have a Fourier transform.

(B)  It has no poles for any bounded signal that is non-zero only inside a finite time interval.

(C)  The number of finite poles and finite zeroes must be equal.

(D)  If a signal can be expressed as a weighted sum of shifted one sided exponentials, then its Laplace Transform will have no poles.

Answer: (B)

4. Consider a signal  where 1[n] = 0 if n < 0, and 1 [n] = 1 if n ≥ The z-transform of x[n – k], k > 0 is   with region of convergence being

(A)  |z| < 2

(B)  |z| > 2

(C)  |z| < 1/2

(D)  |z| > 1/2

Answer: (D)

5. The value of the following complex integral, with C representing the unit circle centered at origin in the counterclockwise sense, is:

(A)  8πi

(B)  −8πi

(C)  −πi

(D)  πi

Answer: (C)

6. xR and xA are, respectively, the rms and average values of x(t) = x(t – T), and similarly, yR and yA are, respectively, the rms and average values of y(t) = kx(t). k, T are independent of t. Which of the following is true?

(A)  yA = kxA; yR = kxR

(B)  yA = kxA; yR ≠ kxR

(C)  yA ≠ kxA; yR = kxR

(D)  yA ≠ kxA; yR ≠ kxR

Answer: (A)

7. A three-phase cylindrical rotor synchronous generator has a synchronous reactance Xs and a negligible armature resistance. The magnitude of per phase terminal voltage is VA and the magnitude of per phase induced emf is E­A. Considering the following two statements, P and Q.

P: For any three-phase balanced leading load connected across the terminals of this synchronous generator, V­A is always more than EA

Q: For any three-phase balanced lagging load connected across the terminals of this synchronous generator, VA is always less than EA

Which of the following options is correct?

(A)  P is false Q is true.

(B)  P is true and Q is false.

(C)  P is false and Q is false.

(D)  P is true and Q is true.

Answer: (A)

8. A lossless transmission line with 0.2 pu reactance per phase uniformly distributed along the length of the line, connecting a generator bus to a load bus, is protected up to 80% of its length by a distance relay placed at the generator bus. The generator terminal voltage is 1 pu. There is no generation at the load bus. The threshold pu current for operation of the distance relay for a solid three phase-to-ground fault on the transmission line is closest to:

(A)  1.00

(B)  3.61

(C)  5.00

(D)  6.25

Answer: (D)

9. Out of the following options, the most relevant information needed to specify the real power (P) at the PV buses in a load flow analysis is

(A)  solution of economic load dispatch

(B)  rated power output of the generator

(C)  rated voltage of the generator

(D)  base power of the generator

Answer: (A)

10. Consider a linear time-invariant system whose input r(t) and output y(t) are related by the following differential equation:

The poles of this system are at

(A)  +2j, −2j

(B)  +2, −2

(C)  +4, −4

(D)  +4j, −4j

Answer: (A)

11. A single-phase, full-bridge diode rectifier fed from a 230 V, 50 Hz sinusoidal source supplies a series combination of finite resistance, R, and a very large inductance, L. The two most dominant frequency components in the source current are:

(A)  50 Hz, 0 Hz

(B)  50 Hz, 100 Hz

(C)  50 Hz, 150 Hz

(D)  150 Hz, 250 Hz

Answer: (C)

12. Thyristor T1 is triggered at an angle α (in degree), and T2 at angle 180° + α, in each cycle of the sinusoidal input voltage. Assume both thyristors to be ideal. To control the load power over the range 0 to 2 kW, the minimum range of variation in α is:

(A)  0° to 60°

(B)  0° to 120°

(C)  60° to 120°

(D)  60° to 180°

Answer: (MTA)

13. Which of the options is an equivalent representation of the signal flow graph shown here?

Answer: (C)

14. A common-source amplifier with a drain resistance, RD = 4.7 kΩ, is powered using a 10 V power supply. Assuming that the transconductance, gm, is 520 μA/V, the voltage gain of the amplifier is closest to:

(A)  −2.44

(B)  −1.22

(C)  1.22

(D)  2.44

Answer: (A)

15. A sequence detector is designed to detect precisely 3 digital inputs, with overlapping sequences detectable. For the sequence (1, 0, 1) and input data (1, 1, 0, 1, 0, 0, 1, 1, 0, 1, 0, 1, 1, 0)

(A)  1, 1, 0, 0, 0, 0, 1, 1, 0, 1, 0, 0

(B)  0, 1, 0, 0, 0, 0, 0, 1, 0, 1, 0, 0

(C)  0, 1, 0, 0, 0, 0, 0, 1, 0, 1, 1, 0

(D)  0, 1, 0, 0, 0, 0, 0, 0, 1, 0, 0, 0

Answer: (B)

16. Consider the initial value problem below. The value of y at x = ln 2, (rounded off to 3 decimal places) is ______.

Answer: (0.8774 to 0.8952)

17. A three-phase, 50 Hz, 4-pole induction motor runs at no-load with a slip of 1%. With full load, the slip increases to 5%. The % speed regulation of the motor (rounded off to 2 decimal places) is _______.

Answer: (4.10 to 4.40)

18. Currents through ammeters A2 and A3 in the figure are 1∠10° and 1∠70°, respectively. The reading of the ammeter A1(rounded off to 3 decimal places) is _______A.

Answer: (1.700 to 1.750)

19. The Thevenin equivalent voltage, VTH, in V (rounded off to 2 decimal places) of the network shown below, is ________

Answer: (13.80 to 14.20)

20. A double pulse measurement for an inductively loaded circuit controlled by the IGBT switch is carried out to evaluate the reverse recovery characteristics of the diode, D, represented approximately as a piecewise linear plot of current vs time at diode turn-off. Lpar is a parasitic inductance due to the wiring of the circuit, and in series with the diode. The point on the plot (indicate your choice by entering 1, 2, 3 or 4) at which the IGBT experiences the highest current stress is ________.

Answer: (3 to 3)

21. A single-phase, 4 kVA, 200 V/100 V, 50 Hz transformer with laminated CRGO steel core has rated no-load loss of 450 W. When the high-voltage winding is excited with 160 V, 40 Hz sinusoidal ac supply, the no-load losses are found to be 320 W. When the high-voltage winding of the same transformer is supplied from a 100 V, 25 Hz sinusoidal ac source, the no-load losses will be _________ W (rounded off to 2 decimal places).

Answer: (162.41 to 162.59)

22. A single-phase inverter is fed from a 100 V dc source and is controlled using a quasi-square wave modulation scheme to produce an output waveform. v(t), as shown. The angle σ is adjusted to entirely eliminate the 3rd harmonic component from the output voltage. Under this condition, for v(t), the magnitude of the 5th harmonic component as a percentage of the magnitude of the fundamental component is ________ (rounded off to 2 decimal places).

Answer: (19.90 to 20.20)

23. A single 50 Hz synchronous generator on droop control was delivering 100 MW power to a system. Due to increase in load, generator power had to be increased by 10 MW, as a result of which, system frequency dropped to 49.75 Hz. Further increase in load in the system resulted in a frequency of 49.25 Hz. At this condition, the power in MW supplied by the generator is ________ (rounded off to 2 decimal places).

Answer: (123.00 to 135.00)

24. Consider a negative unity feedback system with forward path transfer function  where K, a, b, c are positive real numbers. For a Nyquist path enclosing the entire imaginary axis and right half of the s-plane in the clockwise direction, the Nyquist plot of (1+ G(s)), encircles the origin of (1+G(s)-plane) once in the clockwise direction and never passes through this origin for a certain value of K. Then, the number of poles of  lying in the open right half of the s-plane is ________.

Answer: (2 to 2)

25. The cross-section of a metal-oxide-semiconductor structure is shown schematically. Starting from an uncharged condition, a bias of +3 V is applied to the gate contact with respect to the body contact. The charge inside the silicon dioxide layer is then measured to be +Q. The total charge contained within the dashed box shown, upon application of bias, expressed as a multiple of Q (absolute value in Coulombs, rounded off to nearest integer( is __________.

Answer: (0 to 0)

Q26 – Q55 carry two marks each.

26. For real numbers, x and y, with y = 3x2 + 3x + 1, the maximum and minimum value of y for x ∈ [−2, 0] are respectively_______.

(A)  7 and 1/4.

(B)  7 and 1.

(C)  −2 and −1/2.

(D)  1 and 1/4.

Answer: (A)

27. The vector function expressed by F = ax (5y – k1z) + ay (3z + k2x) + az (k3y – 4x) represents a conservative field, where ax, ay, az are unit vectors along x, y and z directions, respectively. The values of constants k1, k2, k3 are given by:

(A)  k1 = 3, k2 = 3, k3 = 7

(B)  k1 = 3, k2 = 8, k3 = 5

(C)  k1 = 4, k2 = 5, k3 = 3

(D)  k1 = 0, k2 = 0, k3 = 0

Answer: (C)

28. A 250 V dc shunt motor has an armature resistance of 0.2 Ω and a field resistance of 100 Ω. When the motor is operated on no-load at rated voltage, it draws an armature current of 5 A and runs at 1200 rpm. When a load is coupled to the motor, it draws total line current of 50 A at rated voltage, with a 5% reduction in the air-gap flux due to armature reaction. Voltage drop across the brushes can be taken as 1 V per brush under all operating conditions. The speed of the motor, in rpm, under this loaded condition, is closest to:

(A)  1200

(B)  1000

(C)  1220

(D)  900

Answer: (C)

29. Two buses, i and j, are connected with a transmission line of admittance Y, at the two ends of which there are ideal transformers with turns ratios as shown. Bus admittance matrix for the system is:

Answer: (C)

30. Consider the diode circuit shown below. The diode, D, obeys the current-voltage characteristic  where n > 1, VT > 0, VD is the voltage across the diode and ID is the current through it. The circuit is biased so that voltage, V > 0 and current, I < 0. If you had to design this circuit to transfer maximum power from the current source (I­1) to a resistive  load (not shown) at the output, what values of R1 and R2 would you choose?

(A)  Large R1 and large R2.

(B)  Small R1 and small R2.

(C)  Large R1 and small R2.

(D)  Small R1 and large R2.

Answer: (D)

31. A non-ideal diode is biased with a voltage of −03 V, and a diode current of I1 is measured. The thermal voltage is 26 m V and the ideality factor for the diode is 15/13. The voltage, in V, at which the measured current increases to 1.5I1 is closest to:

(A)  −0.02

(B)  −0.09

(C)  −1.50

(D)  −4.50

Answer: (B)

32. A benchtop dc power supply acts as an ideal 4 A current source as long as its terminal voltage is below 10 V. Beyond this point, it begins to behave as an ideal 10 V voltage source for all load currents going down to 0 A. When connected to an ideal rheostat, find the load resistance value at which maximum power is transferred, and the corresponding load voltage and current.

(A)  Short, ∞ A, 10 V

(B)  Open, 4 A, 0 V

(C)  2.5 Ω, 4 A, 10 V

(D)  2.5 Ω, 4 A, 5 V

Answer: (C)

33. The static electric field inside a dielectric medium with relative permittivity, εr = 2.25, expressed in cylindrical coordinate system is given by the following expression  where ar, aφ, az are unit vectors along r, φ and z directions, respectively. If the above expression represents a valid electrostatic field inside the medium, then the volume charge density associated with this field in terms of free space permittivity, ε0, in SI units is given by:

(A)  3ε0

(B)  4ε0

(C)  5ε0

(D)  9ε0

Answer: (D)

34. Consider a permanent magnet dc (PMDC) motor which is initially at rest. At t = 0, a dc voltage of 5 V is applied to the motor. Its speed monotonically increases from 0 rad/s to 6.32 rad/s in 0.5 s and finally settles to 10 rad/s. Assuming that the armature inductance of the motor is negligible, the transfer function for the motor is

Answer: (B)

35. Which of the following options is correct for the system shown below?

(A)  4th order and stable

(B)  3rd order and stable

(C)  4th order and unstable

(D)  3rd order and unstable

Answer: (C)

36. Consider a negative unity feedback system with the forward path transfer function  where K is a positive real number. The value of K for which the system will have some of its poles on the imaginary axis is_______.

(A)  9

(B)  8

(C)  7

(D)  6

Answer: (B)

37. Suppose for input x(t) a linear time-invariant system with impulse response h(t) produces output y(t), so that x(t) * h (t) = y(t). Further, if |x(t)| * |h(t)| = z(t), which of the following statements is true?

(A)  For all t ∈ (−∞, ∞) z(t) ≤ y(t)

(B)  For some but not all t ∈ (−∞, ∞), z(t) ≤ y(t)

(C)  For all t ∈ (−∞, ∞), z(t) ≥ y(t)

(D)  For some but not all t ∈ (−∞, ∞), z(t) ≥ y(t)

Answer: (C)

38. The casual realization of a system transfer function H(s) having poles at (2, −1), (−2, 1) and zeroes at (2, 1) (−2, −1) will be

(A)  stable, real, allpass

(B)  unstable, complex, allpass

(C)  unstable, real, highpass

(D)  stable, complex, lowpass

Answer: (B)

39. Which of the following options is true for a linear time-invariant discrete time system that obeys the difference equation:

y[n] – ay[n – 1] = b0x[n] – b1x[n – 1]

(A)  y[n] is unaffected by the values of x[n – k]; k > 2.

(B)  The system is necessarily casual.

(C)  The system impulse response is non-zero at infinitely many instants.

(D)  When x[n] = 0, n < 0, the function y[n]; n > 0 is solely determined by the function x[n].

Answer: (C)

40. Let ar, aϕ and az be unit vectors along, r, ϕ and z directions, respectively in the cylindrical coordinate system. For the electric flux density given b y D = (ar15 + aϕ2r – az3rz) Coulomb/m2, the total electric flux, in Coulomb, emanating from the volume enclosed by a solid cylinder of radius 3 m and height 5 m oriented along the z-axis with its base at the origin is:

(A)  54 π

(B)  90 π

(C)  108 π

(D)  180 π

Answer: (D)

41. A stable real linear time-invariant system with single pole at p, has a transfer function  with a dc gain of 5. The smallest positive frequency, in rad/s, at unity gain is closest to:

(A)  8.84

(B)  11.08

(C)  78.13

(D)  122.87

Answer: (A)

42. The number of purely real elements in a lower triangular representation of the given 3 × 3 matrix, obtained through the given decomposition is _______.

(A)  5

(B)  6

(C)  8

(D)  9

Answer: (MTA)

43. The figure below shows the per-phase Open Circuit Characteristics (measured in V) and Short Circuit Characteristics (measured in A) of a 14 kVA, 400 V, 50 Hz, 4-pole, 3-phase, delta connected alternator, driven at 1500 rpm. The field current, If is measured in A. Readings taken are marked as respective (x, y) coordinates in the figure. Ratio of the unsaturated synchronous impedances (Zs(unsat)/Zs(sat)) of the alternator is closest to:

(A)  2.100

(B)  2.025

(C)  2.000

(D)  1.000

Answer: (A)

44. Let ax and ay be unit vectors along x and y directions, respectively. A vector function is given by

F = axy – ayx

The line integral of the above function

along the curve C, which follows the parabola y = x2 as shown below is _______ (rounded off to 2 decimal places).

Answer: (-3.05 to -2.95)

45. A resistor and a capacitor are connected in series to a 10 V dc supply through a switch. The switch is closed at t = 0, and the capacitor voltage is found to cross 0 V at t = 0.4τ is the circuit time constant The absolute value of percentage change required in the initial capacitor voltage if the zero crossing has to happen at t = 0.2τ is ________(rounded off to 2 decimal places).

Answer: (54.00 to 56.00)

46. A cylindrical rotor synchronous generator with constant real power output and constant terminal voltage is supplying 100 A current to a 0.9 lagging power factor load. An ideal reactor is now connected in parallel with the load, as a result of which the total lagging reactive power requirement of the load is twice the previous value while the real power remains unchanged. The armature current is now ________ A (rounded off to 2 decimal places).

Answer: (123.00 to 127.00)

47. Bus 1 with voltage magnitude V1 = 1.1 pu is sending reactive power Q12 towards bus 2 with voltage magnitude V2 = 1 pu through a lossless transmission line of reactance X. Keeping the voltage at bus 2 fixed at 1 pu, magnitude of voltage at bus 1 is changed, so that the reactive power Q12 sent from bus 1 is increased by 20%. Real power flow through the line number both the conditions is zero. The new value of the voltage magnitude V1, in pu (rounded off to 2 decimal places),at bus 1 is _______.

Answer: (1.11 to 1.13)

48. Windings ‘A’, ‘B’ and ‘C’ have 20 turns each and are wound on the same iron core as shown, along with winding ‘X’ which has 2 turns. The figure shows the sense (clockwise/anti-clockwise) of each of the windings only and does not reflect the exact number of turns. If windings ‘A’, ‘B’ and ‘C’ are supplied with balanced 2-phase voltages at 50 Hz and there is no core saturation, the no-load RMS voltage (in V, rounded off to 2 decimal places) across winding ‘X’ is ________.

Answer: (MTA)

49. A cylindrical rotor synchronous generator has steady state synchronous reactance of 0.7 pu and subtransient reactance of 0.2 pu. It is operating at (1 + j0) pu terminal voltage with an internal emf of (1 + j0.7) pu. Following a three-phase solid short circuit fault at the terminal of the generator, the magnitude of the subtransient internal emf (rounded off to 2 decimal places) is _______pu.

Answer: (1.01 to 1.03)

50. In the dc-dc converter circuit shown, switch Q is switched at a frequency of 10 kHz with a duty ratio of 0.6. All components of the circuit are ideal, and the initial current in the inductor is zero. Energy stored in the inductor in mJ (rounded off to 2 decimal places) at the end of 10 complete switching cycles is ________.

Answer: (4.95 to 5.05)

51. A single-phase, full-bridge, fully controlled thyristor rectifier feeds a load comprising a 10 Ω resistance in series with a very large inductance. The rectifier is fed from an ideal 230 V, 50 Hz sinusoidal source through cables which have negligible internal resistance and a total inductance of 2.28 mH. If the thyristors are triggered at an angle α = 45°, the commutation overlap angle in degree (rounded off to 2 decimal p laces) is _______.

Answer: (4.51 to 5.10)

52. A non-ideal Si-based pn junction diode is tested by sweeping the bias applied across its terminals from −5 V to + 5 V. The effective thermal voltage, VT, for the diode is measured to be (29 ± 2) mV. The resolution of the voltage source in the measurement range is 1 mV. The percentage uncertainty (rounded off to 2 decimal places) in the measured current at a bias voltage of 0.02 V is ______.

Answer: (11.50 to 12.00)

53. The temperature of the coolant oil bath for a transformer is monitored using the circuit shown. It contains a thermistor with a temperature-dependent resistance, Rthermistor = 2(1 + αT) kΩ, where T is the temperature in ° The temperature coefficient, α, is –(4 ± 0.25)%/°C. Circuit parameters; R1 = 1 kΩ, R2 = 1.3 kΩ, R3 = 2.6 kΩ. The error in the output signal (in V, rounded off to 2 decimal places) at 150°C is _______.

Answer: (MTA)

54. An 8085 microprocessor accesses two memory locations (2001 H) and (2002 H), that contain 8-bit numbers 98H and B1H, respectively. The following program is executed:

LXIH,2001H

MVI A, 21H

INX H

ADD M

INX H

MOV M, A

HLT

At the end of this program, the memory location 2003H contains the number in decimal (base 10) form _______.

Answer: (210 to 210)

55. A conducting square loop of side length 1 m is placed at a distance of 1 m from a long straight wire carrying a current I = 2 A as shown below. The mutual inductance, in nH (rounded off to 2 decimal places), between the conducting loop and the long wire is _______.

Answer: (138.10 to 139.20)

GATE Exam 2020 Electronics and Communication Engineering (EC) Question Paper With Answer Key

GATE-2020

EC: Electronics and Communication Engineering

GA: General Aptitude

Q 1 – Q5 Carry one mark each.

1. The untimely loss of life is a cause of serious global concern as thousands of people get killed ______ accidents every year while many other die ______ diseases like cardio vascular disease, cancer, etc.

(A)    in, of

(B)    from, of

(C)    during, from

(D)    from, from

Answer: (A)

2. He was not only accused of theft ______ of conspiracy.

(A)    rather

(B)    but also

(C)    but even

(D)    rather than

Answer: (B)

3. Select the word that fits the analogy:

Explicit : Implicit : : Express: _______

(A)    Impress

(B)    Repress

(C)    Compress

(D)    Suppress

Answer: (B)

4. The Canadian constitution requires that equal importance be given to English and French. Last year, Air Canada lost a lawsuit, and had to pay a six-figure fine to a French-speaking couple after they filed complaints about formal in-flight announcements is English lasting 15 seconds, as opposed to informal 5 second messages in French.

The French-speaking couple were  upset at ______.

(A)    the in-flight announcements being made in English.

(B)    the English announcements being clearer than the French ones.    

(C)    the English announcements being longer than the French ones.

(D)    equal importance being given to English and French.        

Answer: (C)

5. A superadditive function f (∙) satisfies the following property

f(x1 + x2) ≥ f(x1) + f(x2)

Which of the following functions is a superadditive function for x > 1?

(A)    ex

(B)    √x

(C)    1/x

(D)    ex

Answer: (A)

Q6 – Q10 carry two marks each.

6. The global financial crisis in 2008 is considered to be the most serious world-wide financial crisis, which started with the sub-prime lending crisis in USA in 2007. The sub-prime lending crisis led to the banking crisis in 2008 with the collapse of Lehman Brothers may have difficulties in repaying loans, and it arises because of excess liquidity following the East Asian crisis.

Which one of the following sequences shows the correct precedence as per the given passage?

(A)    East Asian crisis → subprime lending crisis → banking crisis global financial crisis.

(B)    Subprime lending crisis → global financial crisis → banking crisis → East Asian crisis.

(C)    Banking crisis → subprime lending crisis → global financial crisis → East Asian crisis.

(D)    Global financial crisis → East Asian crisis → Banking crisis → subprime lending crisis.

Answer: (A)

7. It is quarter past three in your watch. The angle between the hour hand and the minute hand is ________.

(A)    0°

(B)    7.5°

(C)    15°

(D)    22.5°

Answer: (B)

8. A circle with centre O is shown in the figure. A rectangle PQRS of maximum possible area is inscribed in the circle. If the radius of the circle is a, then the area of the shaded portion is ______.

(A)    πa2 – a2

(B)    πa2 – √2a2

(C)    πa2 – 2a2

(D)    πa2 – 3a2

Answer: (C)

9. a, b, c are real numbers. The quadratic equation ax2 – bx + c = 0 has equal roots, which is β, then

(A)    β = b/a

(B)    β2 = ac

(C)    β3 = bc/(2a2)

(D)    b2 ≠ 4ac

Answer: (C)

10. The following figure shows the data of students enrolled in 5 years (2014 to 2018) for two schools P and Q. During this period, the ratio of the average number of the students enrolled in school P to the average of the difference of the number of students enrolled in schools P and Q is ______ .

(A)    8 : 23

(B)    23 : 8

(C)    23 : 31

(D)    31 : 23

Answer: (B)

EC: Electronics and Communication Engineering

Q1- Q25 carry one mark each.

1. If v1, v2,….,v6 are six vectors in R4, which one of the following statements is FALSE

(A)  It is not necessary that these vectors span R4.

(B)  These vectors are not linearly independent.

(C)  Any four of these vectors form a basis for R4.

(D)  If {v1, v3, v5, v6} spans R4, then it forms a basis for R4

Answer: (C)

2. For a vector field , which one of the following is FALSE?

(A)  is solenoidal if 

(B)   is another vector field.

(C)  is irrotational if 

(D)   

Answer: (C)

3. The partial derivative of the function f(x, y, z)  with respect to x at the point (1, 0, e) is

(A)  −1

(B)  0

(C)  1

(D)  1/e

Answer: (B)

4. The general solution of  is

(A)  y = C1e3x + C2e3x

(B)  y = (C1 + C2x) e3x

(C)  y = (C1 + C2x)e3x

(D)  y = C1e3x

Answer: (C)

5. The output y[n] of a discrete-time system for an input x[n] is  The unit impulse response of the system is

(A)  0 for all n.

(B)  1 for all n.

(C)  unit step signal u[n].

(D)  unit impulse signal δ[n].

Answer: (C)

6. A single crystal intrinsic semiconductor is at a temperature of 300 K with effective density of states for holes twice that of electrons. The thermal voltage is 26 mV. The intrinsic Fermi level is shifted from mid-bandgap energy level by

(A)  18.02 meV.

(B)  9.01 meV.

(C)  13.45 meV.

(D)  26.90 meV.

Answer: (B)

7. Consider the recombination process via bulk traps in a forward biased pn homojunction diode. The maximum recombination rate is Umax. If the electron and the hole capture cross-sections are equal, which one of the following is FALSE?

(A)  With all other parameters unchanged, Umax decreases if the intrinsic carrier density is reduced.

(B)  Umax occurs at the edges of the depletion region in t he device.

(C)  Umax depends exponentially on the applied bias.

(D)  With all other parameters unchanged, Umax increases if the thermal velocity of the carriers increases.

Answer: (B)

8. The components in the circuit shown below are ideal. If the op-amp is in positive feedback and the input voltage Vi is a since wave of amplitude 1 V, the output voltage Vo is

(A)  a non-inverted sine wave of 2 V amplitude.

(B)  an inverted sine wave of 1 V amplitude.

(C)  a square wave of 5 V amplitude.

(D)  a constant of either +5V or −5V.

Answer: (D)

9. In the circuit shown below, the Thevenin voltage VTH is

(A)  2.4 V

(B)  2.8 V

(C)  3.6 V

(D)  4.5 V

Answer: (C)

10. The figure below shows a multiplexer where S1 and S0 are the select lines, I0 to I3 are the input data lines, EN is the enable line, and F(P, Q, R) is the output. F is

(A)   

(B)   

(C)   

(D)   

Answer: (A)

11. The pole-zero map of a rational function G(s) shown below. When the closed contour Γ is mapped into the G(s)-plane, then the mapping encircles

(A)  the origin of the G(s)-plane once in the counter-clockwise direction.

(B)  the origin of the G(s)-plane once in the clockwise direction.

(C)  the point −1 + j0 of the G(s)-plane once in the counter-clockwise direction.

(D)  the point −1 + j0 of the G(s)-plane once in the clockwise direction.

Answer: (B)

12. A digital communication system transmits a block of N bits. The probability of error in decoding a bit is α. The error event of each bit is independent of the error events of the other bits. The received block is erroneous if at least one of its bits is decoded wrongly. The probability that the received block is erroneous is

(A)  N(1 – α)

(B)  αN

(C)  1 – αN

(D)  1 – (1 – α)N

Answer: (D)

13. The impedances Z = jX, for all X in the range (−∞, ∞), map to the Smith chart as

(A)  a circle of radius 1 with centre at (0, 0).

(B)  a point at the centre of the chart.

(C)  a line passing through the centre of the chart.

(D)  a circle of radius 0.5 with centre at (0.5, 0).

Answer: (A)

14. Which one of the following pole-zero plots corresponds to the transfer function of an LTI system characterized by the input-output difference equation given below?

Answer: (A)

15. In the given circuit, the two-port network has the impedance matrix  The value of ZL for which maximum power is transferred to the load is ________ Ω.

Answer: (48 to 48)

16. The current in the RL-circuit shown below is i(t) = 10 cos(5t – π/4) A. The value of the inductor (rounded off to two decimal places) is _______ H.

Answer: (2.80 to 2.85)

17. In the circuit shown below, all the components are ideal and the input voltage is sinusoidal. The magnitude of the steady-state output V0(rounded off to two decimal places) is _______ V.

Answer: (644 to 657)

18. In the circuit shown below, all the components are ideal. If Vi is +2V, the current Io sourced by the op-amp is _______mA.

Answer: (6 to 6)

19. In an 8085 microprocessor, the number of address lines required to access a 16 K byte memory bank is _________.

Answer: (14 to 14)

20. A 10-bit D/A converter is calibrated over the full range from 0 to 10 V. If the input to the D/A converter is 13A (in hex), the output (rounded off to three decimal places) is _______V.

Answer: (3.050 to 3.080)

21. A transmission line of length 3λ/4 and having a characteristic impedance of 50 Ω is terminated with a load of 400 Ω. The impedance (rounded off to two decimal places) seen at the input end of the transmission line is _______ Ω.

Answer: (6.25 to 6.25)

22. A binary random variable X takes the value +2 or − The probability P(X = +2) = α. The value of α (rounded off to one decimal place), for which the entropy of X is maximum, is _______.

Answer: (0.5 to 0.5)

23. The loop transfer function of a negative feedback system is  The value of K, for which the system is marginally stable, is ________.

Answer: (160 to 160)

24. The random variable  where  and W(t) is a real white Gaussian noise process with two-sided power spectral density SW(f) = 3 W/Hz, for all f. The variance of Y is _______.

Answer: (6 to 6)

25. The two sides of a fair coin are labelled as 0 and 1. The coin is tossed two times independently. Let M and N denote the labels corresponding to the outcomes of those tosses. For a random variable X, defined as X = min(M, N), the expected value E(X) (rounded off to two decimal places) is _______.

Answer: (0.25 to 0.25)

Q26 – Q55 carry two marks each.

26. Consider the following system of linear equations.

x1 + 2x2 = b1;      2x1 + 4x2 = b2;             3x1 + 7x2 = b3;             3x1 + 9x2 = b4

Which one of the following conditions ensures that a solution exists for the above system?

(A)  b2 = 2b1 and 6b1 – 3b3 + b4 = 0

(B)  b3 = 2b1 and 6b1 – 3b3 + b4 = 0

(C)  b2 = 2b1 and 3b1 – 6b3 + b4 = 0

(D)  b3 = 2b1 and 3b1 – 6b3 + b4 = 0

Answer: (A)

27. Which one of the following options contains two solutions of the differential equation 

(A)  ln|y – 1| = 0.5x2 + C and y = 1

(B)  ln|y – 1| = 2x2 + C and y = 1

(C)  ln|y – 1| = 0.5x2 + C and y = −1

(D)  ln|y – 1| = 2x2 + C and y = −1

Answer: (A)

28. The current I in the given network is

(A)  0 A.

(B)  2.38 ∠−96.37° A.

(C)  2.38 ∠143.63° A.

(D)  2.38 ∠−23.63° A.

Answer: (C)

29. A finite duration discrete-time signal x[n] is obtained by sampling the continuous-time signal x(t) = cos (200 πt) at sampling instants t = n/400, n = 0, 1, …. , 7. The 8-point discrete Fourier transform (DFT) of x[n] is defined as

Which one of the following statements is TRUE?

(A)  All X[k] are non-zero.

(B)  Only X[4] is non-zero.

(C)  Only X[2] and X[6] are non-zero.

(D)  Only X[3] and X[5] are non-zero.

Answer: (C)

30. For the given circuit, which one of the following is the correct state equation?

Answer: (A)

31. A one-sided abrupt pn junction diode has a depletion capacitance CD of 50 pF at a reverse bias of 0.2 V. The plot of 1/CD2 versus the applied voltage V for this diode is a straight line as shown in the figure below. The slope of the plot is ________× 1020 F2 V1.

(A)  −5.7

(B)  −3.8

(C)  −1.2

(D)  −0.4

Answer: (MTA)

32. The band diagram of a p-type semiconductor with a band-gap of 1 eV is shown. Using this semiconductor, a MOS capacitor having VTH of −16 V C′ox of 100 nF/cm2 and a metal work function of 3.87 eV is fabricated. There is no charge within the oxide. If the voltage across the capacitor is VTH, the magnitude of depletion charge per unit area (in C/cm2) is

(A)  1.70 × 108

(B)  0.52 × 108

(C)  1.41 × 108

(D)  0.93 × 108

Answer: (A)

33. The base of an npn BJT T1 has a linear doping profile NB(x) as shown below. The base of another npn BJT T2 has a uniform doping NB of 1017 cm3. All other parameters are identical for both the devices. Assuming that the hole density profile is the same as that of doping, the common-emitter current gain of T2 is

(A)  approximately 2.0 times that of T1.

(B)  approximately 0.3 times that of T1.

(C)  approximately 2.5 times that of T1.

(D)  approximately 0.7 times that of T1.

Answer: (MTA)

34. A pn junction solar cell of area 1.0 cm2, illuminated uniformly with 100 nW cm2, has the following parameters: Efficiency = 15%, open circuit voltage = 0.7 V, fill factor = 0.8, and thickness = 200 μ The charge of an electron is 1.6 × 1019 C. The average optical generation rate (in cm3 s1) is

(A)  0.84 × 1019.

(B)  5.57 × 1019.

(C)  1.04 × 1019.

(D)  83.60 × 1019.

Answer: (A)

35. For the BJT in the amplifier shown below, VBE = 0.7 V, kT/q = 26 mV. Assume that BJT output resistance (r0) is very high and the base current is negligible. The capacitors are also assumed to be short circuited at signal frequencies. The input vi is direct coupled. The low frequency voltage gain v0/vi of the amplifier is

(A)  −89.42

(B)  −128.21

(C)  −178.85

(D)  −256.42

Answer: (A)

36. An enhancement MOSFET of threshold voltage 3 V is being used in the sample and hold circuit given below. Assume that the substrate of the MOS device is connected to −10 V. If the input voltage v1 lies between ±10 V, the minimum and the maximum values of vG required for proper sampling and holding respectively, are

(A)  3 V and −3 V.

(B)  10 V and −10 V.

(C)  13 V and −7 V.

(D)  10 V and −13 V.

Answer: (C)

37. Using the incremental low frequency small-signal model of the MOS device, the Norton equivalent resistance of the following circuit is

(A)  rds + R + gmrdsR

(B)   

(C)   

(D)  rds + R

Answer: (B)

38. P, Q, and R are the decimal integers corresponding to the 4-bit binary number 1100 considered in signed magnitude, 1’s complement, and 2’s complement representations, respectively. The 6-bit 2’s complement representation of (P + Q + R) is

(A)  110101

(B)  110010

(C)  111101

(D)  111001

Answer: (A)

39. The state diagram of a sequence detector is shown below. State S0 is the initial state of the sequence detector. If the output is 1, then

(A)  the sequence 01010 is detected.

(B)  the sequence 01011 is detected.

(C)  the sequence 01110 is detected.

(D)  the sequence 01001 is detected.

Answer: (A)

40. The characteristic equation of a system is s3 + 3s2 + (K + 2)s + 3K = 0. In the root locus plot for the given system, as K varies from 0 to ∞, the break-way or break-in point(s) lie within

(A)  (−1, 0).

(B)  (−2, −1).

(C)  (−3, −2).

(D)  (−∞,−3).

Answer: (A)

41. The components in the circuit given below are ideal. If R = 2 kΩ and C = 1 μF, the –dB cutt-off frequency of the circuit in Hz is

(A)  14.92

(B)  34.46

(C)  59.68

(D)  79.58

Answer: (D)

42. For the modulated signal x(t) = m(t) cos (2πfct), the message signal m(t) = 4 cos(1000 πt) and the carrier frequency f­c is 1 MHz. The signal x(t) is passed through a demodulator, as shown in the figure below. The output y(t) of the demodulator is

(A)  cos(460 πt).

(B)  cos(920 πt).

(C)  cos(1000 πt).

(D)  cos(540 πt).

Answer: (B)

43. For an infinitesimally small dipole in free space, the electric field Eθ in the far field is proportional to (ejkr/r) sin θ, where k = 2π/λ. A vertical infinitesimally electric dipole (δl << λ) is placed at a distance h(h > 0) above an infinite ideal conducting plane, as shown in the figure. The minimum value of h, for which one of the maxima in the far field radiation pattern occurs at θ = 60°, is

(A)  λ

(B)  0.5λ

(C)  0.25λ

(D)  0.75λ

Answer: (A)

44. In the voltage regulator shown below, V1 is the unregulated input at 15 V. Assume VBE = 0.7 V and the base current in negligible for both the BJTs. If the regulated output VO is 9 V, the value of R­2 is ________ Ω.

Answer: (800 to 800)

45. The magnetic field of a uniform plane wave in vacuum is given by  cos(ωt + 3x – y – z). The value of b is _______.

Answer: (1 to 1)

46. For a 2-port network consisting of an ideal lossless transformer, the parameter S21 (rounded off to two decimal places) for a reference impedance of 10 Ω, is _______.

Answer: (0.8 to 0.8)

47. SPM(t) and S­FM(t) as defined below, are the phase modulated and the frequency modulated waveforms, respectively, corresponding to the message signal m(t) shown in the figure.

SPM(t) = cos(1000πt + Kpm(t))

where Kp is the phase deviation constant in radians/volt and Kf is the frequency deviation constant in radians/second/volt. If the highest instantaneous frequencies of SPM(t) and SFM(t) are same, then the value of the ratio kp/kff is _______ seconds.

Answer: (2 to 2)

48. In a digital communication system, a symbol S randomly chosen from the set {s1, s2, s3, s4} is transmitted. It is given that s1 = −3, s2 = −1, s3 = +1 and s4 = +2. The received symbol is Y = S + W. W is a zero-mean unit-variance Gaussian random variable and is independent of S. Pi­ is the conditional probability of symbol error for the maximum likelihood (ML) decoding when the transmitted symbol S = si. The index i for which the conditional symbol error probability Pi is the highest is _________.

Answer: (3 to 3)

49. A system with transfer function  is subjected to an input 5 cos 3t. The steady state output of the system is  The value of a is _______.

Answer: (3.95 to 4.05)

50. For the components in the sequential circuit shown below, tpd is the propagation delay, tsetup is the setup time, and thold is the hold time. The maximum clock frequency (rounded off to the nearest integer), at which the given circuit can operate reliably, is _________ MHz.

Answer: (76 to 77)

51. For the solid S shown below, the value of  (rounded off to two decimal places) is ___________

Answer: (2.25 to 2.25)

52. X(ω) is the Fourier transform of x(t) shown below. The value of  (rounded off to two decimal places) is _________.

Answer: (58.50 to 58.80)

53. The transfer function of a stable discrete-time LTI system is  where K and α are real numbers. The value of α (rounded off to one decimal place) with |α| > 1, for which the magnitude response of the system is constant over all frequencies, is _______.

Answer: (-2 to -2)

54. X is a random variable with uniform probability density function in the interval [−2, 10]. For Y = 2X – 6, the conditional probability P(Y ≤ 7\X ≥ 5) (rounded off to three decimal places) is ________.

Answer: (0.299 to 0.301)

55. Consider the following closed loop control system

where  If the steady state error for a unit ramp input is 0.1, then the value of K is _________.

Answer: (30 to 30)

GATE Exam 2020 Chemistry (CY) Question Paper With Answer Key

GATE-2020

CY: Chemistry

GA-General Aptitude

Q1 – Q5 carry one mark each.

1. While I agree _____ his proposal this time, I do not often agree _____ him.

(A)  to, with

(B)  with, to

(C)  with, with

(D)  to, to

Answer: (A)

2. The recent measures to improve the output would _______ the level of production to our satisfaction.

(A)  increase

(B)  decrease

(C)  speed

(D)  equalise

Answer: (A)

3. Select the word that fits the analogy:

White : Whitening : Light: ______

(A)  Lightning

(B)  Lightening

(C)  Lighting

(D)  Enlightening

Answer: (B)

4. In one of the greatest innings ever seen in 142 years of Test history, Ben Stokes upped the tempo in a five-and-a-half hour long stay of 219 balls including 11 fours and 8 sixes that saw him finish on a 135 not out as England squared the five-match series.

Based on their connotations in the given passage, which one of the following meanings DOES NOT match?

(A)  upped = increased

(B)  squared = lost

(C)  tempo = enthusiasm

(D)  saw = resulted in

Answer: (B)

5. There are five levels {P, Q, R, S, T} in a linear supply chain before a product reaches customers, as shown in the figure.

At each of the five levels, the price of the product is increased by 25%. If the product is produced at level P at the cost of Rs. 120 per unit, what is the price paid (in rupees) by the customers?

(A)  187.50

(B)  234.38

(C)  292.26

(D)  366.21

Answer: (D)

Q6 – Q10 carry two marks each.

6. Climate change and resilience deal with two aspects – reduction of sources of non-renewable energy resources and reducing vulnerability of climate change aspects. The terms ‘mitigation’ and ‘adaptation’ are used to refer to these aspects, respectively.

Which of the following assertions is best supported by the above in information?

(A)  Mitigation deals with consequences of climate change.

(B)  Adaptation deals with causes of climate change.

(C)  Mitigation deals with actions taken to reduce the use of fossil fuels.

(D)  Adaptation deals with actions taken to combat gree-house gas emissions.

Answer: (C)

7. Find the missing element in the following figure.

(A)  d

(B)  e

(C)  w

(D)  y

Answer: (A)

8. It was estimated that 52 men can complete a strip in a newly constructed highway connecting cities P and Q in 10 days. Due to an emergency, 12 men were sent to another project. How many number of days, more than the original estimate, will be required to complete the strip?

(A)  3 days

(B)  5 days

(C)  10 days

(D)  13 days

Answer: (A)

9. An engineer measures THREE quantities X, Y and Z in an experiment. She finds that they follow a relationship that is represented in the figure below : (the product of X and Y linearly varies with Z)

Then, which of the following statements is FALSE?

(A)  For fixed Z; X is proportional to Y

(B)  For fixed Y; X is proportional to Z

(C)  For fixed X; Z is proportional to Y

(D)  XY/Z is constant

Answer: (A)

10. The two pie-charts given below show the data of total students and only girls registered in different streams in a university. If the total number of students registered in the university is 5000, and the total number of the registered girls is 1500; then, the ratio of boys enrolled in Arts to the girls enrolled in Management is ______.

(A)  2:1

(B)  9:22

(C)  11:9

(D)  22:9

Answer: (D)

CY: Chemistry

Q1 – Q25 carry one mark each.

1. Among the following, the suitable reagents for the given transformation is:

(A)  H2, Pd/C

(B)  H2N-NH2/KOH, ∆

(C)  NaBH4/CeCl3∙7H2O

(D)  Li/Liq. NH3

Answer: (C)

2. Major product formed in the following reaction sequence is:

Answer: (C)

3. Major product formed in the following reaction is:

Answer: (A)

4. Major product formed in the following transformation is:

Answer: (C)

5. Absolute stereochemistry of the given compound is:

(A)  4aR, 8aS

(B)  4aR, 8aR

(C)  4aS, 8aS

(D)  4aS, 8aR

Answer: (D)

6. In the following reaction sequence,

the major product P and Q are :

Answer: (C)

7. Major product formed in the given reaction is:

Answer: (B)

8. The CORRECT statement regarding the substitution of coordinated ligands in Ni(CO)4 and CO(NO)(CO)3 is:

(Given Co―N―O bond is nearly linear; atomic numbers of Co and Ni are 27 and 28, respectively)

(A)  Ni(CO)4 and Co(NO)(CO)3 follow associative and dissociative pathways, respectively.

(B)  Ni(CO)4 and Co(NO)(CO)3 follow dissociative and associative pathways, respectively.

(C)  Both Ni(CO)4 and Co(NO)(CO)3 follow associative pathway.

(D)  Both Ni(CO)4 and Co(NO)(CO)3 follow dissociative pathway.

Answer: (B)

9. The CORRECT statement about hexagonal boron nitride is :

(A)  It is a good electrical conductor.

(B)  It has same layer stacking as that of graphite.

(C)  It is reactive towards fluorine.

(D)  It has lower thermal stability in air compared to that of graphite.

Answer: (C)

10. In oxyhemocyanin, the coordination number, mode of oxygen binding, color and the net magnetic behavior of copper ions, respectively are:

(Given: atomic number of Cu is 29)

(A)  Four, μ-η11-O2, colorless and paramagnetic.

(B)  Five, μ-η22-O2, colorless and paramagnetic.

(C)  Five, μ-η22-O22, blue and diamagnetic.

(D)  Four, μ-η11-O22, blue and diamagnetic.

Answer: (C)

11. Among the following species, the one that has pentagonal shape is:

(Given: atomic numbers of O, F, S, I and Xe are 8, 9, 16, 53 and 54, respectively)

(A)  XeOF4

(B)  IF5

(C)  [SF5]

(D)  [XeF5]

Answer: (D)

12. A solution containing a metal complex absorbs at 480 mm with molar extinction coefficient of 15,000 L mol1 cm1. If the path length of the cell is 1.0 cm and transmittance is 20.5%, the concentration (in mol L1) of the metal complex is:

(A)  1.37 × 105

(B)  2.29 × 105

(C)  4.59 × 105

(D)  8.75 × 105

Answer: (C)

13. Among the following linear combination of atomic orbitals, the CORRECT representation of the lowest unoccupied π-molecular orbital of butadiene is:

(A)  Ψ = −0.372 ϕ1 + 0.602 ϕ2 – 0.602 ϕ3 + 0..372 ϕ4

(B)  Ψ = 0.602 ϕ1 – 0.372 ϕ2 – 0.372 ϕ3 + 0.602 ϕ4

(C)  Ψ = 0.602 ϕ1 + 0.372 ϕ2 – 0.372 ϕ3 – 0.602 ϕ4

(D)  Ψ = 0.372 ϕ1 + 0.602 ϕ2 + 0.602 ϕ3 + 0.372 ϕ4

Answer: (B)

14. The activity of ‘m’ molal CuSO­4 solution can be expressed in terms of its mean activity coefficient (γ±) as:

(A)  m2γ±2

(B)  4m3γ±3

(C)  16m4γ±4

(D)  108 m5γ±5

Answer: (A)

15. The character table for a pyramidal AB3 molecule of C3v point group is given below:

The reducible representation of pyramidal AB3 is

The CORRECT option representing all the normal Raman active modes of pyramidal AB3 is :

(A)  A1 + A2 + 2E

(B)  3E

(C)  3A1 + A2 + E

(D)  2A1 + 2E

Answer: (D)

16. In the following reaction,

the number of peaks exhibited by the major product P in its broadband proton decoupled 13C NMR spectrum is_______

Answer: (8.0 to 8.0)

17. Among the following,

the total number of aromatic species is________

Answer: (4.0 to 4.0)

18. The maximum number of microstates of d2 electronic configuration is_________

Answer: (45 to 45)

19. In a uranium recovery process, an aqueous solution of uranyl ion is evaporated, dried in air at 400°C and subsequently reduced with hydrogen at 700°C to obtain a uranium compound (X). The oxidation state of uranium in X is __________

(Given : atomic number of U is 92)

Answer: (4 to 4)

20. For a cubic crystal system, the powder X-ray diffraction pattern recorded using Cu Kα source (λ = 1.54 Å) shows a peak at 33.60° (2θ) for (111) plane. The lattice parameter ‘a’ (in Å, rounded off to two decimal places) is _________

Answer: (4.60 to 4.64)

21. In an NMR spectrometer operating at a magnetic field strength of 16.45 T, the resonance frequency (in MHz, rounded off to one decimal place) of 19F nucleus is ________

(Given : g factor of 19F = 5.255; βN = 5.05 × 1027 J T1; h = 6.626 × 1034 J s)

Answer: (658 to 659)

22. When three moles of helium is mixed with one mole of neon at constant temperature and pressure (25 °C, 1 atm), the entropy of mixing (in J K1, rounded off to two decimal places) is ________

(Given : R = 8.314 J K1 mol1)

Answer: (18.60 to 18.80)

23. At 25°C, the emf(in volts, rounded off to three decimal places) of the cell, Ag | AgBr(s) | Br (a = 0.20), Cu2+ (a = 0.48), Cu+(a = 0.24) | Pt is _______

(Given : The standard emf of the cell is 0.082 V; R = 8.314 J K1 mol1; F = 96500 C mol1)

Answer: (0.058 to 0.060)

24. For an enzyme catalyzed reaction, the plot of inverse of initial rate against inverse of initial substrate concentration is linear with slope 0.16 s and intercept 2.12 mol1 L s. The estimated value of Michaelis constant (in mol L1, rounded off to two decimal places is ________

Answer: (0.07 to 0.08)

25. Fluorescence quantum yield and fluorescence lifetime of a molecule are 0.4 and 5 × 109 s, respectively. If the fluorescence decay rate constant is Y × 107 s1, t he value of Y (rounded off to nearest integer) is _________

Answer: (8 to 8)

Q26 – Q55 carry two marks each.

26. Major product formed in the following reaction sequence is:

Answer: (C)

27. Major products P and Q, in the given reaction sequence, are:

Answer: (B)

28. Major products P and Q, formed in the reactions given below, are:

Answer: (C)

29. A compound with molecular formula C10H12O12 showed a strong IR band at ~ 720 cm1 a peak at m/z 122 in the mass spectrum and the following 1H NMR signals: δ1 – 8.0 (2H, m), 7.6 – 7.5(1H, m), 7.5 – 7.3 (2H, m), 4.3(2H, t), 1.8 (2H sextet) and 1.0 (3H, t). The structure of the compound is:

Answer: (B)

30. Major product formed in the following synthetic sequence is:

Answer: (A)

31. The CORRECT statement with respect to the stereochemistry of α-hydroxy acids P and Q formed in the following reactions is:

(A)  Both P and Q are formed with retention of configuration.

(B)  Both P and Q are formed with inversion of configuration.

(C)  P is formed with retention of configuration and Q with inversion of configuration.

(D)  P is formed with inversion of configuration and Q with retention of configuration.

Answer: (D)

32. The rate of solvolysis of the given compounds is in the order

(A)  T > R > Q > S > P

(B)  Q > T > R > P > S

(C)  R > T > Q > S > P

(D)  T > Q > R > P > S

Answer: (A)

33. In the following reaction sequence, the major products Q and R are:

Answer: (B)

34. In the electronic absorption spectrum of an aqueous solution of [Ni(NH3)6]2+, a very weak band is observed between the bands due to the transitions 3A2g3T2g and 3A2g3T1g(F). The transition responsible for the very weak band is

(Given: atomic number of Ni is 28)

(A)  3A2g1T1g

(B)  3A2g1T2g

(C)  3A2g1Eg

(D)  3A2g1A2g

Answer: (C)

35. The experimental magnetic moment (3.4 BM) of a hydrated salt of Eu3+ at 27°C is significantly different from the calculated value. The difference is due to

(Given: atomic number of Eu is 63)

(A)  population of electrons at higher J level(s) via thermal excitation.

(B)  strong ligand field splitting of f-orbitals.

(C)  strong spin-orbit coupling.

(D)  pairing of electrons in f-orbitals.

Answer: (A)

36. The CORRECT combination of L1 and L2 among H, NO, MeCH2 and CO, that will satisfy the 18 electron rule for both metal centers in the following neutral molecule, is

(Given: atomic number of Ru is 44)

(A)  H, NO

(B)  MeCH2, NO

(C)  MeCH2, CO

(D)  H, CO

Answer: (C)

37. In the following reaction sequence,

the structure of B is

(Given: atomic number of Mo is 42)

Answer: (D)

38. The following table lists the reaction/conversion catalyzed by metalloenzymes.

The correct combination is

(A)  P-II; Q-I; R-III; S-IV

(B)  P-IV; Q-III; R-II; S-I

(C)  P-II; Q-III; R-IV; S-I

(D)  P-I; Q-IV; R-III; S-II

Answer: (C)

39. The fission of  with thermal neutron is represented below.

 are the primary fission fragment pair, which undergo series of radioactive decay to form stable nuclei X3 and X4 (chain enders). The X3 and Y4, respectively are:

(A)   

(B)  

(C)   

(D)   

Answer: (B)

40. The CORRECT ‘voltage (E) versus time’ excitation signal used in cyclic voltammetry is

Answer: (C)

41. The hydrogen-like radial wave function of the 3s orbital is given as

where ρ = 2Zr/a0; Z = atomic number, r = distance from the nucleus and a0 = Bohr radius. Positions of the radial nodes (in  units of a0) of the 3s orbital are at

(A)    

(B)   

(C)   

(D)   

Answer: (C)

42. ∆Gf0 and ∆Hf0 for Fe(g) are 370.7 kJ mol1 and 416.3 kJ mol1 at 298 K, respectively. Assuming ∆Hf0 is constant in the interval 250 K to 375 K, ∆Gf0 (rounded off to the nearest integer) for Fe(g) at 375 K is:

(A)  359 kJ mol1

(B)  338 kJ mol1

(C)  325 kJ mol1

(D)  310 kJ mol1

Answer: (A)

43. Adsorption of N2 on TiO2 was carried out at 75 K. A plot of  gives a straight line with an intercept, 4.0 × 106 mm3 and slope, 1.0 × 103 mm3. The volume (rounded off to the nearest integer) corresponding to the monolayer coverage is:

(A)  996 mm3

(B)  785 mm3

(C)  690 mm3

(D)  555 mm3

Answer: (A)

44. Among the following sets,

the total number of set(s) of diastereomeric pair(s) is _______

Answer: (4.0 to 4.0)

45. Among the following.

the total number of compounds showing characteristic carbonyl stretching frequency less than 1700 cm1 in their IR spectra is _______

Answer: (3.0 to 3.0)

46. Consider the AgX crystallizes in rock salt structure. The density of AgX is 6477 kg/m3 and unit cell length is 577.5 pm. Atomic weight of Ag is 107.87 g mol1. The atomic weight of X (in g mol1, rounded off to two decimal places) is _______

Answer: (79.00 to 81.00)

47. The total number of g|| lines expected in the EPR spectrum of a solution of bis(salicylaldimine) copper(II) having pure 63Cu and 14N at 77 K is ________

(Given: I values of 63Cu, 14N and 1H are 3/2, 1 and 1/2, respectively)

Answer: (60 to 60)

48. Among the following,

[B12H12]2, [Ni5(CO)­12]2, [C2B9H11]2, Rh6(CO)16, Os6(CO)20, B5H11, B6H10 the total number of species having nido structure is________

(Given : atomic numbers of H, B, C, O, Ni, Rh and Os are 1, 5, 6, 8, 28, 45 and 76, respectively)

Answer: (3 to 3)

49. The frequency (in cm1, rounded off to two decimal places) for pure rotational line in the spectrum of NO molecule due to change in the quantum number from J = 1 to J = 2 is _______

(Given: Moment of inertia of NO = 1.6427 × 1046 kg m2; h = 6.626 × 1034 J s; c = 3 × 108 m/s)

Answer: (6.80 to 6.82)

50. The % error (rounded off to two decimal places) in the ground state energy of a particle in a one dimensional box of length ‘a’ described by a trial variation function φ = x(a – x), where 0 ≤ x ≤ a, is ________

(Given: The true ground state energy of the above system is h2/8ma2

Answer: (1.22 to 1.44)

51. Assuming no interaction between vibrational and rotational energy levels in HF, the frequency (in cm1, rounded off to the nearest integer) of the R branch line originating from J = 4 in its IR spectrum is ________

(Given: Rotational constant of HF = 19.35 cm1  

Answer: (4332 to 4332)

52. The van der Waals constants a and b for gaseous CO are given as 1.49 L2 atm mol2 and 0.0399 L mol1, respectively. The fugacity (in atm, rounded off to two decimal places) of CO at 35°C and 95 atm is ______

(Given: R = 0.082 L atm K−1 mol−1)

Answer: (88.11 to 88.60)

53. At 30°C, the vapor pressure and density of a 1.0 M aqueous solution of sucrose are 31.207 mm Hg and 1.1256 g/mL, respectively. If the vapor pressure of pure water at 30°C is 31.824 mm Hg, the activity coefficient (rounded off to three decimal places) of water in the given solution is _______

(Given: The molar mass of sucrose = 342.3 g mol1)

Answer: (1.002 to 1.004)

54. For the ring opening reaction of cyclopropane to propene at 25°C, the pre-exponential factor is 4.3 × 1015 s1. The entropy of activation (in J K1 mol1, rounded off to two decimal places) is _______.

(Given: h = 6.626 × 1034 J s; kB = 1.38 × 1023 J K1; R = 8.314 J K1 mol1)

Answer: (46.00 to 46.10)

55. In a reaction, reactant X is converted to products Y and Z consecutively with rate constants 6.0 × 102 min1 and 9.0 × 103 min1, respectively. If the initial amount of X is 12.5 moles, the number of moles (rounded off to one decimal place) of Y formed after 10 minutes is ________

Answer: (5.3 to 5.5)

GATE Exam 2020 Computer Science and Information Technology (CS) Question Paper With Answer Key

GATE-2020

CS: Computer Science and Information Technology

GA-General Aptitude

Q1 – Q5 carry one mark each.

1. Raman is confident of speaking English ______ six months as he has been practicing regularly ________ the last three weeks.

(A)  during, for

(B)  for, since

(C)  for, in

(D)  within, for

Answer: (D)

2. His knowledge of the subject was excellent but his classroom performance was _______.

(A)  extremely poor

(B)  good

(C)  desirable

(D)  praiseworthy

Answer: (A)

3. Select the word that fits the analogy:

Cook : Cook :: Fly :_____

(A)  Flyer

(B)  Flying

(C)  Flew

(D)  Flighter

Answer: (A)

4. The drawn of the 21st century witnessed the melting glaciers oscillating between giving too much and too little to billions of people who depend on them for fresh water. The UN climate report estimates that without deep cuts to man-made emissions, at least 30% of the northern hemisphere’s surface permafrost could melt by the end of the century. Given this situation of imminent global exodus of billions of people displaced by rising seas, nation-states need to rethink their carbon footprint for political concerns, if not for environmental ones.

Which one of the following statements can be inferred from the given passage?

(A)  Nation-states do not have environmental concerns.

(B)  Nation-states are responsible for providing fresh water to billions of people.

(C)  Billions of people are responsible of man-made emissions.

(D)  Billions of people are affected by melting glaciers.

Answer: (D)

5. There are multiple routes to reach from node 1 to node 2, as shown in the network.

The cost of travel on an edge between two nodes is given in rupees. Nodes ‘a’, ‘b’, ‘c’, ‘d’, ‘e’, and ‘f’ are toll booths. The toll price at toll booths marked ‘a’ and ‘e’ is Rs. 200, and is Rs.100 for the other toll booths. Which is the cheapest route from node 1 to node 2?

(A)  1-a-c-2

(B)  1-f-b-2

(C)  1-b-2

(D)  1-f-e-2

Answer: (B)

Q6 – Q10 carry two marks each.

6. Goods and Services Tax (GST) is an indirect tax introduced in India in 2017 that is imposed on the supply of goods and services, and it subsumes all indirect taxes except few. It is a destination-based tax imposed on goods and services used, and it is not imposed at the point of origin from where goods come. GST also has a few components specific to state governments, central government and Union Territories (UTs).

Which one of the following statements can be inferred from the given passage?

(A)  GST is imposed on the production of goods and services.

(B)  GST includes all indirect taxes.

(C)  GST does not have a component specific to UT.

(D)  GST is imposed at the point of usage of goods and services.

Answer: (D)

7. If P = 3, R = 27, T = 243, then Q + S = _____.

(A)  40

(B)  80

(C)  90

(D)  110

Answer: (C)

8. The figure below shows an annular ring with outer and inner radii as b and a, respectively. The annular space has been painted in the form of blue colour circles touching the outer and inner periphery of annular space. If maximum n number of circles can be painted, then the unpainted area available in annular space is ________.

(A)   

(B)  π[(b2 – a2) – n(b – a)2]

(C)   

(D)  π[(b2 – a2) + n(b – a)2]

Answer: (A)

9. Two straight lines are drawn perpendicular to each other in X-Y plane. If α and β are the acute angles the straight lines make with t he X-axis, then α + β is _______.

(A)  60°

(B)  90°

(C)  120°

(D)  180°

Answer: (B)

10. The total revenue of a company during 2014-2018 is shown in the bar graph. If the total expenditure of the company in each year is 500 million rupees, then the aggregate profit or loss (in percentage) on the total expenditure of the company during 2014-2018 is ______.

(A)  16.67% profit

(B)  16.67% loss

(C)  20% profit

(D)  20% loss

Answer: (C)

CS: Computer Science and Information Technology

Q1 – Q25 carry one mark each.

1. Consider the functions

(I) ex

(II) x2 – sin x

(III)  

Which of the above functions is/are increasing everywhere in [0, 1]?

(A)  III only

(B)  II only

(C)  II and III only

(D)  I and III only

Answer: (A)

2. For parameters a and b, both of which are ω(1), T(n) = T(n1/a) + 1, and T(b) = 1.

Then T(n) is

(A)  Θ(loga logb n)

(B)  Θ(logab n)

(C)  Θ(logb loga n)

(D)  Θ(log2 log2 n)

Answer: (A)

3. Consider the following statements.

(I) Daisy chaining is used to assign priorities in attending interrupts.

(II) When a device raises a vectored interrupt, the CPU does polling to identify the source of interrupt.

(III) In polling, the CPU periodically checks the status bits to know if any device needs its attention.

(IV) During DMA, both the CPU and DMA controller can be bus masters at the same time.

Which of the above statements is/are TRUE?

(A)  I and II only

(B)  I and IV only

(C)  I and III only

(D)  III only

Answer: (C)

4. Consider the following data path diagram.

Consider an instruction: R0 ← R1 + R2. The following steps are used to execute it over the given data path. Assume that PC is incremented appropriately. The subscripts r and w indicated read and write operations, respectively.

(1) R2r, TEMP1r, ALUadd, TEMP2w

(2) R1r, TEMP1w

(3) PCr, MARw, MEMr

(4) TEMP2r, R0w

(5) MIDR­r­, IRw

which one of the following is the correct order of execution of the above steps?

(A)  2, 1, 4, 5, 3

(B)  1, 2, 3, 4, 5

(C)  3, 5, 2, 1, 4

(D)  3, 5, 1, 2, 4

Answer: (C)

5. The preorder traversal of a binary search tree is 15, 10, 12, 11, 20, 18, 16, 19.

Which one of the following is the postorder traversal of the tree?

(A)  10, 11, 12, 15, 16, 18, 19, 20

(B)  11, 12, 10, 16, 19, 18, 20, 15

(C)  20, 19, 18, 16, 15, 12, 11, 10

(D)  19, 16, 18, 20, 11, 12, 10, 15

Answer: (B)

6. What is the worst case time complexity of inserting n2 elements into an AVL-tree with n elements initially?

(A)  Θ(n4)

(B)  Θ(n2)

(C)  Θ(n2 log n)

(D)  Θ(n3)

Answer: (C)

7. Which one of the following regular expressions represents at set of all binary strings with an odd number of 1’s?

(A)  ((0 + 1)*1(0 + 1)*1)*10*

(B)  (0*10*10*)*0*1

(C)  10*(0*10*10*)*

(D)  (0*10*10*)*10*

Answer: (MTA)

8. Consider the following statements.

(I) If L1 ⋃ L2 is regular, then both L1 and L2 must be regular.

(II) The class of regular languages is closed under infinite union.

Which of the above statement is/are TRUE?

(A)  I only

(B)  II only

(C)  Both I and II

(D)  Neither I nor II

Answer: (D)

9. Consider the following statements.

(I) Symbol table is accessed only during lexical analysis and syntax analysis.

(II) Compilers for programming languages that support recursion necessarily need heap storage for memory allocation in the run-time environment.

(III) Errors violating the condition ‘any variable must be declared before its use’ are detected during syntax analysis.

Which of the above statements is/are TRUE?

(A)  I Only

(B)  I and III only

(C)  II only

(D)  None of I, II and III

Answer: (D)

10. Consider the language L = {an | n ≥ 0} ⋃ {anbn | n ≥ 0} and the following statements.

(I) L is deterministic context-free.

(II) L is context-free but not deterministic context-free.

(III) L is not LL(k) for any k.

Which of the above statements is/are TRUE?

(A)  I only

(B)  II only

(C)  I and III only

(D)  III only

Answer: (C)

11. Consider allocation of memory to a new process. Assume that none of the existing holes in the memory will exactly fit the process’s memory requirement. Hence, a new hole of smaller size will be created if allocation is made in any of the existing holes. Which one of the following statements is TRUE?

(A)  The hole created by first fit is always larger than the hole created by next fit.

(B)  The hole created by worst fit is always large than the hole created by first fit.

(C)  The hole created by best fit is never larger than the hole created by first fit.

(D)  The hole created by next fit is never larger than the hole created by best fit.

Answer: (C)

12. Consider the following statements about process state transitions for a system using preemptive scheduling.

(I) A running process can move to ready state.

(II) A ready process can move to running state.

(III) A blocked process can move to running state.

(IV) A blocked process can move to ready state.

Which of the above statements are TRUE?

(A)  I, II, and III only

(B)  II and III only

(C)  I, II and IV only

(D)  I, II, III, and IV

Answer: (C)

13. Consider a relational database containing the following schemas.

The primary key of each table is indicated by underlining the constituent fields.

The number of rows returned by the above SQL query is

(A)  4

(B)  5

(C)  0

(D)  2

Answer: (A)

14. Which of the following is used to represent the supporting many-one relationship of a weak entity set in an entity-relationship diagram?

(A)  Diamonds with double/bold border

(B)  Rectangles with double/bold border

(C)  Ovals with double/bold border

(D)  Ovals that contain underlined identifiers

Answer: (A)

15. Consider the following statements about the functionality of an IP based router.

(I) A router does not modify the IP packets during forwarding.

(II) It is not necessary for a router to implement any routing protocol.

(III) A router should reassemble IP fragments if the MTU of the outgoing link is larger than the size of the incoming IP packet.

Which of the above statement is/are TRUE?

(A)  I and II only

(B)  I only

(C)  II and III only

(D)  II only

Answer: (D)

16. What is the worst case time complexity of inserting n elements into an empty linked list, if the linked list needs to be maintained in sorted order?

(A)  Θ(n)

(B)  Θ(n log n)

(C)  Θ(n2)

(D)  Θ(1)

Answer: (C)

17. Let R be the set of all binary relations on the set {1, 2, 3}. Suppose a relation is chosen from R at random. The probability that the chosen relation is reflexive (round off to 3 decimal places) is _______.

Answer: (0.125 to 0.125)

18. Let G be a group of 35 elements. Then the largest possible size of a subgroup of G other than G itself is _______.

Answer: (7 to 7)

19. A multiplexer is placed between a group of 32 registers and an accumulator to regulate data movement such that at any given point in time the content of only one register will move to the accumulator. The minimum number of select lines needed for the multiplexer is

Answer: (5 to 5)

20. If there are m input lines and n output lines for a decoder that is used to uniquely address a byte addressable 1 KB RAM, then the minimum value of m + n is _______.

Answer: (1034 to 1034)

21. A direct mapped cache memory of 1 MB has a block size of 256 bytes. The cache has an access time of 3 ns and a hit rate of 94%. During a cache miss, it takes 20 ns to bring the first word of a block from the main memory, while each subsequent word takes 5 ns. The word size is 64 bits. The average memory access time in ns (round off to 1 decimal place) is _______.

Answer: (13.3 to 13.3 OR 13.5 to 13.5)

22. Consider the following C program.

The output of the program is ________

Answer: (19 to 19)

23. Consider a double hashing scheme in which the primary hash function is h1(k) = k mod 23, and the secondary hash function is h2(k) = 1 + k(mod 19). Assume that the table size is 23. Then the address returned by probe 1 in the probe sequence (assume that the probe sequence begins at probe 0) for key value k = 90 is _______.

Answer: (13 to 13)

24. Consider the following grammar.

S → aSB | d

B → b

The number of reduction steps taken by a bottom-up parser while accepting the string aaadbbb is ______.

Answer: (7 to 7)

25. Assume that you have made a request for a web page through your web browser to a web server. Initially the browser cache is empty. Further, the browser is configured to send HTTP requests in non-persistent mode. The web page contains text and five very small images. The minimum number of TCP connections required to display the web page completely in your browser is ________.

Answer: (6 to 6)

Q26 – Q55 carry two marks each.

26. Which of the following languages are undecidable? Note that  indicates encoding of the Turing machine M.

(A)  L1, L3, and L4 only

(B)  L1 and L3 only

(C)  L2 and L3 only

(D)  L2, L3 and L4 ony

Answer: (A)

27. Let A and B be two n × n matrices over real numbers. Let rank(M) and det(M) denote that rank and determinant of a matrix M, respectively. Consider the following statements.

Which of the above statements are TRUE?

(A)  I and II only

(B)  I and IV only

(C)  II and III only

(D)  III and IV only

Answer: (C)

28. Consider the Boolean function z(a, b, c).

Which one of the following minterm list represents the circuit given above?

(A)  z = ∑ (0, 1, 3, 7)

(B)  z = ∑ (1, 4, 5, 6, 7)

(C)  z = ∑ (2, 4, 5, 6, 7)

(D)  z = ∑ (2, 3, 5)

Answer: (B)

29. Consider three registers R1, R2 and R3 that store numbers in IEEE-754 single precision floating point format. Assume that R1 and R2 contain the values (in hexadecimal notation) 0x42200000 and 0xC1200000, respectively.

If  what is the value stored in R3?

(A)  0x40800000

(B)  0xC0800000

(C)  0x83400000

(D)  0xC8500000

Answer: (B)

30. A computer system with a word length of 32 bits has a 16 MB byte-addressable main memory and a 64 KB, 4-way set associative cache memory with a block size of 256 bytes. Consider the following four physical addresses represented in hexadecimal notation.

A1 = 0x42C8A4, A2 = 0x546888, A3 = 0x6A289C, A4 = 0x5E4880

Which one of the following is TRUE?

(A)  A1 and A4 are mapped to different cache sets.

(B)  A2 and A3 are mapped to the same cache set.

(C)  A3 and A4 are mapped to the same cache set.

(D)  A1 and A3 are mapped to the same cache set.

Answer: (B)

31. Let G = (V, E) be a weighted undirected graph and let T be a Minimum Spanning Tree (MST) of G maintained using adjacency lists. Suppose a new weighted edge (u, v) ∈ V × V is added to G. The worst case time complexity of determining if T is still an MST of the resultant graph is

(A)  Θ(|E| + |V|)

(B)  Θ(|E| |V|)

(C)  Θ(|E| log|V|)

(D)  Θ(|V|)

Answer: (D)

32. Consider the following languages.

L1 = {wxyx, |w, x, y ∈ (0 + 1)+}

L2 = {xy | x, y ∈ (a +b)*, |x| = |y|, x ≠ y}

Which one of the following is TRUE?

(A)  L1 is regular and L2 is context-free

(B)  L1 is context-free but not regular and L2 is context-free.

(C)  Neither L1 nor L2 is context-free.

(D)  L1 is context-free but L2 is not context-free.

Answer: (A)

33. Consider the productions A → PQ and A → Each of the five non-terminals A, P, Q, X, and Y has two attributes: s is a synthesized attribute, and i is an inherited attribute. Consider the following rules.

Rule 1 : P. i = A.i + 2, Q.i = P.i + A.i, and A.s = P.s + Q.s

Rule 2: X.i = A.i + Y.s and Y.i = X.s + A.i

Which one of the following is TRUE?

(A)  Both Rule 1 and Rule 2 are L-attributed.

(B)  Only Rule 1 is L-attributed.

(C)  Only Rule is L-attributed.

(D)  Neither Rule 1 nor Rule 2 is L-attributed.

Answer: (B)

34. Each of a set of n processes executes the following code using two semaphores a and b initialized to 0 and not used in CODE SECTION P.

What does the code achieve?

(A)  It ensures that no process executes CODE SECTION Q before every process has finished CODE SECTION P.

(B)  It ensures that at most two processes are in CODE SECTION Q at any time.

(C)  It ensures that all processes execute CODE SECTION P mutually exclusively.

(D)  It ensures that at mot n – n1 processes are in CODE SECTION P at any time.

Answer: (A)

35. Consider the following five disk access requests of the form (request id, cylinder number) that are present in the disk scheduler queue at a given time.

(P, 155), (Q, 85), (R, 110), (S, 30), (T, 115)

Assume the head is positioned at cylinder 100. The scheduler follows Shortest Seek Time First scheduling to service the requests.

Which one of the following statements is FALSE?

(A)  T is serviced before P.

(B)  Q is serviced after S, but before T.

(C)  The head reverses its direction of movement between servicing of Q and P.

(D)  R is serviced before P.

Answer: (B)

36. Consider a relational table R that is in 3NF, but not in BCNF. Which one of the following statements is TRUE?

(A)  R has a nontrivial functional dependency X → A, where X is not a superkey and A is a prime attribute.

(B)  R has nontrivial functional dependency X → A, where X  is not superkey and A is non-prime attribute and X is not a proper subset of any key.

(C)  R has a nontrivial functional dependency X → A, where X is not a superkey and A is a non-prime attribute and X is proper subset of some key.

(D)  A cell in R holds a set instead of an atomic value.

Answer: (A)

37. Consider a schedule of transactions T1 and T2:

Here, RX stands for “Read (X)” and WX stands for “Write(X)”. Which one of the following schedules is conflict equivalent to the above schedule?

Answer: (A)

38. An organization requires a range of IP addresses to assign one to each of its 1500 computers. The organization has approached an Internet Service Provider (ISP) for this task. The ISP uses CIDR and serves the requests from the available IP address space 202.61.0.0/17. The ISP wants to assign an address space to the organization which will minimize the number of routing entries in the ISP’s router using route aggregation. Which of the following address spaces are potential candidates from which the ISP can allot any one to the organization?

(I) 202.61.84.0/21

(II) 202.61.104.0/21

(III) 202.61.64.0/21

(IV) 202.61.144.0/21

(A)  I and II only

(B)  II and III only

(C)  III and IV only

(D)  I and IV only

Answer: (B)

39. Which one of the following predicate formulae is NOT logically valid?

Note that W is a predicate formula without any free occurrence of x.

Answer: (C)

40. Let G = (V, E) be a directed, weighted graph with weight function w: E → ℝ. For some function f : V → ℝ, for each edge (u, v) ∈ E, define w’(u, v) as w(u, v) + f(u) – f(v).

Which one of the options completes the following sentence so that it is TRUE?

“The shortest paths in G under w are shortest paths under w’, too, _______”.

(A)  for every f : V → ℝ

(B)  if and only if ∀u ∈ V, f(u) is positive

(C)  if and only if ∀u ∈V, f(u) is negative

(D)  If and only if f(u) is the distance from s to u in the graph obtained by adding a new vertex s to G and edges of zero weight from s to every vertex of G

Answer: (A)

41. In an balanced binary search tree with n elements, what is the worst case time complexity of reporting all elements in range [a, b]? Assume that the number of reported elements is k.

(A)  Θ(log n)

(B)  Θ(log n + k)

(C)  Θ(k log n)

(D)  Θ(n log k)

Answer: (B)

42. The number of permutations of the characters in LILAC so that no character appears in its original position, if the two L’s are indistinguishable, is ______.

Answer: (12 to 12)

43. Consider a non-pipelined processor operating at 2.5 GHz. It takes 5 clock cycles to complete an instruction. You are going to make a 5-stage pipeline out of this processor. Overheads associated with pipelining force you to operate the pipelined processor at 2 GHz. In a given program, assume that 30% are memory instructions, 60% are ALU instructions and the rest are branch instructions. 5% of the memory instructions cause stalls of 50 clock cycles each due to cache misses and 50% of the branch instructions cause stalls of 2 cycles each. Assume that there are not stalls associated with the execution of ALU instructions. For this program, the speedup achieved by the pipelined processor over the non-pipeline processor (round off to 2 decimal places) is ______.

Answer: (2.15 to 2.18)

44. A processor has 64 registers and uses 16-bit instruction format. It has two types of instructions: I-type and R-type. Each I-type instruction contains an opcode, a register name, and a 4-bit immediate value. Each R-type instruction contains an opcode and two register names. If there are 8 distinct I-type opcodes, then the maximum number of distinct R-type opcodes is _______.

Answer: (14 to 14)

45. For n > 2, let a ∈ {0, 1}n be a non-zero vector. Suppose that x is chosen uniformly at random from {0, 1}n. Then the probability that  is an odd number is ______.

Answer: (0.5 to 0.5)

46. Consider the following C functions.

The return value of fun 2 (5) is ______.

Answer: (55 to 55)

47. Consider the array representation of a binary min-heap containing 1023 elements. The minimum number of comparisons required to find the maximum in the heap is ______.

Answer: (511 to 511)

48. Consider the following C functions.

The value returned by pp(3, 4) is _____.

Answer: (81 to 81)

49. Consider a graph G = (V, E), where V = {v1, v2 ….., v100}, E = {(vi, vj) |1 ≤ i < j ≤ 100}, and weight of the edge (vi, vj) is |i – j|. The weight of minimum spanning tree of G is ______.

Answer: (99 to 99)

50. Consider the following set of processes, assumed to have arrived at time 0. Consider the CPU scheduling algorithms Shortest Job First (SJF) and Round Robin (RR). For RR, assume that the processes are scheduled in the order P1, P2, P3, P4.

If the time quantum for RR is 4 ms, then the absolute value of the difference between the average turnaround times (in ms) of SJF and RR (round off to 2 decimal places) is ______.

Answer: (5.25 to 5.25)

51. Consider the following language.

L = {x ∈ {a, b}* | number of a’s in x is divisible by 2 but not divisible by 3}

The minimum number of states in a DFA that accepts L is ______.

Answer: (6 to 6)

52. Graph G is obtained by adding vertex s to K3, 4 and making s adjacent to every vertex of K3,4. The minimum number of colours required to edge-colour G is ______.

Answer: (7 to 7)

53. Consider a paging system that uses 1-level page table residing in main memory and a TLB for address translation. Each main memory access takes 100 ns and TLB lookup takes 20 ns. Each page transfer to/from the disk takes 5000 ns. Assume that the TLB hit ratio is 95%, page fault rate is 10%. Assume that for 20% of the total page faults, a dirty page has to be written back to disk before the required page is read in from disk. TLB update time is negligible. The average memory access time in ns(round of to 1 decimal places) is ______.

Answer: (154.5 to 155.5)

54. Consider a database implemented using B+ tree for file indexing and installed on a disk drive with block size of 4 KB. The size of search key is 12 bytes and the size of tree/disk pointer is 8 bytes. Assume that the database has one million records. Also assume that no node of the B+ tree and no records are present initially in main memory. Consider that each record fits into one disk block. The minimum number of disk accesses required to retrieve any record in the database is ________.

Answer: (4 to 4)

55. Consider a TCP connection between a client and a server with the following specifications: the round trip time is 6 ms, the size of the receiver advertised window is 50 KB, slow-start threshold at the client is 32 KB, and the maximum segment size is 2 KB. The connection is established at time t = 0. Assume that there are no timeouts and errors during transmission. Then the size of the congestion window (in KB) at time t + 60 ms after all acknowledgements are processed is _______.

Answer: (44 to 44)

GATE Exam 2020 Chemical Engineering (CH) Question Paper With Answer Key

GATE-2020

CH: Chemical Engineering

GA-General Aptitude

Q1-Q5 carry one mark each.

1. Rajiv Gandhi Khel Ratna Award was conferred _____Mary Kom, a six-time world champion in boxing, recently in a ceremony _____ the Rashtrapati Bhawan (the President’s official residence) in New Delhi.

(A)  with, at

(B)  on, in

(C)  on, at

(D)  to, at

Answer: (C)

2. Despite a string of a poor performances, the changes of K. L. Rahul’s selection in the team are ______.

(A)  slim

(B)  bright

(C)  obvious

(D)  uncertain

Answer: (B)

3. Select the word that fits the analogy:

Cover : Uncover :: Associate : _______

(A)  Unassociate

(B)  Inassociate

(C)  Missassociate

(D)  Dissociate

Answer: (D)

4. Hig by floods, he kharif (summer sown) crops in various parts of the county have been affected. Officials believe that the loss in production of the kharif crops can be recovered in the output of the rabi (winter sown) crops so that the country can achieve its food-grain production target of 291 million tons in the crop year 2019-20 (July-June). They are hopeful that good rains in July-August will help the soil retain moisture for a longer period, helping winter sown crops such as wheat and pulses during the November-February period.

Which of the following statements can be inferred from the given passage?

(A)  Officials declared that the food-grain production target will be met due to good rains.

(B)  Officials want the food-grain production target to be met by the November-February period.

(C)  Officials feel that the food-grain production target cannot be met due to floods.

(D)  Officials hope that the food-grain production target will be met due to a good rabi produce.

Answer: (D)

5. The difference between the sum of the first 2n natural numbers and the sum of the first n odd natural numbers is ______.

(A)  n2 – n

(B)  n2 + n

(C)  2n2 – n

(D)  2n2 + n

Answer: (B)

Q6-Q10 carry two marks each.

6. Repo rate is the at which Reserve Bank of India (RBI) lends commercial banks, and reverse repo rate is the rate at which RBI borrows money from commercial banks.

Which of the following statements can be inferred from the above passage?

(A)  Decrease in repo rate will increase cost of borrowing and decrease lending by commercial banks.

(B)  Increase in repo rate will decrease cost of borrowing and increase lending by commercial banks.

(C)  Increase in repo rate will decrease cost of borrowing and decrease lending by commercial banks.

(D)  Decrease in repo rate will decrease cost of borrowing and increase lending by commercial banks.

Answer: (D)

7. P, Q, R, S, T, U, V, and W are seated around a circular table.

(I) S is seated opposite to W.

(II) U is seated at the second place to the right of R.

(III) T is seated at the third place to the left of R.

(IV) V is a neighbor of S.

Which of the following must be true?

(A)  P is a neighbor of R.

(B)  Q is a neighbor of R.

(C)  P is not seated opposite to Q.

(D)  R is the left neighbor of S.

Answer: (C)

8. The distance between Delhi and Agra is 233 km. A car P started travelling from Delhi to Agra and another car Q started from Agra to Delhi along the same road 1 hour after the car P started. The two cars crossed each other 75 minutes after the car Q started. Both cars were travelling at constant speed. The speed of car P was 10 km/hr more than the speed of car Q. How many kilometers the car Q had travelled when the cars crossed each other?

(A)  66.6

(B)  75.2

(C)  88.2

(D)  116.5

Answer: (B)

9. For a matrix M = [mij], i. j= 1, 2, 3, 4, the diagonal elements are all zero and mij = −mij. The minimum number of elements required to fully specify the matrix is_______.

(A)  0

(B)  6

(C)  12

(D)  16

Answer: (B)

10. The profit shares of two companies P and Q are shown in the figure. If the two companies have invested a fixed and equal amount every year, then the ratio of the total revenue of company P to the total revenue of company Q, during 2013-2018 is ______.

(A)  15 : 17

(B)  16 : 17

(C)  17 : 15

(D)  17 : 16

Answer: (B)

Chemical Engineering

Q1 – Q26 carry one mark each.

1. Which one of the following methods requires specifying an initial interval containing the root (i.e., bracketing) to obtain the solution of f(x) = 0, where f(x) is a continuous non-linear algebraic function?

(A)  Newton-Raphson method

(B)  regula falsi method

(C)  secant method

(D)  fixed point iteration method

Answer: (B)

2. Consider the hyperbolic function s in Group-1 and their definitions in Group-2.

The correct combination is

(A)  P-IV, Q-I, R-III, S-II

(B)  P-II, Q-III, R-I, S-IV

(C)  P-IV, Q-I, R-II, S-III

(D)  P-I, Q-II, R-IV, S-III

Answer: (C)

3. Consider the following continuously differentiable function v(x, y, z) = 3x2yi + 8y2zj + 5xyzk where i, j and k represent the respective unit vectors along the x, y, and z directions in the Cartesian coordinate system. The curl of this function is

(A)  −3x2i – 8y2j + 5z(x + y) k

(B)  6xyi – 16y j + 5xy k

(C)  (5xz – 8y2)I – 5yzj – 3x2k

(D)  y(11x + 16z)

Answer: (C)

4. Consider the following unit step function.

The Laplace transform of this function is

(A)  e3s/S

(B)  e3s/S2

(C)  e3s/3S

(D)  e6s/S

Answer: (A)

5. In a constant-pressure cake filtration with an incompressible cake layer, volume of the filtrate (V) is measured as a function of time t. The plot of t/V versus V results in a straight line with an intercept of 104 s m3. Area of the filter is 0.05 m2, viscosity of the filtrate is 103 Pa, s and the overall pressure drop across the filter is 200 kPa. The value of filter-medium resistance (in m1) is

(A)  1 × 109

(B)  1 × 1010

(C)  1 × 1011

(D)  1 × 1012

Answer: (C)

6. In a laboratory experiment, a unit pulse input of tracer is given to an ideal plug flow reactor operating at steady state with a recycle ratio, R = 1. The exit age distribution, E(t), of the tracer at the outlet of the reactor is measured. The first four pulses observed at t1, t2, t3, and t4 are shown below.

In addition, use the following data and assumptions

• R is defined as ratio of the volume of fluid returned to the entrance of the reactor to the volume leaving the system

• No reaction occurs in the reactor

• Ignore any dead volume in the recycle loop

If the space time of the plug flow reactor is τ seconds, which one of the following is correct?

Answer: (B)

7. The square of Thiele modulus, MT, is given by  where L is the characteristic length of the catalyst pellet, k is the rate constant of a first order reaction, and Deff is the effective diffusivity of the species in the pores. MT2 is a measure of

Answer: (A)

8. Hot-wire anemometer is used for the measurement of

(A)  composition

(B)  flow

(C)  pressure

(D)  temperature

Answer: (B)

9. Pure gas P is being absorbed into a liquid, The dissolved P undergoes an irreversible reaction in the liquid film. The reaction is first order with respect to P. Which one of the following represents the concentration profile of P in the liquid film at steady state?

Answer: (B)

10. Consider a batch distillation process for a equimolar mixture of benzene and toluene atmospheric pressure. The mole fraction of benzene in the distillate collected after 10 minutes is 0.6. The process is further continued for additional 10 minutes. The mole fraction of benzene in the total distillate collected after 20 minutes of operation is

(A)  less than 0.6

(B)  exactly equal to 0.6

(C)  exactly equal to 0.7

(D)  greater than 0.7

Answer: (A)

11. Which one of the following is NOT CORRECT?

(A)  nylon-6, 6 is produced by condensation polymerization

(B)  phenol-formaldehyde resin is a thermosetting polymer

(C)  high density polyethylene (HDPE) is produced by condensation polymerization

(D)  poly (ethylene terephthalate) (PET) is a polyester

Answer: (C)

12. The operating temperature range for the Haber process is 350-500° It is used for the production of ammonia at

(A)  20 MPa using Fe catalyst in an exothermic reaction

(B)  0.1 MPa using Fe catalyst in an exothermic reaction

(C)  20 MPa using Fe catalyst in an endothermic reaction

(D)  20 MPa using zeolie catalyst in an endothermic reaction

Answer: (A)

13. Consider the refinery processes in Group-1 and the catalysts in Group-2.

The correct combination is

(A)  P-II, Q-I, R-III

(B)  P-III, Q-II, R-I

(C)  P-III, Q-I, R-II

(D)  P-I, Q-III, R-II

Answer: (C)

14. Consider the processes in Group-1 and the reactions in Group-2.

The correct combination is

(A)  P-II, Q-I, R-III

(B)  P-III, Q-II, R-I

(C)  P-III, Q-I, R-II

(D)  P-I, Q-III, R-II

Answer: (B)

15. Annual capacity of a plant producing phenol is 100 metric tons. Phenol sells at INR 200 per kg, and its production cost is INR 50 per kg. The sum of annual fixed charges, overhead costs and general expenses is INR 30,00,000. Taxes are payable at 18% on gross profit. Assuming the plant runs at full capacity and that all the phenol produced is sold, the annual net profit of the plant (in INR) is

(A)  1,39,40,000

(B)  1,50,00,000

(C)  98,40,000

(D)  1,20,00,000

Answer: (C)

16. A rigid spherical particle undergoes free settling in a liquid of density 750 kg m−3 and viscosity 9.81 × 10−3 Pa s. Density of the particle is 3000 kg m−3 and the particle diameter is 2 × 10−4 Acceleration due to gravity is 9.81 m s−2. Assuming Stoke’s law to be valid, the terminal settling velocity (in ms−1) of the particle is

(A)  2 × 10−3

(B)  3 × 10−3

(C)  4 × 10−3

(D)  5 × 10−3

Answer: (D)

17. Consider an compressible flow of a constant property fluid over a smooth, thin and wide flat plate. The free stream flows parallel to the surface of the plate along its length and its velocity is constant. Value of the Reynolds number at a distance of 2.0 m from the leading edge of the plate is 8000. The flow within the boundary layer at a distance of 1.0 m from the leading edge of the plate is

(A)  laminar

(B)  turbulent

(C)  transitioning from laminar to turbulent

(D)  inviscid

Answer: (A)

18. Ratio of momentum diffusivity to thermal diffusivity is

(A)  Reynolds number

(B)  Prandtl number

(C)  Nusselt number

(D)  Peclet number

Answer: (B)

19. Mole fraction and activity coefficient of component 1 in a binary liquid mixture are x1 and γ1, respectively. GE is excess molar Gibbs energy of the mixture, R is universal gas constant and T is absolute temperature of the mixture. Which one of the following is always true?

(A) 

(B) 

(C) 

(D) 

Answer: (A)

20. Leidenfrost phenomenon refers to

(A)  the condensation of vapour on a cold surface

(B)  the exchange of heat between two solids

(C)  the melting of frost

(D)  film boiling and evaporation of liquid droplets falling on a very hot surface

Answer: (D)

21. An irreversible gas phase reaction 2P → 4Q + R is conducted in an isothermal and isobaric batch reactor. Assume ideal gas behavior. The feed is an equimolar mixture of the reactant P and an inert gas. After complete conversion of P, the fractional change in volume is _______(round off to 2 decimal places).

Answer: (0.74 to 0.76)

22. Consider two Carnot engines C1 and C2 as shown in the figure.

The efficiencies of the engines C1 and C2 are 0.40 and 0.35, respectively. If the temperature of Reservoir R1 is 800 K, then the temperature (in K) of Reservoir R3 is ___________ (round off to nearest integer).

Answer: (311 to 313)

23. Consider the following closed loop system with Gp and Gc as the transfer functions of the process and the controller, respectively.

For a unit step change in the set point (ysp), the change in the value of response (y) at steady state is _______ (round off to 1 decimal place).

Answer: (0.9 to 1.1)

24. The decomposition of acetalydehyde (X) to methane and carbon monoxide follows four-step free radical mechanism. The overall rate of decomposition of X is  where k1, k2, and k3 denote the rate constants of the elementary steps, with corresponding activation energies (in kJ mol1) of 320, 40 and 0, respectively. The temperature dependency of the rate constants is described by Arrhenius equation. Cx denotes the concentration of acetaldehyde. The rate constant for the overall reaction is koverall. The activation energy for the overall reaction (in kJ mol1) is ________ (round off to nearest integer).

Answer: (199 to 201)

25. Sum of the eigenvalues of the matrix  is _______ (round off to nearest integer).

Answer: (14 to 14)

Q26 – Q55 carry two marks each.

26. In a box, there are 5 green balls and 10 blue balls. A person picks 6 balls randomly. The probability that the person has picked 4 green balls and 2 blue balls is

(A)  42/1001

(B)  45/1001

(C)  240/1001

(D)  420/1001

Answer: (B)

27. The maximum value of the function  in the interval (0.5, 3.5) is

(A)  0

(B)  8

(C)  16

(D)  48

Answer: (C)

28. SO2 from air is absorbed by pure water in a counter current packed column operating at constant pressure. The compositions and the flow rates of the streams are shown in the figure.

In addition, use the following data and assumptions

• Column operates under isothermal conditions

• At the operating temperature of the column, y* = 40x, where yx is the mole fraction of SO2 in the gas that is the equilibrium with water containing SO2 at a mole fraction of x

• Solution is dilute and the operating line is linear

• Negligible amount of water evaporates

The number of transfer units (NTU) for this column is

(A)  0.5

(B)  1.0

(C)  1.5

(D)  2.0

Answer: (D)

29. Two film theory applies for absorption of a solute from a gas mixture into a liquid solvent. The interfacial mass transfer coefficient (in mol m2 s1) for the gas side is 0.1 and for the liquid side is 3. The equilibrium relationship is y* = 2x, where x and y* are mole fractions of the solute in the liquid and gas phases, respectively. The ratio of the mass transfer resistance in the liquid film to the overall resistance is

(A)  0.0161

(B)  0.0322

(C)  0.0625

(D)  0.0645

Answer: (C)

30. Consider the equilibrium data for methanol-water system at 1 bar given in the figure below.

A distillation column operating at 1.0 bar is required to produce 92 mol % methanol. The feed is a saturated liquid. It is an equimolar mixture of methanol and water. The minimum reflux ratio is

(A)  0.33

(B)  0.50

(C)  0.54

(D)  1.17

Answer: (B)

31. Consider the gas phase reaction N2O4 ⇄ 2NO2 occurring in an isothermal and isobaric reactor maintained at 298 K and 1.0 bar. The standard Gibbs energy change of the reaction at 298 K is ∆G°298 = 5253 J mol1. The standard states are those of pure ideal gases at 1.0 bar. The equilibrium mixture in the reactor behaves as an ideal gas. The value of the universal gas constant is 8.314 J mol1 K1. If one of pure N2O4 is initially charged to the reactor, the fraction of N2O4 that decomposes into NO2 at equilibrium is

(A)  0

(B)  0.17

(C)  0.38

(D)  1

Answer: (B)

32. A tank initially contains a gas mixture with 21 mol% oxygen and 79 mol% nitrogen. Pure nitrogen enters the tank, and a gas mixture of nitrogen and oxygen exits the tank. The molar flow rate of both the inlet and exit streams is 8 mol s1.

In addition, use the following data and assumptions

• Assume the tank contents to be well mixed

• Assume ideal gas behavior

• The temperature and pressure inside the tank are held constant

• Molar density of the gas mixture in the tank is constant at 40 mol m3

If the volume of the tank is 20 m3, then t he time (in seconds) required for oxygen content in the tank to decrease to 1 mol % is

(A)  100.45

(B)  304.45

(C)  3.445

(D)  10

Answer: (B)

33. Consider steady, laminar, fully developed flow of an incompressible Newtonian fluid through two horizontal straight pipes I and II, of circular cross section. The volumetric flow rates in both the pipes are the same. The diameter of pipe II is twice the diameter of pipe I, i.e., d­II­ = 2dI. The ratio of the shear stress at the wall of pipe I to the shear stress at the wall of pipe II is

(A)  0.5

(B)  2

(C)  4

(D)  8

Answer: (D)

34. Equilibrium data for a binary mixture of E and F at two different pressures is shown in the figure

It is desired to process a feed containing 80 mol % E and 20 mol % F, and obtain a product with a purity of 99.5 mol % E. A sequence of two distillation columns, cone operating at pressure P1 and another at P2, is employed for this operation, as shown below.

Mole fraction of E in the distillate obtained from column 1 is 0.9. If the column pressures P1 and P2 are in kPa, which one of the following is correct?

(A)  P1 = 100, P2 = 20, and high purity E is recovered from the top of column 2

(B)  P1 = 100, P2 = 20, and  high purity E is recovered from the bottom of column 2

(C)  P1 = 20, P2 = 100, and high purity E is recovered from the top of column 2

(D)  P1 = 20, P2 = 100, and high purity E is recovered from the bottom of column 2

Answer: (D)

35. A hollow cylinder of equal length and inner diameter (i.e., L = D) is sealed at both ends with flat plate, as shown in the figure. Its inner surfaces, A1, A2, and A3 radiate energy.

Fij denotes the fraction of radiation energy leaving the surface Ai which reaches the surface Aj. It is also known that F13 = 3 – 2√2. Which one of the following is correct?

(A)  F21 = √2 – 1

(B) 

(C) 

(D) 

Answer: (B)

36. A student performs a flow experiment with Bingham Plastic under fully developed laminar flow conditions in a tube of radius 0.01 m with a pressure drop (∆P) of 10 kPa over tube length (L) of 1.0 m. The velocity profile is flat for r < rc and parabolic for r ≥ rc, as shown in the figure.

Consider r and x as the radial and axial directions, and the shear stress is finite as r approaches zero. A force balance results in the following equation  where τrx is the shear str3ess. If r­c is 0.001 m, then t he magnitude of yield stress for this Bingham Plastic (in Pa) is

(A)  1

(B)  5

(C)  8

(D)  12

Answer: (B)

37. A feed stream containing pure species L flows into a reactor, where L is partly converted to M as shown in the figure.

The mass flow rate of the recycle stream is 20% of that of the product stream. The overall conversion of L (based on mass units) in the process is 30%. Assuming steady state operation, the one-pass conversion of L (based on mass units) through the reactor is

(A)  34.2%

(B)  30%

(C)  26.3%

(D)  23.8%

Answer: (C)

38. A U-tube manometer contains two manometric fluids of densities 1000 kg m3 and 600 kg m3. When both the limbs are open to atmosphere, the difference between the two levels is 10 cm at equilibrium, as shown in the figure.

The rest of manometer is filled with air of negligible density. The acceleration due to gravity is 9.81 ms2 and the atmospheric pressure is 100 kPa. How much absolute pressure (in kPa) has to be applied on the limb ‘P’ to raise the fluid in the limb ‘Q’ by another 20 cm?

(A)  100.175

(B)  103.924

(C)  547.231

(D)  833.206

Answer: (B)

39. A pure gas obeys the equation of state given by where P is the pressure, T is the absolute temperature, V is the molar volume of the gas, R is the universal gas constant, and B, is a parameter independent of T and P. The residual molar Gibbs energy, GR, of the gas is given by the relation  where Z is the compressibility factor and the integral is evaluated at constant T. If the value of B is 1 × 104 m3 mol1, the residual molar enthalpy (in J mol1) of the gas at 1000 kPa and 300 K is

(A)  100

(B)  300

(C)  2494

(D)  30000

Answer: (A)

40. Consider one mole of an ideal gas in a closed system. It undergoes a change in state from L to N through two different non-isothermal processes, as shown in the P-V diagram (where P is the pressure and V is the molar volume of the gas). Process I is carried out in a single step, namely LN, whereas process II is carried out in two steps, namely LM and MN. All the steps are reversible.

The net heat flowing into the system for process I is QI and that for process II is Qu. The value of QI – QII is

(A)  250

(B)  500

(C)  1000

(D)  1500

Answer: (B)

41. A fluid is heated from 40°C to 60°C in a countercurrent, double pipe heat exchanger. Hot fluid enters at 100°C and exits at 70° The log mean temperature difference, i.e. LMTD (in °C), is _______ (round off to 2 decimal places).

Answer: (34020 to 35.20)

42. Consider an infinitely long rectangular fin exposed to a surrounding fluid at a constant temperature Ta = 27°C

The steady state one dimensional energy balance on an element of the fin of thickness dx at a distance x from its base yields   where θ = Tx – Ta, Tx is the temperature of the fin at the distance x from its base in °C. The value of m is 0.04 cm1 and the temperature at the base is T0 = 227°C. The temperature (in °C) at x = 25 cm is _______ (round off to 1 decimal place).

Answer: (100.0 to 101.5)

43. Liquid water is pumped at a volumetric flow rate of 0.02 m3 s1 form Tank I to Tank II, as shown in the figure.

Both the tanks are open to the atmosphere. The total frictional head loss for the pipe system is 1.0 m of water.

In addition, use the following data and assumptions

• Density of water is 1000 kg m3

• Acceleration due to gravity is 9.81 m s2

• Efficiency of the pump is 100%

• The liquid surfaces in the tanks have negligible velocities

The power supplied (in W) by the pump to lift the water is ________ (round off to 1 decimal place).

Answer: (1175.0 to 1180.0)

44. An elementary liquid phase reversible reaction P ⇄ Q is carried out in an ideal continuous stirred tank reactor (CSTR) operated at steady state. The rate of consumption of P, −rP (in mol liter1 minute1), is given by –rP = CP – 0.5CQ where CP and CQ are the concentrations (in mol liter1) of P and Q, respectively.

The feed contains only the reactant P at a concentration of 1 mol liter1, and the conversion of P at the exit of the CSTR is 75% of the equilibrium conversion. Assume that there is no volume change associated with the reaction, and the temperature of the reaction mixture is constant throughout the operation. The space time (in minutes) of the CSTR is ________ (round off to 1 decimal place).

Answer: (1.9 to 2.1)

45. An aqueous suspension at 60°C is fed to the first effect of a double effect forward feed evaporator with a mass flow rate of 1.25 kg s1. The sum of the rates of water evaporated from the first and second effects is 1.0 kg s1. Temperatures of the exit streams from the first and the second effects are 100°C, and 60°C, respectively. Consider the specific heat of the aqueous suspension, and the latent heat of phase change for water to be 4 kJ kg1 K1 and 2200 kJ kg1, respectively, over this temperature range. The steam economy (in kg per kg) is _______ (round off to 2 decimal places).

Answer: (1.70 to 1.90)

46. A vertically held packed bed has a height of 1 m, and a void fraction of 0.1, when there is not flow through the bed. The incipient (minimum) fluidization is set in by injection of a fluid of density 1 kg m3. The particle density (ρp) of the solids is 3000 kg m3. Acceleration due to gravity is 9.81 ms2. The pressure drop (in Pa) across the height of the bed is ________ (round off to nearest integer).

Answer: (26450 to 26500)

47. Two ideal cross-current stages operate to extract P from a feed containing P and Q, as shown below.

The mass flow rates of P and Q fed to Stage 1 are, 1,000 kg h1 and 10,000 kg h1, respectively. Pure solvent (S) is injected at mass flow rates of 5,000 kg h1 and 15,000 kg h1 to Stages 1 and 2, respectively. The components Q and S are immiscible. The equilibrium relation is given by Y* = 1.5X, where X is the mass o P per unit mass of Q in the raffinate, and Y* is the mass of P per unit mass of S in the extract, which is in equilibrium with the raffinate. The mass flow rate of P (in kg h1) in the raffinate from Stage 2 is ______ (round off to nearest integer).

Answer: (171 to 181)

48. Consider a vertically falling film of water over an impermeable wall. The film is in contact with a static pool of non-reactive pure gas. The gas diffuses into the water film over the entire height of the falling film. The height of the film is 1.0 m, and its thickness is  104 The velocity of water, averaged over the film thickness, is 0.01 ms1. The gas concentration (in kg m3), averaged over the film thickness is where y is the vertical position in meters measured from the top of the wall.

In addition, use the following data and assumptions

• The flow is fully developed

• The width of the film is much larger than the thickness of the film, and the dissolved gas concentration is invariant over this width

• The solubility of the gas in water, CAi, is constant

• Pure water enters at y = 0

• The evaporation of water is negligible

The mass transfer coefficient on the liquid side (in mm s1), averaged over the entire height of the falling film is ________ (round off to 3 decimal places).

Answer: (0.025 to 0.035)

49. An exothermic, aqueous phase, irreversible, first order reaction, Y → Z is carried out in an ideal continuous stirred tank reactor (CSTR) operated adiabatically at steady state. Rate of consumption of Y (in mol liter1 minute1) is given by  where Cγ is the concentration of Y (in mol liter1), and T is the temperature of the reaction mixture (in K). Reactant Y is fed at 50° Its inlet concentration is 1.0 mol liter1, and its volumetric flow rate is 1.0 liter minute1.

In additions, use the following data and assumptions

• Heat of the reaction = −42000 J mol1

• Specific heat capacity of the reactions mixture = 4.2 J g1 K1

• Density of the reaction mixture = 1000 g liter1

• Heat of the reaction, specific heat capacity and density of the reaction mixture do not vary with temperature

• Shaft work is negligible

If the conversion of Y at the exit of the reactor is 90%, the volume of the CSTR (in liter) is _______ (round off to 2 decimal places).

Answer: (2.40 to 3.00)

50. The liquid phase irreversible reactions,  are carried out in an ideal continuous stirred tank reactor (CSTR) operating isothermally at steady state. The space time of the CSTR is 1 minute. Both the reactions are first order with respect to the reactant P, and k1 and k2 denote the rate constants of the two reactions. At the exit of the reactor, the conversion of reactant P is 60%, and the selectivity of Q with respect to R is 50%. The value of the first order rate constant k1 (in minute1) is _______ (correct  up to one decimal place).

Answer: (0.49 to 0.51)

51. A catalytic gas phase reaction P → Q is conducted in an isothermal packed bed reactor operated at steady state. The reaction is irreversible and second order with respect to the reactant P. The feed is pure P with a volumetric flow rate of 1.0 liter minute1 and concentration of 2.0 mol liter1.

In addition, use the following assumptions

• The reactant and product are ideal gases

• There is no volume change associated with the reaction

• Ideal plug flow conditions prevail in the packed bed

When the mass f catalyst in the reactor is 4 g, the concentration of P measured at the exit is 0.4 mol liter1. The second order rate constant (in liter2 g1catalyst mol1 minute1) is _______ (correct up to one decimal place).

Answer: (0.49 to 0.51)

52. Flow of water through an equal percentage vale is 900 liter h1 at 30% opening, and 1080 liter h1 at 35% opening. Assume that the pressure drop across the valve remains constant. The flow rate (in liter h1) through the valve at 45% opening is ________ (round off to nearest integer).

Answer: (1540 to 1580)

53. Consider the following closed loop system.

Gc, Gf, and Gp are the transfer functions of the controller, the final control element and the process, respectively. y and ysp are the response and its set point, respectively. For a gain margin of 1.6, the design value of KC is _______ (correct  up to one decimal place).

Answer: (0.49 to 0.51)

54. Given  the value of y at x = 2 is _______ (round off to nearest integer).

Answer: (162 to 170)

55. Consider the following dataset.

The value of the integral  using Simpson’s 1/3rd rule is ________ (round off to 1 decimal place).

Answer: (241.4 to 242.4)

© Copyright Entrance India - Engineering and Medical Entrance Exams in India | Website Maintained by Firewall Firm - IT Monteur